Tải bản đầy đủ (.pdf) (140 trang)

Tuyển chọn các bài toán hình học hay và khó bồi dưỡng HSG và luyện thi vào lớp 10 chuyên

Bạn đang xem bản rút gọn của tài liệu. Xem và tải ngay bản đầy đủ của tài liệu tại đây (5.23 MB, 140 trang )

<span class='text_page_counter'>(1)</span><div class='page_container' data-page=1>





<b> </b>



<b>CÁC BÀI TỐN HÌNH HỌC HAY</b>


<b>TRUNG HỌC CƠ SỞ</b>



</div>
<span class='text_page_counter'>(2)</span><div class='page_container' data-page=2>

<b>TUYỂN CHỌN CÁC BÀI HÌNH HAY VÀ KHĨ</b>


<b>BỒI DƯỠNG HỌC SINH GIỎI TỐN 9VÀ LUYỆN THI VÀO LỚP 10 CHUYÊN </b>


<b>TOÁN </b>


Hình học phẳng là một nội dung quan trọng trong chương trình mơn toán ở
trường THCS cũng như THPT chuyên toán. Trong những năm gần đầy các bài tốn


về hình học phẳng xuất hiện trong các đề thi vào lớp 10 THPT, lớp 10 năng khiếu toán
và trong các kì thi học sinh giỏi các cấp với độ khó ngày càng cao. Với mong muốn
tuyển chọn ra các bài hình hay và khó nhằm mục đích làm tài liệu học tập cho học


sinh và tài liệu giảng dạy cho giáo viên, chúng tôi đã soạn ra cuốn tài liệu ”<b>Tuyển </b>
<b>chọn các bài hình hay và khó bồi dưỡng học sinh giỏi tốn 9 và luyện thi vào lớp 10 </b>


<b>chuyên toán</b>”. Nội dung cơ bản của tài liệu là giới thiệu các bài toán hình học phẳng
mà bản thân tác giả thấy hay và khó, cùng với lời giải được trình bày cơng phu và
chính xác. Với cách viết đặt bạn đọc vào vị trí người giải, lối suy nghĩ hình thành lời
giải bài toán một cách tự nhiên nhưng vẫn đảm bảo tính khoa học, hy vọng cuốn tài
liệu sẽ thực sự có ích cho bạn đọc trên con được chinh phục các bài toán hình học
phẳng. Mặc dù chúng tơi đã thực sự cố gắng và dành nhiều tâm huyết để hoàn thiện
cuốn sách với hiệu quả cao nhất, song sự sai sót là điều khó tránh khỏi. Chúng tơi rất
mong được sự đónggóp ý kiến của bạn đọc để chúng tơi hồn thiện tài liệu tốt hơn.



<b>Bài 1. </b>Đường tròn ngoại tiếp và đường tròn nội tiếp tam giác ABC có bán kính lần
lượt là R và r. Biết rằng BAC ACB ABC BAC− = − . Tính diện tích tam giác ABC theo R


và r.


<b>Lời giải</b>


Gọi O và I lần lượt là tâm đường tròn ngoại tiếp và
tâm đường tròn nội tiếp tam giác ABC và gọi D, E, F
lần lượt là các tiếp điểm của đường trịn

( )

I với AB,
AC, BC. Khi đó ta có


(

)



(

)



ABC


1 1 1 1


S AB.r AC.r BC.r r AB AC BC


2 2 2 2


1


r AD AE BD BF CE CF
2



= + + = + +


= + + + + + <sub>M</sub>


F
E
D


H
I <sub>O</sub>


C
B


</div>
<span class='text_page_counter'>(3)</span><div class='page_container' data-page=3>

Mà ta có AD AE, BD BF,CE CF= = = nên S<sub>ABC</sub> 1r 2.AD 2 BF CF

(

)

r AD BC

(

)



2  


= <sub></sub> + + <sub></sub>= + .


Ta có BAC ACB ABC BAC− = − nên 2BAC ABC ACB= + . Do đó 0


3BAC 180= nên ta


được BAC 60= 0 suy ra DAI=300.


Trong tam giác vng DAI có <sub>AD DI.cot DAI r.cot 30</sub>= = 0 =<sub>r 3</sub><sub>. </sub>


Kẻ đường kính AM của đường trịn

( )

O , khi đó ta được OAB OBA=



Từ đó suy ra BOM=OAB OBA+ hay BOM=2.OAB. Tương tự ta có COM=2.OAC
Suy ra BOM COM+ =2 OAB OAC

(

+

)

nên BOC=2.BAC 2.60= 0 =1200


Kẻ OH vng góc với BC ta được HB HC BC
2


= = và <sub>HOC</sub> 1<sub>BOC 60</sub>0


2


= =


Trong tam giác OHC có HC OC.sin HOC R.sin 600 R 3
2


= = =


Từ đó suy ra BC=2.HC R 3= nên ta được S<sub>ABC</sub> =r r 3 R 3

(

+

)

= 3.r R r

(

+

)



<b>Bài 2. </b>Cho tam giác ABC nhọn có AB AC nội tiếp đường tròn

( )

O . Gọi H là trực
tâm của tam giác và AH vng góc với BC tại F. Gọi M là trung điểm của BC. Trên
đường tròn

( )

O lấy các điểm Q và K sao cho <sub>HQA HKQ 90</sub>= = 0 <sub>(Các điểm A, B, C, K, </sub>


Q theo thứ tự đó trên đường trịn). Chứng minh rằng đường tròn ngoại tiếp tam giác
KHQ tiếp xúc với đường tròn ngoại tiếp tam giác MFK.


<b>Lời giải</b>


<b>Lời giải 1. </b>Do <sub>HQA HKQ 90</sub>= = 0<sub> nên các </sub>



đường tròn ngoại tiếp các tam giác AHQ
và QHK lần lượt nhận AH và QH đường
kính. Gọi AD là đường kính của đường


trịn

( )

O và E là điểm đối xứng với H qua
BC. Khi đó dễ thấy tứ giác HBDC là hình


hình hành nên ba điểm H, M, D thẳng
hàng. Mà ta lại có HQA 90= 0 nên suy ra


ba điểm Q, H, D thẳng hàng. Từ đó ta
được các điểm Q, H, M, D thẳng hàng.


D E


F


Q


H
A


B <sub>C</sub>


O


K


M X



</div>
<span class='text_page_counter'>(4)</span><div class='page_container' data-page=4>

tiếp tam giác QHK cắt nhau tại X. Khi đó ta có XKH XHK KQH= = . Mà ta lại có


KQH KQD KAD= = . Từ đó ta có biến đổi góc


(

)



(

)



0 0 0


0


KXH 180 2KHX 180 2KQH 2 90 KQH


2 90 KAD 2ADK 2KEH


= − = − = −


= − = =


Lại có XH=XK nên X là tâm đường tròn ngoại tiếp tam giác KHE. Do E và H đối
xứng nhau qua BC nên BC là đường trung trực của DH, từ đó suy ra X thuộc đường
thẳng BC. Do XH là tiếp tuyến của đường tròn đường kính QH nên ta có XH vng
góc với QH tại H. Tam giác XHM vng tại H có đường cao HF nên


2 2


XK =XH =XF.XM.Từ đó suy ra XK là tiếp tuyến chung của hai đường tròn ngại
tiếp tam giác KQH và KFM với tiếp điểm K là giao điểm của hai đường trịn. Do đó
suy ra hai đường tròn ngại tiếp tam giác KQH và KFM tiếp xúc nhau K.



<b>Lời giải 2.</b> Do <sub>HQA HKQ 90</sub>= = 0 <sub>nên các đường tròn ngoại tiếp các tam giác AHQ và </sub>


QHK lần lượt nhận AH và QH đường kính. Gọi AD là đường kính của đường tròn


( )

O và E là điểm đối xứng với H qua BC. Khi đó dễ thấy tứ giác HBDC là hình hình
hành nên ba điểm H, M, D thẳng hàng. Mà ta lại có <sub>HQA 90</sub>= 0 <sub>nên suy ra ba điểm Q, </sub>


H, D thẳng hàng. Từ đó ta được các điểm Q, H, M, D thẳng hàng. Kẻ đường kính PQ
của đường trịn

( )

O , khi đó do <sub>HKQ 90</sub>= 0 <sub>nên suy ra ba điểm P, H, K thẳng hà</sub><sub>ng. </sub>


Đến đây ta có biến đổi góc PKE PQE OQE 900 1QOE 900 QBE
2


= = = − = − Ta dễ dàng


chứng minh được DE song song với BC nên ta lại có


(

) (

)



1 1 1


QMC EMC sdQC sdBD sdQC sdCE sdQE QBE


2 2 2


= = + = + = = .


Để ý rằng <sub>MHE 90</sub>= 0−<sub>QMC 90</sub>= 0−<sub>QBE</sub><sub>, từ đó ta suy ra được </sub><sub>PKE</sub>=<sub>MHE</sub><sub> nên DQ </sub>



là tiếp tuyến tại tại H của đường tròn ngoại tiếp tam giác KHE. Giả sử đường thẳng
vuông góc với HQ tại H cắt BC tại X, khi đó đường thẳng HX đi qua tâm đường trịn
ngoại tiếp tam giác KHE. Mặt khác dễ thấy BC là đường trung trực của HE nên


XE=XH, từ đó suy ra X là tâm đường tròn ngoại tiếp tam giác KHE, do đó


</div>
<span class='text_page_counter'>(5)</span><div class='page_container' data-page=5>

trịn ngoại tiếp tam giác KHQ nên suy ra XK là tiếp tuyến tại K với đường tròn ngoại
tiếp tam giác KHQ. Áp dụng hệ thức lượng cho tam giác MHX có đường cao HF ta có


2 2


XK =XH =XF.XM nên XK cũng là tiếp tuyến tại K với đường tròn ngoại tiếp tam
giác KMF. Vậy đường tròn ngoại tiếp tam giác KHQ tiếp xúc với đường tròn ngoại
tiếp tam giác MFK.


<b>Bài 3. </b>Cho tam giác ABC có ba đường cao AD, BE, CF. Gọi G, P lần lượt là hình chiếu
của D trên AB và AC. Gọi I, K lần lượt là hình chiếu của E trên AB và BC. Gọi M, N
lần lượt là hình chiếu của F trên AC và BC. Chứng minh rằng sáu điểm G, P, I, K, M,


N cùng nằm trên một đường tròn.


<b>Lời giải</b>
Ta xét các trường hợp sau.


• <b>Trường hợp 1. </b>Tam giác ABC vng. Khơng mất tính tổng qt ta giả sử tam giác
ABC vng tại A, khi đó ba điểm A, E, F trùng nhau. Ba điểm N, D, K trùng nhau.
Các điểm I và M trùng với A. Khi đó ta có tứ giác AGDP là hình chữ nhật nên các
điểm A, G, P, D cùng nằm trên một đường tròn. Từ đó suy ra G, P, I, K, M, N.


• <b>Trường hợp 2.</b> Tam giác ABC có ba góc nhọn.


Khi đó dễ thấy tứ giác AGDP nội tiếp đường tròn.


Suy ra ta được GAD GPD= nên 0


GPC GBC 180+ =


nên tứ giác BCPG nội tiếp đường tròn. Dễ thấy các
tứ giác BCEF và EFIM nội tiếp đường tròn nên ta
được FEC FBC 180+ = 0. Do đó FEC GPC= nên EF


song song với GP. Mặt khác 0


MIF MEF 180+ = nên
suy ra MIF MPG 180+ = 0.


P
M
I


G


K
N


F


E


D C



B


A


Do đó tứ giác GPMI nội tiếp được trong đường tròn

( )

O<sub>1</sub> . Tương tự ta chứng minh
được tứ giác MNPK nội tiếp trong đường trịn

( )

O<sub>2</sub> . Hồn tồn tương tự như trên ta
được tứ giác AIKC nội tiếpđường tròn. Từ tứ giác BCPG nội tiếp đường tròn ta được


MPG=GBC kết hợp với MIF MPG 180+ = 0 suy ra MIF GBC 180+ = 0 nên IM song


song với BC. Tương tự ta được LP song song với AB. Từ đó ta được


0


</div>
<span class='text_page_counter'>(6)</span><div class='page_container' data-page=6>

KPMI nội tiếp đường tròn

( )

O<sub>3</sub> . Qua ba điểm không thẳng hàng chỉ xác định duy
nhất một đường tròn nên ba đường tròn

( )

O<sub>1</sub> ,

( )

O<sub>2</sub> và

( )

O<sub>3</sub> trùng nhau. Vậy sáu
điểm G, P, I, K, M, N cùng nằm trên một đường trịn.


• <b>Trường hợp 3.</b> Tam giác ABC là tam giác tù. Chứng minh hoàn toàn tương tự ta
cũng được sáu điểm G, P, I, K, M, N cùng nằm trên một đường tròn.


Vậy sáu điểm G, P, I, K, M, N luôn cùng nằm trên một đường tròn.


<b>Bài 4.</b> Cho tam giác ABC có 0


BAC=30 . Đường phân giác trong và phân giác ngoài


của góc ABC cắt cạnh AC lần lượt tại B , B<sub>1</sub> <sub>2</sub>. Đường phân giác trong và phân giác


ngồi của góc ACB cắt cạnh AB lần lượt tại C ,C<sub>1</sub> <sub>2</sub>. Đường tròn ngoại tiếp tam giác



1 2


BB B cắt đường tròn ngoại tiếp tam giác CC C<sub>1</sub> <sub>2</sub> tại điểm P ở trong tam giác ABC.
Gọi O là trung điểm B B<sub>1</sub> <sub>2</sub>. Chứng minh rằng CP vng góc với BP


<b>Lời giải</b>


Do BB ; BB<sub>1</sub> <sub>2</sub> lần lượt là đường phân giác
trong và phân giác ngoài của ABC nên


0
1 2


B BB =90 . Khi đó tam giác BB B1 2 là tam


giác vuông tại B. Do O là trung điểm của


1 2


B B nên O là tâm đường tròn ngoại tiếp


tam giác BB B<sub>1</sub> <sub>2</sub>.


P


O


C2



B2


C1


B1 C


B


A


Do vậy ta có OBC OBB= <sub>1</sub>−CBB<sub>1</sub> =BB O B BA<sub>1</sub> − <sub>1</sub> =BAC. Từ đó suy ra hai tam giác


OBA và OCB đồng dạng với nhaunên ta được OB OA


OC= OB hay


2


OA.OC OB= . Do đó ta
được OA.OC OP= 2 hay OA OP


OP =OC nên hai tam giác OPC và OAP đồng dạng với
nhau, từ đó suy ra OPC PAC= . Ta có biến đổi góc như sau


(

1 1

) (

1 1

)

1 1


PBC PBA PBB B BC ABB PBB 2PBB POB


PCA OPC PCA PAC



− = + − − = =


= − = −


Do đó suy ra PAC PBC PBA PCA+ = + . Tương tự ta được PAB PCB PBA PCA+ = + .


</div>
<span class='text_page_counter'>(7)</span><div class='page_container' data-page=7>

PAC PBC PAB PCB PBA PCA PBA PCA+ + + = + + +


Từ đó suy ra 0

(

) (

)



180 − PBA PCA+ =2 PBA PCA+ hay 0


PBA PCA+ =60 . Mà theo giả
thiết thì BAC=300 nên suy ra PBC PCB 90+ = 0 hay PB và PC vng góc với nhau.


<b>Bài 5.</b>Cho tam giác nhọn ABC không cân nội tiếp đường tròn tâm O. Lấy điểm P trên
cạnh AB sao cho BOP=ABC và lấy điểm Q trên cạnh AC sao cho COQ ACB= .


Chứng minh rằng đường thẳng đối xứng với BC qua PQ là tiếp tuyến của đường tròn
ngoại tiếp tam giác AQP.


<b>Lời giải</b>
Giả sử đường thẳng AO cắt BC tại D. Do O
là tâm đường tròn nội tiếp tam giác ABC


nên OA OB= . Từ đó ta được OAB OBA= .


Mà theo giả thiết ta có OAB ABC= nên ta


suy ra được hai tam giác ABD và BOP đồng


dạng với nhau.


O
D


Q
P


M
E


C
B


A


Suy ra ta có AB AD


BO = BP hay AB.BP AD.BO= và BPO=ADB. Tương tự ta cũng có hai


tam giác ACD và COQ đồng dạng với nhau.Từ đó ta cũng được AC.CQ AD.CO= và
CQO ADC= . Do đó ta có biến đổi góc


(

)



0 0 0 0


APO AQO 180+ = −BPO 180+ −CQO=360 − ADB ADC+ =180


Suy ra tứ giác APOQ nội tiếp đường tròn hay điểm O nằm trên đường tròn nội tiếp


tam giác APQ. Gọi E là giao điểm của đường tròn ngoại tiếp tam giác APQ với đường
trịn ngoại tiếp tam giác ABC. Từ đó ta có OE OA= nên suy ra OAE=OEA và tứ giác
AEQO nội tiếp đường tròn nên AEO AQO= . Kết hợp với từ giác AQOP nội tiếp
đường tròn ta suy ra được AEO AQO BPO ADB= = = . Từ đó ta có EAO=ADB nên


AE song song với BC. Mà ta lại có OB OC= nên AB.BP AC.CQ= .


</div>
<span class='text_page_counter'>(8)</span><div class='page_container' data-page=8>

<b>Nguyễn Công Lợi Website: tailieumontoan.com</b>


Do đó EPQEAQ;EAQACB nên EPQ ACB. Từ đó suy ra hai tam giác EPQ và ACB
đồng dạng với nhau. Điều này dân đến hai tam giác ABC và EBC bằng nhau nên
ta được APE APQ EPQ AOQ EOQ AOB= − = − = .


Mặt khác ta lại có OAC 1

(

1800 AOC

)

900 ABC
2


= − = − và EAC=ACB


Từ đó suy ra 0 0

(

0

) (

)



AOB 180= −2OAE 180= −2 90 −ABC ACB+ =2 ABC ACB−


Do đó ta được APE=2 ABC ACB

(

)

.


Ta lại có ABE=ABC EBC ABC ACB− = − nên suy ra APE 2ABE= hay PBE PEB=


Ta cũng có AQP=AEP 180= 0−

(

BAC ACB+

) (

− ABC ACB−

)

=2ACB ABC−


Nên ta được PMB ACB MQP ACB AQP ABC ACB= − = − = −



Suy ra PMB=BEP=ABC ACB− nên tứ giác BPEM nội tiếp đường tròn.


Kết hợp với PBE PEB= ta được BMP=PME nên MP là phân giác của góc BME hay


nọi cách khác thì hai đường thẳng ME và BC đối xứng với nhau qua PQ.


Lại có EQM 180= 0−EQP 180= 0−ABC và PEM 180= 0−ABC. Từ đó dẫn đến hai tam
giác MQE và MPE đồng dạng với nhau, do đó ta được ME2 =MP.MQ. Suy ra ME là


tiếp tuyến của đường tròn ngoại tiếp tam giác APQ.


<b>Bài 6.</b>Cho tam giác ABC nội tiếp đường tròn

(

O; R

)

. Kẻ các đường cao BE và CF của
tam giác ABC. Gọi M là trung điểm của BC. Gọi I là tâm đường tròn nội tiếp tam giác
ABC và D là giao điểm của AI với đường trịn

( )

O . Tìm các giá trị của k để ID kOA=


biết rằng tam giác MEF đều.


</div>
<span class='text_page_counter'>(9)</span><div class='page_container' data-page=9>

D
F


E
M


I
O


C
B


A



M
I


D
O


F
E


C
B


A


Ta có I là tâm đường tròn nội tiếp nên DB DC DI= = . Do M là trung điểm của BC nên


MB MC ME= = . Do đó khi BAC 90= 0 thì EF, điều này trái với giả thiết. Vậy ta


được 0


BAC90 . Ta xét các trường hợp sau.
• <b>Trường hợp 1.</b> Nếu 0


BAC 90 . Khi đó ta có 0

(

)



EMF 180= − BMF CME .+ Tam giác
BEC và BFC vuông nên tam giác BMF và MCE là tam giác cân.


Do đó 0



BMF 180= −2FBM và 0


CME 180= −2ECM


Suy ta 2EMF 2 FBM ECM 90=

(

+ − 0

) (

=2 1800−BAC 90− 0

)

=1800 −2BAC


Mà ta lại có 0


EMF 60= nên ta được 0 0


180 −2BAC 60 .=


Do đó 0


BAC 60= nên suy ra 0


BAD 30= . Từ đó ta có 0


BOD 60= . Do đó ta được


BI R OA= = hay DI=R nên k 1= .
• <b>Trường hợp 2.</b>Nếu 0


BAC 90 . Khi đó ta có


(

)

(

)



(

) (

)




(

)

(

) (

)



0 0 0 0


0 0 0 0


0 0 0 0


EMF 180 BME CMF 180 180 2EBC 180 2FCB


2 EBC FCB 90 2 90 ECB 90 FBC 90


180 2 ECB FBC 180 2 180 BAC 2 BAC 90


= − + = − − + −


= + − = − + − −


= − + = − − = −


Mà ta có EMF 60= 0 nên BAC 90− 0 =300suy ra BAC 120= 0. Suy ra BAD 60= 0 nên ta


được BOD 120= 0 hay MBD 60= 0. Từ đó ta được BD=R 3 hay ID=R 3 nên suy ra
k= 3.


</div>
<span class='text_page_counter'>(10)</span><div class='page_container' data-page=10>

<b>Bài 7.</b> Cho tam giác nhọn ABC nội tiếp đường tròn

( )

O . Các đường cao AD, BE, CF
cắt nhau tại H. Đường thẳng EF cắt đường tròn

( )

O tại M và N(F nằm giữa M và E).
Chứng minh rằng AM là tiếp tuyến của đường tròn ngoại tiếp tam giác MDH.


<b>Lời giải</b>



Trong nửa mặt phẳng bờ AB không chứa điểm C vẽ tia tiếp tuyến Ax với đường trịn


( )

O . Khi đó ta có ACB=BAx. Mặt khác 0


BFC=BEC 90= nên tứ giác BCEF nội tiếp
đường tròn. Suy ra ta được BFE=ACB. Do đó ta được BAx BFE= nên suy ra Ax


song song với EF.


Mà ta có OA vng góc với Ax nên ta suy ra OA
vng góc với EF. Từ đó suy ra AM=AN nên ta


được ABM=AMF. Xét hai tam giác ABM và
AMF có BAM là góc chung và ABM=AMF nên
hai tam giác ABM và ÀM đồng dạng với nhau.
Do đó tađược AB AM


AM= AF hay


2


AM =AB.AF.


O
x


H


N



M
F


E


D C


B


A


Xét hai tam giác AFH và ADB có FAH là góc chung và AFH ADB= nên hai tam giác


AFH và ADB đồng dạng với nhau.Từ đó ta được AF AH


AD= AB hay AB.AF AH.AD= .


Kết hợp các kết quả trên ta được <sub>AM</sub>2=<sub>AH.AD</sub><sub> nên </sub>AM AH


AD =AM. Suy ra hai tam giác


AMH và ADM đồng dạng với nhau, do đó ta được AMH=ADM. Vẽ tia tiếp tuyến
Mt của đường tròn ngoại tiếp tam giác ADH, tia Mt nằm trên nửa mặt phẳng bờ MH


không chứa điểm D. Khi đó ta có HMt=HDM nên ta suy ra được HMt=HMA, điều
này dẫn đến hai tia Mt và MA trùng nhau hay MA là là tiếp tuyến của đường tròn
ngoại tiếp tam giác MDH.


<b>Bài 8. </b>Cho tam tứ giác ABCD nội tiếp đường trịn

( )

O có E là giao điểm của hai

đường chéo AC và BD. Chứng minh rằng nếu ABD 60= 0 và AE 3CE= thì tứ giác


ABCD ln có AB CD AD BC+ = + hoặc AB BC AD CD+ = + .


</div>
<span class='text_page_counter'>(11)</span><div class='page_container' data-page=11>

Vẽ DH vng góc với AB tại H, DK vng góc
với BC tại K.


Ta có <sub>AH AD.cosHAD AD.cos 60</sub>0 AD


2


= = = và


0 3AD


DH AD.sin HAD AD.sin 60


2


= = = .


Tam giác HBD vuông tại H nên theo định lí


Pitago ta có 2 2 2


BD =BH +DH nên suy ra


E
K



H
D


C


B
A


2
2


2 AD 3AD


BD AB


2 2


 


 


=<sub></sub> − <sub></sub> + <sub></sub> <sub></sub>


  <sub></sub> <sub></sub>


Áp dụng định lí Cosin cho tam giác ABD ta được


2 2 2 2 2


BD =AB +AD −2AB.AD.cosBAD AB= +AD −AB.AD



Trong tam giác DCK vuông tại K có DCK=600 nên CK CD
2


= và DK 3.CD
2


= .


Từ đó ta được


2
2


2 2 2 CD 3CD 2 2


BD BK KD BC BC CD BC.CD


2 2


 


 


= + =<sub></sub> + <sub></sub> +<sub></sub> <sub></sub> = + +


  <sub></sub> <sub></sub>


Do đó ta được AB2+AD2−AB.AD BC= 2+CD2+BC.CD. Mặt khác dễ thấy hai tam
giác EAD và EBC đồng dạng với nhau nên ta được AD EA



BC = EB . Lại có hai tam giác


EBA và ECD đồng dạng với nhau nên AB EB


CD =EC. Kết hợp với giả thiết AE 3CE= ta


được AD AB. EA EB. 3


BC CD = EB EC = hay AD.AC 3BC.CD= . Từ đó ta được


(

) (

)



(

)



2 2


2 2 2 2


AB AD 2AB.AD BC CD 2BC.CD AB AD BC CD


AB BC AD CD


AB AD BC CD


AB CD AD BC


+ − = + −  − = −


 + = +



 − =  − <sub> </sub>


+ = +




Vậy ta có điều phải chứng minh.


<b>Bài 9.</b> Cho hình bình hành ABCD có A900 và M là trung điểm BC. Đường thẳng
AM cắt đường tròn ngoại tiếp tam giác ABC tại N. Gọi H là hình chiếu của C trên
cạnh AB. Chứng minh rằng tứ giác ADNH nội tiếp đường tròn.


</div>
<span class='text_page_counter'>(12)</span><div class='page_container' data-page=12>

Theo giả thiết ta có MB MC= và CH vng góc


với AB. Từ đó ta được MH MB= . Đồng thời
cũng có BAN=BCN và AMB CMN= nên ta


được hai tam giác AMB và CMN đồng dạng với


nhau. Từ đó suy ra MB AB


MN=CN. Mà tứ giác ABCD
là hình bình hành nên AB CD= .


O
H


N



M C


D


B
A


Do đó từ hệ thức trên ta có MN CD


MH= CN. Mặt khác ta lại có HMB 2HCB= nên suy ra


(

)

(

)



(

o

) (

)



1


HMN HMB BMN HMB BN AC 2HCB BAN ANC


2


2 90 ABC BAN ANC


= + = + + = + +


= − + +


Mà ta lại có ABC=ANC. Do đó ta suy ra được


0



HMN 180= −ABC BAN+ =BAC BAN+ =BCD BCN+ =DCN


Đến đây suy ra hai tam giác MHN và CDN đồng dạng với nhau nên CDN MHN= .
Suy ra NDA CDA CDN= − =ABC MHN MHB MHN− = − =NHB nên tứ giác ADNH
nội tiếp đường tròn.


<b>Bài 10.</b>Cho tam giác nhọn ABC có AB AC và hai đường cao BD, CE cắt nhau tại H.
Gọi I là trung điểm BC. Hai đường tròn ngoại tiếp của hai tam giác BEI và CDI cắt
nhau ở K. Gọi giao điểm của DE với BC là M. Chứng minh rằng tứ giác BKDM nội
tiếp đường tròn.


<b>Lời giải</b>
Theo giả thiết ta được các tứ giác BEKI và
CDKI nội tiếp đường trịn. Từ đó ta được


0


ABC EKI IKD ACB 180+ = + = . Mà ta lại có


0


BAC ABC ACB EKD EKI IKD+ + + + + =540


Do đó suy ra 0


BAC EKD 180+ = nên tứ giác
EAKD nội tiếp đường trịn. Từ đó ta có


ADE AKE= .



M I


K
H
E


C
D


</div>
<span class='text_page_counter'>(13)</span><div class='page_container' data-page=13>

tiếp. Do đó ta được ADE=ABC. Kết hợp ADE AKE= hay ABI=AKE nên suy ra


0


EKI AKE EKI ABI 180+ = + = . Vậy ta được ba điểm A, K, I thẳng hàng. Tam giác
BCD vng tại D có I là trung điểm cạnh huyền BC nên ta được IKC IDC ICD= = . Lại


có IKC KAC ACK= + và ICD ICK KCD= + nên KAC ICK= . Mà lại có KAD DEK=
nên ICK=DEK do đó tứ giác MEKC nội tiếpđường trịn. Từ đó suy ra MEC MKC= .


Theo kết quả trên ta có 0


IKC=AED MEB; MEC MEB 90 ; MKC MKI IKC= = + = + . Suy
ra MKI=900hay MK vng góc với KI nên các điểm A, E, H, I, K nằm trên đường
tròn đường kính AH. Do đó HK vng góc với AInên ba điểm M, H, K thẳng hàng.
Tứ giác DEHK nội tiếp nên ta được HEK HDK= và tứ giác MEKC nội tiếp nên ta
được KEC KMC= . Do đó suy ra KMC HDK= hay KMB=BDK nên tứ giác BKDM


nội tiếp đường tròn.



<b>Bài 11. </b>Cho tam giác ABC nội tiếp đường tròn

( )

O . Đường tròn

( )

K tiếp xúc với AC,
AB lần lượt tại E, F và tiếp xúc trong với đường tròn

( )

O tại S. Các đường thẳng SE,


SF cắt đường tròn

( )

O lần lượt tại M, N khác S. Các đường tròn ngoại tiếp các tam
giác AEM, AFN cắt nhau tại P khác A. Gọi giao điểm của EN, FM với đường tròn

( )

K


lần lượt là G, H. Gọi giao điểm của GH với MN là T. Chứng minh rằng AP đi qua
trung điểm của MN và tam giác AST cân.


<b>Lời giải</b>


I H
G


P
O
N


M


T


S
K
F


E


C
B



A


<b> </b>


</div>
<span class='text_page_counter'>(14)</span><div class='page_container' data-page=14>

<i><b>Trước hết ta phát biểu và chứng minh bài tốn phụ</b><b>. </b>Cho đường trịn </i>

( )

O <i>và một điểm T</i>
<i>nằm ngồi đường trịn. Từ T kẻ cát tuyến TNM với đường trịn </i>

( )

O <i>. Khi đó TA là tiếp tuyến</i>
<i>tại A của đường tròn </i>

( )

O <i>khi và chỉ khi</i>


2
2


TN AN


TM= AM <i>. </i>
<i><b>Chứng minh. </b></i>


<i>+ Điều kiện cần. Giả sử TA là tiếp tuyến tại A với đường trịn </i>

( )

O <i>. Khi đódễ thấy hai tam</i>
<i>giác TAN và TMA đồng dạng với nhau nên ta có </i>TA2=TN.TM<i>. Đồng thời cũng từ hai tam </i>
<i>giác đồng dạng đó ta có </i>TN AN


TA= AM<i> nên </i>


2 2


2 2


TN AN


TA = AM <i>. Do vậy ta được</i>



2
2


TN AN


TM= AM <i>. </i>
<i>+ Điều kiện đủ. Giả sử điểm A thuộc đường tròn </i>

( )

O <i>thỏa mãn</i>


2
2


TN AN


TM =AM <i>. Từ T kẻ tiếp </i>
<i>tuyến TD của đường tròn </i>

( )

O <i>với D là tiếp điểm nằm cùng phía với A so với MN. Khi đó</i>
<i>theo chứng minh như trên ta có</i>


2
2


TN DN


TM = DM <i>. Mà ta lại có </i>


2
2


TN AN



TM =AM <i> nên </i>


DN AN


DM=AM<i>. </i>
<i>Điều này dẫn đến hai điểm A và D trùng nhau.Vậy TA là tiếp tuyến của đường trịn </i>

( )

O <i>.</i>
<i>Vậy bài tốn phụ được chứng minh. </i>


<b>Trở lại bài toán.</b> Dễ thấy APF 180= 0 −ANS 180= 0−APE nên ba điểm E, P, F thẳng


hàng. Ta có APM=AEM và AEM=SEC kết hợp với SEC EFS= nên EFS=PAN.


Điều này dẫn đến tứ giác ANFP nội tiếp đường tròn, suy ra APM=PAN nên AN


song song với PM. Chứng minh hoàn toàn tương tự ta được AM song song với PN.
Do vậy tứ giác AMEN là hình bình hành.Dễ thấy các ta giác SKF và SON đồng dạng
với nhau nên suy ra KF song song với ON. Tương tự ta cũng có KE song song với


OM. Suy ra SF SK SE


SN= SO=SM nên MN song song với EF. Từ đó HGE HFE HMN= =


nên tứ giác MNGH nội tiếp đường tròn. Giả sử TS cắt đường tròn

( )

O và

( )

K lần
lượt tại L và J, khi đó ta được TS.TL TM.TN TH.TG= = =TS.TJ. Từ đó suy ra ba điểm
S, L, J trùng nhau nên TS là tiếp tyến của đường tròn

( )

O . Tứ giác AMPN là hình bình


hành nên AP và MN cắt nhau tại trung điểm I của mỗi đường.


Ta có IAM PES= =FST=NAS và AMI=AMN=ASN nên hai tam giác AIM và ANS



</div>
<span class='text_page_counter'>(15)</span><div class='page_container' data-page=15>

ta được AN.SM AI.SN AM.SN= = . Do TS là tiếp tuyến với đường trịn

( )

O nên ta có


2 2


2 2


TM SM AM


TN = SN = AN nên theo bài toán phụ trên ta suy ra được TA là tiếp tuyến tại A


với đường tròn

( )

O . Do đó tam giác AST cân tại T.


<b>Bài 12.</b>Cho tứ giác ABCD có hai đường chéo vng góc với nhau tại O. Gọi M, N, P,
Q theo thứ tự đối xứng với O qua AB, BC, CD, DA. Đường thẳng AN cắt đường tròn
ngoại tiếp tam giác OMN tại E, đường thẳng AP cắt đường tròn ngoại tiếp tam giác
OQP tại F. Chứng minh rẳng bốn điểm M, E, F, Q cùng thuộc một đường tròn.


<b>Lời giải</b>
Vì O, M đối xứng nhau qua AB và O, N đối xứng
nhau qua BC nên tâm đường tròn ngoại tam giác
OMN là B. Hơn nữa AC vuông góc với BD nên AO
là tiếp tuyến của đường tròn ngoại tiếp OMN. Vì
hai tam giác AOB và AMB bằng nhau nên AM là
tiếp tuyến đường trịn ngoại tiếp OMN.Do đó


AM AN= . Tương tự ta có OC và CN là các tiếp
tuyến với đường tròn ngoại tiếp tam giác OMN.
Lại có AO AQ AM= = và CO CN CP= = nên tam
giác AMQ cân.



F
E


Q <sub>P</sub>


N
M


O


D


C
B


A


Do đó AMQ AQM= nên AME QME AQF MQF− = − . Vì AM và AQ lần lượt là tiếp
tuyến của các đường tròn ngoại tiếp tam giác MNQ và PQO nên suy ra AME MNE=
và AQF QPF= . Do đó MNE QME QPF MQF− = − hay QME MQF MNE QPF− = − . Vì


AM tiếp tuyến của đường trịn ngoại tiếp tam giác MNO nên AM2=AE.AN. Vì AQ


là tiếp tuyến của đường tròn ngoại tiếp tam giác PQO nên AQ2 =AF.AP. Khi đó ta có
AE.AN AF.AP= nên tứ giác EFPN nội tiếp nên FEA FPN= và EFA ENP= . Chú ý


</div>
<span class='text_page_counter'>(16)</span><div class='page_container' data-page=16>

(

) (

)



(

)




(

) (

) (

)



(

)



0 0


0


0 0


FEM EFQ FEA AEM EFA APQ FPN 180 MNE ENP 180 QFP


FPN ENP MNO QOP 180 MOA NOC


FPN ENP 180 MOA NOC 180 QOA POC


FPN ENP MOA NOC QOA POC


FPN ENP MNO ONC QPO OPC


FPN ENP MNO ONP PNC QPO OPN NPC
FPN ENP


− = + − + = + − − − +


= − − + = − −


= − − − − + − −


= − + + − −



= − + + − −


= − + + + − − −


= −


(

) (

)



(

) (

)



MNO ONP QPO OPN


FPN ONP ENP ONP MNO OPQ


FPO ENO MNO QPO MON ENO QPO FPO MNE QPF


+ + − −


= − − − + −


= − + − = − − − = −


Kết hợp với QME MQF MNE QPF− = − ta được FEM EFQ QME MQF− = − . Từ đó suy


ra FEM MQF EFQ QME+ = + hay FEM MQF 180+ = 0 nên tứ giác MEFQ nội tiếp
đường tròn.


<b>Bài 13. </b>Cho tứ giác ABCD có H, K, L lần lượt là chân đường vng góc hạ từ D xuống
AB, BC, CA. Chứng minh rằng tứ giác ABCD nội tiếp khi và chỉ khi CA AB BC



DK =DH+DL .


<b>Lời giải</b>


• <b>Điều kiện cần.</b>Giả sử tứ giác ABCD nội tiếp đường
trịn. Khi đó gọi

( )

O ngoại tiếp tứ giác ABCD. Trên


đường tròn

( )

O lấy điểm N sao cho AN BC= suy ra
AB NC= . Từ đó ta được ADN=BDC. Gọi E là giao
điểm của ND với AC. Khi đó CAD CBD= . Lại có


(

) (

)



1 1


AED sdAD sdCN sdAD sdAB BCD


2 2


= + = + =


N


E


L
K


H



D


C
B


A


Từ đó hai tam giác ADE và BDC đồng dạng với nhau. Mà DK và DL là hai đường cao
tương ứng nên DK AE


DL = BC hay


BC AE


DL = DK. Lại có hai tam giác CDE và BDA đồng dạng


với nhau, mà DK và DH là hai đường cao tương ứng nên CE BA


DK =DH. Từ đó ta được


BC BA AE CE AC


</div>
<span class='text_page_counter'>(17)</span><div class='page_container' data-page=17>

• <b>Điều kiện đủ. </b>Giả sử tứ giác ABCD thỏa mãn


CA AB BC


DK= DH+DL . Gọi


'



D là giao điểm của DB với
đường tròn ngoại tiếp tam giác ABC. Khi đó tứ
giác ABCD’ nội tiếp đường tròn. Gọi '


H , '


K , '


L lần
lượt là chân đường vuông góc hạ từ D’ xuống AB,
BC, CA. Chứng minh hồn tồn tương tự như trên


ta có CA<sub>'</sub> <sub>'</sub> AB<sub>'</sub> <sub>'</sub> BC<sub>' '</sub>
D K =D H +D L


D'


L'
K'


H' L


K


H


M


D



C
B


A


Mặt khác theo định lí Talets ta lại có


' ' ' '


D H D B D L


DH = DB = DL .


Từ đó ta suy ra được


'
' ' D B.DH


D H


DB
= và


'
' ' D B.DL


D L


DB


=


Từ đó ta thu được


'
' '


AB BC AC.D B


DH+DL = DB.D K , mà ta lại có


CA AB BC


DK =DH+DL


Gọi M là giao điểm của BD và AC, khi đó ta được


' ' ' '


D B D K D M


DB = DK = DM .
Theo tính chất dãy tỉ số bằng nhau tađược


' '


DB D B DM D M


DB DM



− −


= nên


' '


DD DD


DB = DM .


Mà ta thấy DB DM nên từ đó ta suy ra được DD' =0 hay hai điểm D và D' trùng


nhau. Do đó tứ giác ABCD nội tiếp đường tròn.


<b>Bài 14.</b>Cho hai đường tròn

(

O ; R<sub>1</sub> <sub>1</sub>

)

(

O ; R<sub>2</sub> <sub>2</sub>

)

với R<sub>1</sub> R<sub>2</sub> sao cho tiếp tuyến chung


ngoài M M<sub>1</sub> <sub>2</sub> vng góc với tiếp tuyến chung trong N N<sub>1</sub> <sub>2</sub> tại điểm A. Gọi P P<sub>1 2</sub> là tiếp
tuyến chung trong thứ hai của hai đường tròn trên trong đó các điểm M ; N ; P<sub>1</sub> <sub>1</sub> <sub>1</sub> thuộc
đường tròn

(

O ; R1 1

)

và M ; N ; P2 2 2 thuộc đường trong

(

O ; R2 2

)

. Tính diện tích tam


giác AP P<sub>1 2</sub> theo R<sub>1</sub> và R<sub>2</sub>.


</div>
<span class='text_page_counter'>(18)</span><div class='page_container' data-page=18>

Dễ thấy các đường thẳng O O ; N N ; P P<sub>1</sub> <sub>2</sub> <sub>1</sub> <sub>2</sub> <sub>1 2</sub>


đồng quy tại một điểm, gọi điểm đó là điểm


K. Theo tính chất các tiếp tuyến chung của
hai đường trong ta có N N<sub>1</sub> <sub>2</sub> =P P<sub>1 2</sub>. Lại có


ngồi M M<sub>1</sub> <sub>2</sub> vng góc với tiếp tuyến



chung trong N N1 2 tại điểm A nên ta được


1 2 1 2 1 2


N N =P P =R −R .


E
K


A
H


P<sub>2</sub>


P<sub>1</sub> N2


N<sub>1</sub>


M<sub>2</sub>
M<sub>1</sub>


O<sub>2</sub>
O<sub>1</sub>


Mặt khác ta có O N<sub>1</sub> <sub>1</sub> song song vớiO N<sub>2</sub> <sub>2</sub> nên áp dụng định lí Thales ta có 2 2


1 1


N K R



N K = R


hay 2 2


1 2 1 2


N K R


N N =R +R . Mà N N1 2 =P P1 2 =R1−R2 nên ta được


(

)



2 1 2


2 2


1 2


R R R


P K N K


R R




= =


+ .



Suy ra 2

(

1 2

)

1 2


2 2 2


1 2 1 2


R R R 2R R


AK AN N K R


R R R R




= + = + =


+ + .


Gọi E là giao điểm của hai tiếp tuyến P P<sub>1 2</sub> và M M<sub>1</sub> <sub>2</sub>.


Khi đó ta có M M<sub>1</sub> <sub>2</sub> =EM<sub>1</sub>+EM<sub>2</sub> =EM<sub>1</sub>+EP<sub>2</sub> =EM<sub>1</sub>+EP<sub>1</sub>+P P<sub>1 2</sub> =2EM<sub>1</sub>+P P<sub>1 2</sub>


Mặt khác ta lại có M M<sub>1</sub> <sub>1</sub> =R<sub>1</sub>+R<sub>1</sub> =AN<sub>1</sub>+AN<sub>1</sub> =2AN<sub>2</sub> +N N<sub>1</sub> <sub>2</sub>


Do đó kết hợp với N N<sub>1</sub> <sub>2</sub> =P P<sub>1 2</sub> =R<sub>1</sub>−R<sub>2</sub> ta được EM<sub>1</sub> =AN<sub>2</sub> =R<sub>2</sub>


Từ đó suy ra AE=AM<sub>1</sub>−EM<sub>1</sub> =R<sub>1</sub>−R<sub>2</sub>


Do vậy ta được EM<sub>2</sub> =AE AM+ <sub>2</sub> =R<sub>1</sub>−R<sub>2</sub>+R<sub>2</sub> =R<sub>1</sub>



Suy ra

(

)



2 2


2 1 2 1 2


2 2 2 2 1


1 2 1 2


R R R R R


EK EP KP EM KP R


R R R R


− +


= − = − = − =


+ +


Tam giác AEK vuông tại A có đường cao AH nên ta được AH.EK AE.AK=


Từ đó ta tính được 1 2<sub>2</sub>

(

1 <sub>2</sub> 2

)



1 2


2R R R R



AE.AK
AH


EK R R




= =


+


Vậy ta có

(

)



1 2


2
1 2 1 2


AP P 1 2 2 2


1 2


R R R R


1


S AH.P P


2 R R





= =


+ .


</div>
<span class='text_page_counter'>(19)</span><div class='page_container' data-page=19>

<b>Lời giải</b>
Gọi G là giao điểm của đường tròn ngoại
tiếp tam giác CDF và EF (F khác G). Ta


cần chứng minh được hai tứ giác AOCG
và BOCG nội tiếp đường trịn, từ đó ta
suy ra được OG vng góc với EF. Khi đó
ta chỉ cần chứng minh được ba điểm O, H
và G thẳng hàng là bài toán được chứng


minh.


G


O
H


E C


B


D
A



F


Thật vậy, ta có các tứ giác CDFG và ACBD nội tiếp đường trịn nên CGE=CDA và
CDA=CBE. Từ đó ta được CGE CBE= , do đó tứ giác CBGE nội tiếp đường tròn.Suy
ra GBE GCE= , mà ta lại có GCE GFA= nên suy ra GBE=GFA. Từ đó suy ra tứ giác
ABGF nội tiếp đường trịn. Do vậy AGF ABF= kết hợp với ABF CDA= thì ta được


AGF CDA= nên ta lại có tứ giác ADEG nội tiếp đường tròn, suy ra AGF CDA= . Mà
ta có CGE CDA= , do đó ta được AGF CGE+ =2CDA. Mà ta lại có 2CDA COA= nên


AGF CGE+ =COA. Suy ra 0


AGF CGE CGA COA CGA 180+ + = + = nên tứ giác
AOCG nội tiếp đường trịn. Ta có DGF=DCB và BGE=DCB nên


DGF=BGE 2DCB+ . Lại có 2DCF=BOD nên DGF BGE+ =DOB.


Suy ra DGF BGE DGB+ + =BOD BGD 180+ = 0 nên tứ giác BODG nội tiếp đường trịn.
Do đó OGA=OGC mà AGF CGE= , suy ra OGF OGE= . Mà hai góc này kề bù nhau
nên suy ta mỗi góc có số đo 900. Do đó ta được OG vng góc với EF. Hai đường trịn
AOCG và BODG có hai giao điểm là O và G. Kết hợp với HA.HC HB.HD= suy ra


điểm H phải nằm trên đường thẳng OG hay ba điểm H, O, G thẳng hàng. Vậy ta được
OH vng góc với EF.


<b>Bài 16.</b> Cho tứ giác ABCD nội tiếp đường trịn tâm O có AC vng góc với BD. Các
tiếp tuyến tại A, B với đường tròn

( )

O cắt nhau ở X, các tiếp tuyến tại B,C với đường


</div>
<span class='text_page_counter'>(20)</span><div class='page_container' data-page=20>

<b>Lời giải</b>



L
O'
O
Q


P


N
M


I


D


C
B


A


I<sub>8</sub>


I<sub>7</sub> I6


I<sub>5</sub>
I<sub>4</sub>


I<sub>3</sub>


I<sub>2</sub>


I<sub>1</sub>


O
T


Y
Z


X


D


E


C


B
A


<i><b>Trước hết ta phát biểu và chứng minh bổ đề</b></i>. <i>Cho tứ giác ABCD ngoại tiếp đường tròn </i>

( )

I
<i>với M, N, P, Q theo thứ tự là tiếp điểm của </i>

( )

I <i>với AB, BC, CD, DA. Gọi O là giao điểm của</i>
<i>AC và BD. Khi đó O cũng là giao điểm của hai đoạn thẳng MP, NQ và tứ giác ABCD nội tiếp </i>
<i>khi và chỉ khi MP vng góc với QM.</i>


<i><b>Chứng minh.</b></i>Bài tốn có hai trường hợp


<b>+ Trường hợp 1.</b>Tứ giác ABCD có một cặp cạnhđối diện song song với nhau, khi đó
bổ để hiển nhiên đúng.


<b>+ Trường hợp 2.</b>Tứ giác ABCD khơng có cặp cạnh đối diện nào song song với nhau.



Khi đó gọi <sub>O </sub>' <sub>là giao điểm của AC và MP. Qua A kẻ AL song song với CD với L </sub>


thuộc MP.


Ta có ALM DPM 1sdMQP AML


2


= = = nên ta được AL AM= . Theo định lí Talet ta có


'
'


O A LA


CP


O C = hay


'
'


O A MA


CP


O C = .


Hoàn toàn tương tự ta gọi <sub>O </sub>" <sub>là giao điểm của AC và NQ, khi đó ta được </sub>


"
"


O A QA


CN


O C = .


Để ý là MA AQ= và CP CN= nên ta được


' "
' "


O A O A


O C = O C , suy ra


'


O và O trùng nhau. "


<b> </b>


Chứng minh rằng tâm đường tròn nội tiếp các tam giác ETX, EXY, EYZ, EZT, TXY,


</div>
<span class='text_page_counter'>(21)</span><div class='page_container' data-page=21>

(

)



(

) (

) (

)




0


0 0 0 0


POQ 360 PDQ OQD OPD


360 180 QIP 90 NQI 90 MPI QIP NQI MPI


= − + +


= − − − − − − = + +


Hay ta được QIP POQ= −

(

NQI MPI+

)



Mà ta có NQI MPI INQ PMI+ = + , do đó ta được QIP POQ= −

(

INQ PMI+

)



Mặt khác ta có MIB IMO INO MON= + + .


Từ đó suy ra QIP MIN POQ+ = −

(

INQ PMI+

)

+IMO INO MON+ + =2MON


Do đó 0


MON=90 hay <sub>QIP MIN 180</sub>+ = 0<sub> nên </sub><sub>PDQ MBN 180</sub>+ = 0<sub>. Do vậy tứ giác </sub>


ABCD nội tiếp đường tròn. Vậy bổ đề được chứng minh xong.


<b>Trở lại bài toán.</b>Gọi I ; I ; I ; I ; I ; I ; I ; I<sub>1</sub> <sub>2</sub> <sub>3</sub> <sub>4</sub> <sub>5</sub> <sub>6</sub> <sub>7</sub> <sub>8</sub> lần lượt là tâm đường tròn nội tiếp các tam
giác ETX, EXY, EYZ, EZT, TXY, XYZ, YZT, ZTX. Theo bổ đề trên ta có E thuộc hai
đường chéo XZ, YT và tứ giác XYZT nội tiếp đường trịn. Từ đó ta được ZTY ZXY= .
Mà EDT EBX= nên ta được TED XED= . Kết hợp với AC vng góc với BD ta được



AET=AEX, do đó điểm I<sub>1</sub> thuộc EA. Hồn tồn tương tự ta cũng chứng minh được


điểm I2 thuộc EB. Mà ta lại có XA=XB nên ta có


1 2


1 2


I A XA XB I B


I E = XE =XE =I E.Từ đó theo


định lí Talets ta suy ra I I<sub>1 2</sub>song song với AB. Chứng minh hoàn toàn tương tự ta cũng
được I I<sub>2 3</sub>song song với BC, I I<sub>3 4</sub>song song với CD, I I<sub>4 1</sub>song song với DA.Mà tứ giác
ABCD nội tiếp đường tròn nên ta được DAB DCB 180+ = 0. Do đó ta được


0
4 1 2 4 3 2


I I I +I I I =180 , suy ra tứ giác I I I I1 2 3 4 nội tiếp đường trịn.Lại có


0 0 0 0


1 5 2 1 3 2 5


XTY XYT 1 1


I I I I I I TI Y ACB 180 XAB 90 TXY 90 AXB 180



2 2 2


+


+ = + = − + = + + − =


Do đó tứ giác I I I I<sub>1 5 2 3</sub> nội tiếp đường trịn. Chứng minh hồn tồn tương tự như trên
ta cũng được các tứ giác I I I I2 6 3 4, I I I I1 3 7 4, I I I I4 8 1 2 nội tiếp đường tròn. Vậy các điểm


1 2 3 4 5 6 7 8


I ; I ; I ; I ; I ; I ; I ; I cùng nằm trên một đường tròn.


</div>
<span class='text_page_counter'>(22)</span><div class='page_container' data-page=22>

<b>Bài 17.</b>Cho đường tròn

( )

I nội tiếp tam giác ABC, tiếp xúc với BC và AC lần lượt tại


D và E. Đường thẳng AD cắt đường tròn

( )

I tại điểm thứ hai là P. Giả sử 0


BPC 90= .


Chứng minh rằng AE AP+ =PD.


<b>Lời giải</b>
Trước hết ta phát biểu và chứng minh hai bổ đề sau.


<b>Bổ đề 1.</b>Cho tam giác ABC và các điểm D, E lần
lượt trên đường thẳng BC sao cho DB EB


DC =EC


(với BD BC BE  và DAE 90= 0). Khi đó AD


và AE theo thứ tự là đường phân giác trong và
phân giác ngoài của tam giác ABC.


L
K


E
C


D
B


A


<b>Chứng minh.</b> Qua D kẻ đường thẳng KL song song với AE

(

K AB; L AC 

)

. Do AD


vng góc với AE nên AD vng góc với KL. Theo định lí Talets và chú ý đến


DB EB


DC =EC, BD BC BE  ta có


KD KD AE BD CE BD CE


. . . 1 KD LD


LD = AE LD= BE CD=CD BE =  =


Từ đó suy ra tam giác KAL cân tại A nên AD là đường phân giác của góc BAC, kết



hợp với 0


DAE 90= ta suy ra được AD và AE theo thứ tự là đường phân giác trong và
phân giác ngoài của tam giác ABC.Vậy bổ đề 1 được chứng minh.


<b>Bổ đề 2.</b>Cho tứ giác ABCD nội tiếp đường tròn


( )

O . Các tiếp tuyến với đường tròn

( )

O tại A, C
cắt nhau tại S. khi đó ba điểm S, B, D thẳng hàng
khi và chỉ khi AB.CD AD.BC=


<b>Chứng minh.</b> Giả sử ba điểm S, B, D thẳng hàng.
Khi đó ta có SAB SDA= và SCB SDC= nên suy ra


SAB SDA


 ∽  và SCB ∽ SDC.


O


D'


S


D


C
B


A



Kết hợp với SA SC= ta được AB SA SC CB


</div>
<span class='text_page_counter'>(23)</span><div class='page_container' data-page=23>

Ngược lại, giả sử AB.CD AD.BC= . Khi đó gọi '


D là giao điểm tứ hai của SB với
đường tròn

( )

O . Chứng minh tương tự như trên ta được AB.CD' =AD .BC'


Mà ta lại có AB.CD AD.BC= nên ta thu được


' '


CD AD


CD = AD .


Kết hợp với <sub>CD A CDA</sub>' = <sub>ta được </sub><sub>CD A</sub>' <sub>∽</sub> <sub>CDA</sub><sub>. </sub><sub>Do đó suy ra </sub><sub>CAD</sub>' =<sub>CAD</sub><sub>, suy </sub>


ra hai tia AD và AD’ trùng nhau nên hai điểm D và D’ trùng nhau.Vậy ba điểm S, B,
D thẳng hàng. Bổ đề 2 được chứng minh.


I
F


E


Q


K
L



P


D C


B


A


<b>Trở lại bài toán.</b>Gọi F là tiếp điểm của của đường tròn

( )

I với AB, K là giao điểm của
BC và EF. Lấy đểm L thuộc đoạn EF sao cho CL song song với AB. Gọi Q là giao điểm
thứ hai của PC với đường trịn

( )

I .


Theo định lí Thales và chú ý là AE AF; BD BF; CD CE= = = ta có


KB BF BF AF BF AF BF BD CK BK


. .


KD =CL= AF CL=AF CE =CE=CDCD=BD


Ta có PC vng góc với PB và theo bổ đề 1 trên ta được QPK CPK CPD QPD= = = .


Áp dụng bổ đề 2 cho tứ giác EPFD ta được EP.DF FP.DE= . Áp dụng bổ đề 2 cho tứ
giác EPFD ta được KP tiếp xúc với đường trịn

( )

I . Từ đó ta suy ra được QPK QDP= .


Do đó QPD QDP= . Từ đó dẫn đến tam giác DPQ cân tại Q nên suy ra QD QP= . Lại
áp dụng bổ đề 2 và định lí Ptoleme cho tứ giác PEQD nội tiếp ta được


2PE.PQ 2PE.DQ PE.DQ PD.EQ DE.QP= = + =



Tờ đó suy ra 2PE=DE. Chú ý đến hai tam giác APE và AED đồng dạng với nhau ta
được 2AE AD; 2AP AE= = .


Do đó ta có AE AP+ =2AE AE AP− + =AD 2AP AP− + =AD AP− =DP


</div>
<span class='text_page_counter'>(24)</span><div class='page_container' data-page=24>

<b>Bài 18.</b> Cho hình chữ nhật ABCD và đường tròn

( )

O tiếp xúc với các cạnh AB, BC,
DA. Từ C kẻ tiếp tuyến với đường tròn

( )

O cắt AD tại E. Vẽ đường trong

( )

I nội tiếp
tam giác DEC và đường tròn

( )

J tiếp xúc với CB, CE và tiếp xúc ngồi với đường trịn
(O). Biết rằng hai đường trịn

( )

I và

( )

J có cùng bán kính. Tính tỉ số của AB và AD.


<b>Lời giải</b>


<b> Trước hết ta phát biểu và chứng minh bổ </b>
<b>đề.</b> <i>Cho hai đường tròn </i>

(

O; R

)

<i> và </i>

( )

I; r <i>tiếp xúc </i>
<i>ngoài với nhau. Gọi BC là tiếp tuyến chung ngồi của </i>
<i>hai đường trịn, khi đó ta có</i> BC=2 R.r


<b>Chứng minh.</b> Gọi A là giao điểm của hai đường


tròn

( )

O và

( )

I . Vẽ tiếp tuyến chung trong tại A
của hai đường tròn, tiếp tuyến này cắt BC tại M.


C
B


M


A I



O


Theo tính chất của hai tiếp tuyến cắt nhau ta có MA MB= và MB MC= nên suy ra


được MA MB MC BC


2


= = = . Do đó tam giác ABC vng tại A nên ta được BAC 90= 0.


Lại theo tính chất của hai tiếp tuyến cắt nhau ta có MO là phân giác BMA và MI là
phân giác AMC mà BMA và AMC là hai góc kề bù nên 0


OMI=90 . Theo hệ thức
lượng trong tam giác vuông ta có MA2 =OA.IA R.r= nên suy ra MA= R.r. Do đó ta
được BC=2MA=2 R.r


<b>Trở lại bài tốn. </b>Ta kí hiệu d<sub>(</sub><sub>X,YZ</sub><sub>)</sub> là khoảng
cách từ điểm X đến đường thẳng YZ. Gọi T là
tiếp điểm của hai đường tròn

( )

O và

( )

J . Gọi F
là giao điểm của BC và tiếp tuyến với đường


tròn

( )

J kể từ E (khác EC). Gọi G là giao điểm
của TF và AD. Gọi M, N lần lượt là giao điểm
của đường tròn

( )

J với EF, BC. Gọi P là giao
điểm thứ hai của đường tròn

( )

O với TM.


T
P



S
Q


N
M
E


J
L
O


K
I


G


D C


</div>
<span class='text_page_counter'>(25)</span><div class='page_container' data-page=25>

tiếp điểm của CE với đường tròn

( )

O ,

( )

J . Đặt R, r lần lượt là bán kính của đường


trịn

( )

O ,

( )

J và CL CN x= = . Ta thấy EF, CD, OQ, JN song song với nhau và GP


song song với EF nên ta được GP song song với IN hoặc GP và JN trùng nhau.Lại có
T, N, Q thẳng hàng nên ta có ( )


( )


( )
( )



T ,PG T ,JN


T ,EF T ,EF


d <sub>TP</sub> <sub>OP</sub> <sub>OQ</sub> <sub>TQ</sub> d


d =TM= JM = JN =TN =d


Kết hợp với OP OQ R 1


JM = JM = r  nên ta được d(T,PG)=d(T,JN) d(T,EF)


Từ đó chú ý là T, PG, JN cùng thuộc nửa mặt phẳng bờ EF nên ta suy ra được PG và
JN trùng nhau. Điều đó có nghĩa là GN song song với QS. Lại thấy tứ giác OPGQ là
hình vng kết hợp theo bổ đề trên ta thu được R=OP=OQ SN= =2 R.r nên suy ra


R=4r. Ta có tứ giác JNFM là hình vng nên suy ra


EC EK KL LC EQ SN LC SF SN LC 2SN FN LC 2R r x= + + = + + = + + = − + = − +


Hay ta được EC 2R 1R x 7R x


4 2


= − + = + . Mà ta có 0


EFC 90= và EF 2R; FC 1R x
4


= = + .



Nên theo định lí Pitago ta được

( )



2 2


2


7 1 1


R x 2R R x x R


4 4 3


 <sub>+</sub>  <sub>=</sub> <sub>+</sub> <sub>+</sub>  <sub> =</sub>


   


    .


Do đó ta được 6AD 6BC 6 BN NC

(

)

6 2R 1R 7.2R 7AB
3


 


= = + = <sub></sub> + <sub></sub>= =


  hay


AB 6



AD=7.


<b>Bài 19.</b>Cho tứ giác ABCD nội tiếp đường trịn có I là giao điểm của hai đường chéo.
Gọi I ; I ; I ; I<sub>1</sub> <sub>2</sub> <sub>3</sub> <sub>4</sub> theo thứ tự là tâm đường tròn nội tiếp các tam giác IAB, IBC, ICD, IAD.
Chứng minh rằng tứ giác I I I I1 2 3 4 nội tiếp khi và chỉ khi tứ giác ABCD ngoại tiếp.


<b>Lời giải</b>


<b>Trước hết ta phát biểu và chứng minh bổ đề.</b><i>Nếu I là tâm đường tròn nội tiếp tam giác </i>


<i>ABC thì ta có </i>AI2 AB.AC AB AC BC

(

)



AB BC CA
+ −
=


+ + <i>. </i>


</div>
<span class='text_page_counter'>(26)</span><div class='page_container' data-page=26>

<b>Chứng minh. </b>Gọi D là giao điểm của AI với BC, E là
giao điểm thứ hai của AI với đường tròn ngoại tiếp tam
giác ABC. Khi đó ta thấy AEB∽ ACD và


DEB DCA


 ∽  nên ta được AE AC


AB=AD và


DE DC



DB= DA


Từ đó ta được AE.AD AB.AC= và DE.DA DB.DC=


Do đó

(

AE DE AD AB.AC DB.DC−

)

= −


I


E


D C


B


A


Hay ta được <sub>AD</sub>2 =<sub>AB.AC DB.DC</sub>− <sub>. </sub><sub>Ta có AD là đường phân giác của tam giác ABC </sub>


nên ta được BD AB


CD= AC suy ra


BD AB


BD CD+ =AB AC+ hay


AB.BC
DB


AB AC


=


+ . Hoàn toàn


tương tự ta được CD AC.BC
AB AC
=


+ nên ta được


(

) (

)

(

)(

)



2
2


2 2


AB.AC.BC AB.AC


AD AB.AC AB BC CA AB AC BC


AB AC AB AC


= − = + + + −


+ +


Mà ta lại có AI là phân giác của tam giác ABD nên AI AB AB AB AC


AB.BC



DI BD BC


AB AC


+


= = =


+


Suy ra AI AB AC


AD AB BC CA


+
=


+ + . Từ đó ta được


(

)



2 AB.AC AB AC BC


AI


AB BC CA
+ −
=



+ + .


<b>Trở lại bài toán</b>. Do tứ giác ABCD nội tiếp đường tròn nên ta được IAB∽ IDC và


IBC IAD


 ∽  . Do đó IA IB AB IC ID CD


IA IB AB IC ID CD


+ − + −


=


+ + + + và


IB IC BC IA ID AD


IB IC BC IA ID AD


+ − + −


=


+ + + + .


Theo bổ đề trên ta có


(

)




2
1


IA.IB IA IB AB
II


IA IB AB
+ −
=
+ + ;

(

)


2
3


IC.ID. IC ID CD
II


IC ID CD
+ −
=
+ +

(

)


2
2


IB.IC IB IC BC
II


IB IC BC
+ −


=
+ + ;

(

)


2
4


ID.IA. ID IA DA
II


ID IA DA
+ −
=


+ +


<b>+ Điều kiện cần.</b>Giả sử tứ giác I I I I1 2 3 4 nội tiếp đường


trịn. Khi đó ta có II .II<sub>1</sub> <sub>3</sub> =II .II<sub>2</sub> <sub>4</sub>


I<sub>4</sub>
I3
I<sub>2</sub>
I<sub>1</sub>
I
D C
B
A


Từ đó ta được IA IB AB IC ID CD. IB IC BC IA ID AD.



IA IB AB IC ID CD IB IC BC IA ID AD


+ − + − <sub>=</sub> + − + −


+ + + + + + + +


Kết hợp với kết quả trên ta được


IA IB AB IC ID CD IB IC BC IA ID AD


IA IB AB IC ID CD IB IC BC IA ID AD


+ − <sub>=</sub> + − <sub>=</sub> + − <sub>=</sub> + −


</div>
<span class='text_page_counter'>(27)</span><div class='page_container' data-page=27>

Do đó ta được AB CD BC DA


IA IB+ =IC ID+ = IB IC+ =IA ID+


Nên suy ra AB CD BC DA


IA IB IC ID IB IC ID IA


+ +


=


+ + + + + + hay ta được AB CD BC DA+ = +


Điều này có nghĩa là tứ giác ABCD ngoại tiếp đường tròn.



<b>+ Điều kiện đủ.</b>Giả sử tứ giác ABCD ngoại tiếp đường tròn ta cần chứng minh tứ


giác I I I I<sub>1 2 3 4</sub> nội tiếp đường tròn.


Thật vậy giả sử tứ giác I I I I<sub>1 2 3 4</sub> không nội tiếp đường trịn. Khi đó ta có II .II<sub>1</sub> <sub>3</sub> II .II<sub>2</sub> <sub>4</sub>


Từ đó ta được IA IB AB IC ID CD. IB IC BC IA ID AD.


IA IB AB IC ID CD IB IC BC IA ID AD


+ − + − + − + −




+ + + + + + + +


Kết hợp với kết quảtrên ta được


IA IB AB IC ID CD IB IC BC IA ID AD


IA IB AB IC ID CD IB IC BC IA ID AD


+ − <sub>=</sub> + − <sub></sub> + − <sub>=</sub> + −


+ + + + + + + + .


Do đó ta được AB CD BC DA


IA IB+ =IC ID+  IB IC+ = IA ID+



Nên suy ra AB CD BC DA


IA IB IC ID IB IC ID IA


+ <sub></sub> +


+ + + + + + hay ta được AB CD BC DA+  +


Điều này có nghĩa là tứ giác ABCD khơng thể ngoại tiếp đường trịn. Điều này mâu
thuẫn với giả thiết tứ giác ABCD ngoại tiếp đường tròn. Vậy điều giả sử là sai hay tứ


giác I I I I<sub>1 2 3 4</sub> nội tiếp đường trịn.
Vậy bài tốn được chứng minh.


<b>Bài 20.</b> Cho tứ giác ABCD có 0


ADC=BCD 90= . Lấy một điểm E trên cạnh CD. Các
đường cao AM, BN của tam giác ABE cắt nhau tại H. Gọi giao điểm của DM và CN là
K. giao điểm của KH và CD là L. Chứng minh rằng KH=KL.


<b>Lời giải</b>


<b>Trước hết ta phát biểu và chứng minh bổ đề</b>. <i>Cho tam </i>
<i>giác ABC có các đường cao AD, BE, CF cắt nhau tại H. Gọi </i>
<i>I là trung điểm của AH. Khi đó bốn điểm D, E, F, I cùng </i>
<i>thuộc một đường tròn.</i>


<b>Chứng minh.</b> Dễ thấy HIE=2HAE; HIF=2HAF nên


ta được EIF=2BAC. Do tứ giác BDHF nội tiếp đường



H
I
F


E


D C


B


</div>
<span class='text_page_counter'>(28)</span><div class='page_container' data-page=28>

trịn nên ta có HBF HDF= .


Do tứ giác CDHE nội tiếp đường trịn nên ta có HDE=HCE. Do tứ giác BCEF nội tiếp
đường tròn nên ta có EBF FCE= . Từ đó suy ra EDF 2FBH= .


Do vậy ta được

(

)

0 0


EDF EIF 2 FBH BAC+ = + =2.90 =180 . Suy ra tứ giác DEIF nội tiếp
đường tròn. Bổ đề được chứng minh.


<b>Trở lại bài toán.</b>Gọi P là giao điểm của EH với


AB và Q là trung điểm của EH. Khi đó theo bổ đề
trên ta được bốn điểm M, N, P, Q cùng thuộc một
đường tròn. Do đó ta có


0


ADE=AME=APE 90= =BCE=BNE=BPE


Suy ra các đa giác ADEMP và BCENP nội tiếp
đường trịn. Do đó ta được


H


L


K


N
Q


P
M


E


D


C B


A


(

) (

)



0 0


0


0 0



0


0


MKN 180 KMN KNM 180 EMN EMK ENM ENK


180 EMN ENM EMK ENK MEN EMD ENC


AEB EAD EBC AEB 90 DEA 90 CEB


AEB 180 DEA CEB AEB BEA NEB MEA


APN BPN 180 MPN


= − − = − − − −


= − − + + = + +


= + + = + − + −


= + − − = + = +


= + = −


Từ đó ta được K thuộc đường trịn ngoại tiếp tam giác MNP nên K thuộc đường tròn
ngoại tiếp tam giác MPQ. Do đó suy ra EDM EPM QPM QKM= = = nên ta được DE


song song với KQ.Kết hợp với QH QE= ta được KH=KL.



<b>Bài 21.</b>Cho tứ giác ABCD và các điểm P, Q nằm trong tứ giác sao cho các tứ giác
ABPQ và CDPQ nội tiếp đường tròn. Giả sử tồn tại điểm E thuộc đoạn PQ thỏa mãn


EAQ EBP= và EDQ ECP= . Chứng minh rằng tứ giác ABCD nội tiếp đường tròn.
<b>Lời giải</b>


</div>
<span class='text_page_counter'>(29)</span><div class='page_container' data-page=29>

E Q
P


S'
S


D
C


B


A


A


B


C
D


S


S' Q E P



<b>+ Trường hợp 1. </b>Điểm S nằm trên tia đối của tia BA. Do góc BPS là góc ngồi của tam
giác BPE. Lại có tứ giác ABPQ nội tiếp đường tròn và EAQ EBP= nên ta được


BES BPS EBP SAQ EAQ SAE= − = − =


Từ đó suy ra SE là tiếp tuyến của đường tròn ngoại tiếp tam giác ABE.


<b>+ Trường hợp 2</b>. Điểm S nằm trên tia đối của tia AB.Khi đó Do góc BES là góc ngồi


của tam giác BPE. Lại có tứ giác ABPQ nội tiếp đường tròn và EAQ EBP= nên ta có
BES BPE EBP SAQ EAQ SAE= + = + = . Từ đó BEP 180= 0−BES 180= 0−SAE=BAE.
Suy ra SE là tiếp tuyến của đường tròn ngoại tiếp tam giác ABE.


Như vậy cả hai trường hợp ta đều được SE là tiếp tuyến của đường tròn ngoại
tiếp tam giác ABE. Từ đó dễ dàng chứng minh được SA.SB SE= 2. Do tứ giác ABPQ
nội tiếp đường trịn nên ta cũng có SA.SB SP.SQ= . Từ đó suy ra <sub>SE</sub>2 =<sub>SP.SQ</sub><sub>. Chú ý </sub>


là xét từng trường hợp như trên ta thu được


+ Nếu S nằm trên tia đối của tia BA thì ta được


(

)(

)

(

)



2 2


SE =SP.SQ= SE EP SE EQ− + =SE + EQ EP .SE EP.EQ− −
Suy ra SE EQ EP


EP.EQ



= .


+ Nếu S nằm trên tia đối của tia AB, lập luận tương tự ta thu được SE EP EQ
EP.EQ



=


Như vậy kết hợp lại ta được SE EP EQ
EP.EQ




= .


Hoàn toàn tương tự gọi S’ là giao điểm của PQ và CD. Xét các trường hợp như
trên ta cũng chứng minh được S' E EP EQ


EP.EQ


</div>
<span class='text_page_counter'>(30)</span><div class='page_container' data-page=30>

SA.SBSC.SD , do đó suy ra tứ giác ABCD
nội tiếp đường tròn.


<b>Bài 22.</b> Cho tam giác ABC có góc BAC 90 0. Giả sử P là một điểm thuộc miền trong
của tam giác ABC sao cho BAP=ACP và CAP=ABP. Gọi M và N lầ lượt là tâm
đường tròn nội tiếp các tam giác ABP và ACP,R là độ dài đường tròn ngoại tiếp tam


giác AMN. Chứng minh rằng 1 1 1 1



R = AB+AC+AP


<b>Lời giải</b>
Gọi E là giao điểm của PM và AB, F là giao
điểm của PN và AC. Từ giả thiết ta có hai


tam giác ABP và CAP đồng dạng với nhau.


Do đó ta được APB APC 180= = 0 −BAC.
Suy ra <sub>EPF</sub> 1

(

<sub>APB APC</sub>

)

<sub>180</sub>0 <sub>BAC</sub>


2


= + = −


nên tứ giác AEPF nội tiếp đường tròn. Mặt


khác APB=APC ta được APE APF= , do


đó ta được AE=AF.


F


E


P
N
M



C
B


A


Theo tính chất đường phân giác ta có PM AP AP PN


EM=AE = AF = PF .


Do M, N là tâm đường tròn nội tiếp các tam giác ABP và ACP nên AM và AN là lần
lượt là phân giác của các góc EAP và FAP . Do đó ta được MNsong song với EF nên


ta được MN MP MP AP


EF = PE =ME MP+ =AE AP+ . Mặt khác lại có


BAC
EF 2AE.sin


2


= nên ta


được MN EF.PM EF.AP 2AE.AP.sinBAC


PE AP AE AP AE 2


= = =


+ + . Áp dụng định lí sin cho tam giác



AMN ta có 1 2 sin MAN 2 .sinBAC AP AE 1 1


R MN MN 2 AE.AP AE AP


+


= = = = + .


Mặt khác do hai tam giác ABP và CAP đồng dạng với nhau nên ta có AB.AP AC.BP=
và AC.AP AB.CP= . Do đó AB.AP AB.CP AC.BP AC.AP+ = + hay AB.AP AC.AP


BP AP+ = AP CP+


</div>
<span class='text_page_counter'>(31)</span><div class='page_container' data-page=31>

Đồng thời 1 BP


AC= AB.AP và


1 CP


AB= AC.CP


giác ta có AE BE AE BE AB


AP BP AP BP AP BP


+


= = =



+ + nên


AP.AB
AE


AP BP
=


+ . Tương tự ta cũng có
AC.AP


AF


AP CP
=


+ . Do đó ta được


AB.AP AC.AP


AE AF


BP AP AP CP


= = =


+ + . Từ đó suy ra


1 1 BP AP AP CP 1 1 1 BP CP 1 1



AE 2 AB.AP AC.AP 2 AB AB AB.AP AC.CP AB AC


 + +   


= <sub></sub> + <sub></sub>= <sub></sub> + + + <sub></sub>= +


   


Do đó ta được 1 1 1 1


R = AB+BC+CA


<b>Bài 23. </b>Cho tứ giác ABCD nội tiếp đường tròn tâm O. Gọi I và J lần lượt là trung điểm
của BD và AC. Chứng minh rằng BD là tia phân giác của góc AIC khi và chỉ khi AC
là tia phân giác của góc BJD .


<b>Lời giải</b>


<b>Trước hết ta phát biểu và chứng minh công thức.</b> Cho
tam giác ABC nội tiếp đường trịn

(

O; R

)

. Khi đó ta


ln có S<sub>ABC</sub> AB.BC.CA
4R


= .


<b>Chứng minh.</b>Vẽ đường kính ADcủa đường trịn

( )

O .
Khi đó ta có ACB ADB=


D


O


C
B


A


Mà ta có tam giác ABD vuông tại B nên sin ADB AB AB


AD 2R


= = . Do đó sin C AB


2R
= .


Ta có S<sub>ABC</sub> 1BC.CA.sin A 1BC.CA.AB AB.BC.CA


2 2 2R 4R


= = = .


<b>Trở lại bài toán.</b>Kẻ CMsong song với BD với M nằm trên đường trịn O. Khi đó ta có


BM CD= và DBM=BDC. Do I là trung điểm của BD nên IB ID= , từ đó ta được hai


tam giác BIM và DIC bằng nhau nên suy ra BIM=DIC


</div>
<span class='text_page_counter'>(32)</span><div class='page_container' data-page=32>

<b>+ Điều kiên cần.</b>Nếu BD là phân giác của góc AIC



thì ta được AID=DIC. Mà lại có BIM=AID nên


ba điểm A, I, M thẳng hàng. Gọi R là bán kính của
đường trịn

( )

O , khi đó ta có S<sub>ABM</sub> AB.BM.MB


4R


= và


ADM


AD.DM.MB
S


4R


= . Vì I là trung điểm của AB nên


ta được S<sub>AMB</sub> =S<sub>AMD</sub>. Suy ra AB.BM AD.DM=


N


M
O


J
I


C
D



B


A


Lại có CD BM, BC DM= = nên ta có AB.CD AD.BC= . Kẻ DNsong song với AC với
N nằm trên đường tròn

( )

O khi đó ta được AD CN= và AN CD= . Mặt khác ta lại có


ABN


AB.BN.NA
S


4R


= và S<sub>CBN</sub> BC.CN.NB


4R


= . Do đó S<sub>ABN</sub> =S<sub>CBN</sub>. Điều này chứng tỏ DN


đi qua trung điểm J của BD. Khi đó ta có AD NC= và DAJ=NCJ nên suy ra
AJD CJN AJB= = . Do đó AD là phân giác của góc BJC .


<b>+ Điều kiện đủ.</b>Nếu AD là phân giác của góc BJC . Khi đó lặplại các chứng minh
như trên ta được BD là phân giác của góc AIC.


Vậy bài tốn được chứng minh.


<b>Bài 24. </b>Cho tứ giác ABCD có diện tích S nội tiếp đường trịn bán kính R. Biết độ dài


các cạnh của tứ giác là AB a, BC b, CD c, DA d= = = = và AC e= . Giả sử tồn tại một
đường tròn tiếp xúc với các tia đối của BA, DA, CD, CB và 2p a b c d= + + + . Chứng
minh rằng R <sub>2</sub>S.e <sub>2</sub>


p e


=


− và


2 2 2 2 8SR 2


a b c d 2p


e


+ + + + =


<b>Lời giải</b>


Trước hết ta nhắc lại định lí Ptoleme và cơng thức diện tích tam giác như sau.


<b>+ Định lí Ptoleme.</b>Cho tứ giác ABCD nội tiếp đường tròn

( )

O . Khi đó ta ln có:


AD.BC AB.CD AC.BD+ =


<b>+ Cơng thức về diện tích tam giác.</b>Cho tam giác ABC nội tiếp đường trịn

(

O; R

)

khi


đó ta ln có S<sub>ABC</sub> AB.BC.CA
4R



</div>
<span class='text_page_counter'>(33)</span><div class='page_container' data-page=33>

O
Q


P


N


M
D


C


B
A


<b>Trở lại bài tốn.</b>Gọi

( )

O là đường trịn tiếp xúc với các tia đối của BA, DA, CD, CB
theo thứ tự tại M, N, P, Q. Khi đó theo tính chất hai tiếp tuyến cắt nhau ta có


AM AQ; BM BP; CN CP; DN AQ.= = = = Khi đó ta thấy


AM AB BM AB BP AB BC CP


AQ AD DQ AD DN AD DC CN


= + = + = + +


= + = + = + +


Do ta có AM AQ; CN CP= = nên ta được AB BC AD DC+ = +



Mà lại có 2p a b c d= + + + nên ta được a b c d p+ = + = .


Đặt BD f= . Khi đó ta có S<sub>ABCD</sub> =S<sub>ABC</sub>+S<sub>ADC</sub> =S<sub>ABD</sub>+S<sub>BCD</sub>


Áp dụng cơng thức về diệntích trên cho các tam giác ABC, ADC, ABD, BCD nội tiếp
đường tròn ta được


ABC ACD


ABD BCD


AB.BC.CA abe AC.CD.DA cde


S ; S


4R 4R 4R 4R


AB.BD.DA adf BC.CD.DB bcf


S ; S


4R 4R 4R 4R


= = = =


= = = =


Do đó ta được S<sub>ANCD</sub>

(

ab cd e

)


4R

+


= hoặc S<sub>ANCD</sub>

(

ad bc f

)



4R
+
=


Nhân theo vế hai đẳng thức trên ta được


(

)(

)

<sub>(</sub>

<sub>)(</sub>

<sub>) ( )</sub>



2 2 2


ABCD 2


ab cd ad bc ef


S 16S .R ab cd ad bc ef *


16R


+ +


=  = + +


Áp dụng định lí Ptoleme cho tứ giác nội tiếp ABCD ta được


AD.BC AB.CD AC.BD+ = hay ac bd ef+ =



Từ S<sub>ANCD</sub>

(

ab cd e

)


4R
+


= ta được ab cd 4R.S


e


+ = thay vào hệ thức

( )

* ta được


(

)(

)

(

) (

)

(

) (

)



(

)



2 2 2 2 2 2


2


2 2


4 Re.S ac bd ad bc ab c d cd a b ab p 2cd cd p 2ab


4Rp S


p ab cd 4abcd 4S


2


= + + = + + + = − + −



</div>
<span class='text_page_counter'>(34)</span><div class='page_container' data-page=34>

Do đó ta được <sub>Rp</sub>2 =<sub>Re</sub>2+<sub>e.S</sub><sub> hay </sub>


2 2


S.e
R


p e


=


Mặt khác ta có a2 b2 c2 d2

(

a b

) (

2 c d

)

2 2ab 2cd 2p2 2 4SR
e
 


+ + + = + + + − − = <sub>− </sub> <sub></sub>


 


Do đó ta được a2 b2 c2 d2 8SR 2p2
e


+ + + + = .


<b>Bài 25.</b>Cho tam giác ABC có E, F tuộc đoạn CA và BA sao cho EF song song với BC.


Đường trung trực của đoạn thẳng BC cắt AC tại M, đường trung trực của đoạn EF cắt
cắt AB tại N. Đường tròng ngoại tiếp tam giác BCM cắt CF tại P khác C, đường trong
ngoại tiếp tam giác EFN cắt CF tại Q khác F. Chứng minh rằng đường trung trực của


PQ đi qua trung điểm của MN.


<b>Lời giải</b>
Gọi H và K theo thứ tự là hình chiếu của M và
N trên CF. Gọi S, T, U, V theo thứ tự là trung
điểm của MN, HK, BC, EF. Dễ thấy các tứ giác
BCMP và NEFQ nội tiếp đường trịn. Do đó ta
được MPH MBU= và NQK VEN= .


Lại có MHP=MUB 90 ; NKQ= 0 =NVE 90= 0
Suy ra MHP ∽ MUB và NKQ∽ NVE


nên ta được HP UB KQ; VE


MP =MB NQ =NE


N


V


U
S


T
Q


P
M


H


K


F E


C
B


A


Để ý U, V lần lượt là trung điểm của BC và EF nên ta có


HP UB BC MP


HP MP. MP. .


MP MB 2 MB


KQ VE EF NQ


KQ NQ. NQ. .


NQ NE 2 NE




= = =






 <sub>=</sub> <sub>=</sub> <sub>=</sub>





Do các tứ giác BCMP và NEFQ nội tiếp và lại có EFsong song với BC nên ta được


MBP MCP ECF; PMB PCB EFC
NQE NFE CBF; QNE EFC BCF


 <sub>=</sub> <sub>=</sub> <sub>=</sub> <sub>=</sub>





= = = =


</div>
<span class='text_page_counter'>(35)</span><div class='page_container' data-page=35>

Do đó ta được MBP∽ FCE và NQE∽ CBF suy ra MP FE
MB=FC và


NQ CB


NE =CF . Từ


đó ta được HP BC.EF KQ
2CF


= = . Kết hợp với TP TQ= suy ra TH TK= . Mà ta có MH


song song với NK và SM SN= nên suy ra ST, MH, NK song song với nhau. Điều này
có nghĩa là SI là đường trung trực của QP. Từ đó ta được đường trung trực của PQ đi


qua trung điểm của MN.


<b>Bài 26.</b> Cho tam giác ABC nhọn nội tiếp đường tròn

( )

O với đường cao AD. Tiếp
tuyến tại B và C vớiđường tròn

( )

O cắt nhau tại T. Trên đoạn thẳng AD lấy K sao cho


0


BKC=90 . Gọi G là trọng tâm tam giác ABC, KG cắt OT tại L. Lấy các điểm P và Q


thuộc đoạn BC sao cho LPsong song với OB và LQ song song với OC. Các điểm E và


F lần lượt thuộc đoạn AC và AB sao cho QE và PF cùng vng góc với BC. Gọi

( )

O là


đường tròn tâm T đi qua B, C. Chứng minh rằng đường tròn ngoại tiếp tam giác AEF
tiếp xúc với đường tròn

( )

T .


<b>Lời giải</b>
Gọi H là trực tâm tam giác ABC, gọi M
và I lần lượt là trung điểm của BC và EF.
Gọi N và Y theo thứ tự là giao điểm của
BH và TO với AC, X là giao điểm của hai
đường tròn ngoại tiếp hai tam giác BMF
và CME. Ta thấy G là giao điểm của OH
và MA. Điểm M và I thuộc đoạn OT. Xét


hai tam giác AHN và YCM có


0


ANB CMY= =90 và HAN MYC= nên



hai tam giác AHN và YMC đồng dạng
với nhau


N
O
Y


Q
P


F


E
I


L
G
K


X


T
H


D C


B
A



Suy ra ta được AH HN


CY =CM. Lại có OM song song với HA và ba điểm H, G, O thẳng


hàng nên theo định lí Talet ta được KA LM


</div>
<span class='text_page_counter'>(36)</span><div class='page_container' data-page=36>

Mà LQ song song với CO nên theo định lí Talets ta có LM QM


LO = QC . Mà QE song song


với MY nên ta được QM EY


QC =EC. Từ đó suy ra


KA LM QM EY


KH= LO = QC =EC. Do đó ta được


KA KH EY EC


KH EC


+ +


= nên AH CY


KH = CE hay


AH KH



CY = CE . Do đó ta được


AH HN KH


CY =CM= CE


nên suy ra hai tam giác KHN và ECM đồng dạng với nhau. Mặt khác ta thấy tứ giác
BKNC nội tiếp nên KCB KHN EMC= = . Tương tự ta được KBC=FMB.


Do đó FME 180= 0−FMB EMC 180− = 0−KBC KCB− =BKC 90= 0. Ta có biến đổi góc


(

) (

)



0 0 0 0


FXE=360 −FXM EXM− = 180 −FXM + 180 −EXM =FBM ECM 180+ = −FAE


Do đó tứ giác AFXE nội tiếp đường trịn. Lại có


0 0 0


0


0 0 0


BXC 360 FXB EXC FXE 180 FMB EMC 180 FXE


180 2BAC BTC


FME BAC 90 BAC 180 180



2 2


= − − − = − − + −




= + = + = − = −


Do đó X thuộc đường tròn tâm T. Từ các kết quả trên suy ra X là một giao điểm của
đường tròn ngoại tiếp tam giác AEF với đường tròn

( )

T . Gọi X’ là giao điểm thứ hai
của đường tròn ngoại tiếp tam giác AEF với đường tròn

( )

T . Khi đó ta lại có
BXF=BXX ' FXX '+ =BCX ' FEX '+ . Lại có tứ giác BFXM nội tiếp đường tròn nên


0


EMF EQM 90= = , EQ song song với IM và IM IE IF= = . Từ đóta có biến đổi góc


FXB FMB MEQ EMI IEM IEX MEX FEX MCX FEX BCX= = = = = + = + = +


Kết hợp các kết quả lại ta được BCX ' FEX ' FEX BCX+ = + , suy ra X và X’ trùng nhau.


Vậy đường tròn ngoại tiếp tam giác AEF tiếp xúc với đường tròn

( )

T .


<b>Bài 27.</b>Cho tam giác ABC và một điểm D trên cạnh BC(D khác B và C). Gọi E và F lần
lượt là tâm đường tròn nội tiếp các tam giác ABD và ACD. Chứng minh rằng nếu bốn
điểm B, C E, F cùng nằm trên một đường trịn thì ta có AD DB AB


AD CD AC



+
=


+ .


<b>Lời giải</b>


</div>
<span class='text_page_counter'>(37)</span><div class='page_container' data-page=37>

S
R


M


N
I


P


Q
F


E


D C


B


A


Trước hết ta biểu diễn được các tỉ số bằng nhau có chứa AD BD+ và AD CD+ . Theo



tính chất đường phân giác trong tam giác ta được


AD AM AD DB AM BM AB AD BD AD


BD BM AD AM AM AB AM


AD AN AD DC AN NC AC AD DC AD


CD CN AD AN AN AC AN


+ + +


=  = =  =


+ + +


=  = =  =


Như vậy bài toán sẽ được chứng minh nếu ta chỉ ra được AM AN= .


Thật vậy, do tứ giác BEFC nội tiếp nên ta được APQ ABE PEB ABE BCF= + = + .


Từ đó ta được APQ 1

(

ABC ACB

)


2


= + . Chứng minh hoàn toàn tương tự ta cũng được


(

)



1



AQP ABC ACB


2


= + . Như vậy tam giác APQ cân tại A nên AP AQ= . Qua A kẻ
đường thẳng song song với PQ và cắt DM, DN lầnlượt tại R, S.


Ta có EP AP AQ FQ


EI = AI = AI = IF nên ta được


IE EP


IF =FQ


Theo định lí Thales ta có IE AR


IF = AS . Từ đó ta được


EP AR


FQ = AS nên suy ra


EP FQ


AR = AS.


Mặt khác ta lại có EP MP



AR =MA và


FQ QN


AS= AN, từ đó ta được


MP NQ


MA=NA nên MN song


song với PQ. Mà AP AQ= nên suy ra AM AN= . Từ đó ta được AD DB AD DC


AB AC


+ <sub>=</sub> +


Do đó ta suy ra được AD DB AB


AD CD AC


+
=


+ . Vậy ta có điều phải chứng minh.


</div>
<span class='text_page_counter'>(38)</span><div class='page_container' data-page=38>

<b>Lời giải</b>
Trước hết ta phát biểu hai bổ đề sau.


<b>+ Bổ đề 1.</b><i>Cho tam giác ABC và hai điểm M, N sao cho </i>



2
2


BM.BN AB


CM.CN = AC <i>. Khi đó ta ln có </i>
BAM=CAN<i>. </i>


<b>+ Bổ đề 2. </b><i>Cho điểm A nằm ngoài đường tròn </i>

( )

O <i> và cát tuyến ABC. Các tiếp tuyến tại B, C</i>
<i>với đường tròn cắt nhau tại K. AE và AF là các tiếp tuyến với đường tròn </i>

( )

O <i>kẻ từ A (E, F</i>
<i>là các tiếp điểm). Khi đó ba điểm K, E, F thẳng hàng.</i>


Trong hai bổ đề trên thì bổ đề thứ nhất chính là tính chất của hai điểm đẳng
giác và bổ đề thứ hai chính là bài tốn quen thuộc về hai tiếp tuyến cắt nhau.
<b>Trở lại bài toán.</b> Gọi M, N, P, Q lần lượt là


các tiếp điểm của đường tròn với các cạnh
AB, BC, CD, DA. Lấy diểm D’ đối xứng với
Aqua AC. Gọi giao điểm của BD’ với AC là
H’. Dựng OH vng góc với AC, ta cần
chứng minh hai điểm H và H’ trùng nhau.
Gọi S là giao điểm của MP và BD, kẻ BK
song song với CD với K thuộc MP, khi đó ta


có BS BK


DS= DP. Mà ta có BKM CPM= =BMK


nên ta được BK BM= .



I
H'


K


H


E'


D'


Q


P


N
M


F


E


D C


B
A


Do đó BS BM


DS= DP . Gọi S’ là giao điểm của NQ và BD, khi đó tương tự như trên ta có



BS' BN


DS'=QD . Do BM BN= và DP DQ= nên ta được


BS BS'


DS= DS', suy ra hai điểm S và S’


trùng nhau. Từ đó suy ra ba đường thẳng BD, MP, NQ đồng quy tại S.Chứng minh
tương tự ta được AC, MP. NQ cũng đồng quy tại S.Từ cách dựng điểm H’ ta được
H’C là tia phân giác của góc BH' D nên suy ra BH' BS BM


DH'= DS = DP


Mặt khác ta có <sub>BE.BF BM , DE.DF DP</sub>= 2 = 2<sub>, suy ra </sub>
2


2


H' B BE.BF


</div>
<span class='text_page_counter'>(39)</span><div class='page_container' data-page=39>

Áp dụng bổ đề 1 cho tam giác H’BD ta được BH'E=DH'F. Từ đó FH' C EH' C= . Gọi
I và I’ lần lượt là giao điểm của NM, PQ với AC. Áp dụng định lí Menelaus cho tam
giác ABC với ba điểm M, N, I thẳng hàng và tam giác ACD với ba điểm P, Q, I’ thẳng


hàng ta được AM BN CI. . AQ DP CI '. . 1


BM CN AI = DQ CP AI '= . Từ đó suy ra



CI CI'


AI = AI', từ đó suy ra


hai điểm I và I’ trùng nhau. Điều này có nghĩa là MN, AC, PQ đồng quy tại I. Giả sử
tiếp tuyến với đường tròn (O) tại E, F cắt nhau tại I”. khi đó theo bổ đề 2 thì I” thuộc
MN và cũng thuộc PQ. Nên hai điểm I và I” trùng nhau. Tứ giác HEIF nội tiếp đường


trịn đường kính OI nên ta được EHC FHC= .


Gọi E’ là điểm đối xứng với E qua AC, khi đó ta có E'H'C EH'C FH'C


E'HC EHC FHC


 <sub>=</sub> <sub>=</sub>





= =





Từ đó suy ra ba điểm F, E’, H’ thẳng hàng và F, E’ H thẳng hàng.Từ đó suy ra hai
điểm H và H’ thẳng hàng, kết hợp với BH'E=DH'F ta được BHE DHF= .


Vậy bài toán được chứng minh.


<b>Bài 29.</b> Cho tam giác ABC nội tiếp đường tròn

( )

O . Đường tròn

( )

O ' tiếp xúc trong



với đường tròn

( )

O và tiếp xúc với các cạnh AB, BC lần lượt tại R, P, Q. Gọi K là tâm
đường tròn nội tiếp tam giác ABC. Chứng minh rằng ARK=CRK.


<b>Lời giải</b>
Tại R vẽ tiếp tuyến với đường tròn


( )

O , tiếp tuyến này cắt BC tại D. Khi


đó dễ thấy DR=DQ. Từ đó ta được


2


DC.DB DR= nên ta được


DR DC DQ DQ DC QC


DB DR DB DB DR QB




= = = =




Các tam giác RDC và BDR có BDR là
góc chung và RBC=DRC nên đồng
dạng với nhau.


E D



R


H
K


Q
P


N


M


C
B


</div>
<span class='text_page_counter'>(40)</span><div class='page_container' data-page=40>

Từ đó ta được RC DC QC


BR = DR = QB. Điều này chứng tỏ RM là đường phân giác của góc
CRB. Chứng minh hoàn toàn tương tự ta được RN là phân giác của góc ARB . Gọi
AM, CN lần lượt là phân giác trong của các góc BAC; ACB, khi đó giao của AM và
CN là điểm K. Xét các tam giác BQR và MCR có RBQ RBC RMC= = và BRM=MRC


nên hai tam giác đó đồng dạng với nhau, do đó suy ra BQ BR


MC= MR. Mặt khác ta lại có


(

)



1



KCM KCB BCM ACB BAC


2


= + = + và CKM KAC KCA 1

(

BAC ACB

)



2


= + = +


Do đó suy ra KCM CKM= nên tam giác KCM cân tại M. Từ đó ta được MK MC= ,


mà ta lại có BP BQ= nên kết hợp với các kết quả trên ta được BP BR


MK=MR . Mặt khác ta
lại có PBR=KMR nên hai tam giác BPR và MKR đồng dạng với nhau.. Từ đó dẫn
đến PRB NRB KRM ERM= = = , nên suy ra BN=AN EM= . Ta có BN BE EM BE+ = +
hay EC EA= . Điều này chứng tỏ điểm E nằm chính giữa cung ABC. Do đó ta được


ARK=CRK. Bài tốn được chứng minh.


<b>Bài 30.</b>Giả sử M và N là các điểm nằm trong tam giác ABC sao cho MAB NAC= và


MBA=NBC . Chứng minh rằng AM.AN BM.BN CM.CN 1


AB.AC + BA.BC + CA.CB = .


<b>Lời giải</b>


Trên tia BN lấy điểm K sao cho BCK=BMA. Khi đó ta được ABM=KBC. Xét hai


tam giác BMA và BCK có BCK=BMA và ABM=KBC nên BMA ∽ BCK, do đó ta


được AB BM AM


BK = BC = KC hay


AB BK


BM = BC. Mặt khác ta lại có


ABK=ABM MBK; MBC MBK KBC+ = +


Nên ta được ABK=MBC. Hai tam giác
ABK và MBC có AB BK


BM = BC và ABK=MBC


nên đồng dạng với nhau.


K


N
M


C
B


</div>
<span class='text_page_counter'>(41)</span><div class='page_container' data-page=41>

Từ đó suy ra AB BK AK


MB= BC =CM.



Kết hợp với AB BM AM


BK = BC = KC ta thu được


AM.BC AB.CM AB.BC


CK ; AK ; BK


BM MB BC


= = =


Ta lại có MAB NAC= và MAB=NKC nên ta được NAC=NKC, từ đó suy ra tứ giác
AKCN nội tiếp đường trịn.


Áp dụng định lí Ptoleme ta được AC.NK AK.NC AN.CK= +


Hay ta được AC. BK BN

(

)

=AK.NC AN.CK+


Nên suy ra AC AB.BC BN AB.CM.CN AM.BC.AN


BM BM BM


 


− = +


 



 


Từ đó suy ra AB.BC.AC BN.BM.AC AB.CM.CN BC.AN.AN− = + . Chia cả hai vế cho


AB.BC.CA và chuyển vế ta thu được AM.AN BM.BN CM.CN 1
AB.AC + BA.BC + CA.CB = .


Bài toán được chứng minh xong.


<b>Bài 31.</b> Cho tứ giác ABCD có 0


ABC CDA 90= = . Gọi H là hình chiếu của A trên BD.


Gọi S và T lần lượt là điểm nằm trên AB, AD sao cho H nằm trong tam giác SCT và


0 0


CHS CSB 90 ; THC DTC 90− = − = . Chứng minh rằng đường thẳng BD tiếp xúc với
đường tròn ngoại tiếp tam giác TSH.


<b>Lời giải</b>
Gọi E và F lần lượt là điểm đối xứng với


C qua B và D. Ta có SE SC= do đó tam
giác SCE cân tại S, suy ra ta được


SEB SCB= . Mà CHS CSB 90− = 0 nên ta


được 0 0



CHS CSB 90= + =180 −SCB suy ra


tứ giác CHSE nội tiếp đường tròn. Chứng
minh tương tự ta cũng được tứ giác


CHTF nội tiếpđường tròn. P


M


H
T


S


F
E


D


C
B


A


</div>
<span class='text_page_counter'>(42)</span><div class='page_container' data-page=42>

ESC EHC= mà hai tam giác SEC và HEP lần lượt cân tại S và H nên hai tam giác SEC


và HEP đồng dạng với nhau, do đó ta được SEH=CEP. Từ đó ta lại có hai tam giác


SHE và CEP đồng dạng với nhau. Do vậy SHE SCE HPE HEP= = = nên SH song song



với EP. Mà tứ giác CHTF nội tếp đường trịn nên ta có FTC CHF= suy ra hai tam giác


TCF và HPE đồng dạng với nhau. Do SHE SCE= nên HS là phân giác ngoài của góc


CHE. Tương tự ta có HT là phân giác ngồi của góc CHF. Từ đó SHT=SCB TCD+
nên EPF SHT= . Lại có HS HS HF. CP FP. FP


HT =HE HT= EP CP =EP nên hai tam giác SHT và FPE


đồng dạng với nhau, do đó suy ra STH FEP= . Kết hợp với BD//EF và SH//EP nên ta
được STH SHB= . Vậy BD là tiếp tuyến của đường tròn ngoại tiếp tam giác HST hay
đường thẳng BD tiếp xúc với đường trong ngoại tiếp tam giác HST.


<b>Bài 32.</b> Cho tam giác ABC có trực tâm H. Đường phân giác ngồi của góc BHC cắt
cạnh AB, AC lần lượt tại D và E. Đường phân giác trong của góc BAC cắt đường trịn
ngoại tiếp tam giác ADE tại K. Chứng minh rằng KH đi qua trung điểm của đoạn BC.


<b>Lời giải</b>
Trước hết ta chứng minh tam giác ADE cân tại A.


Thật vậy, vì HD là phân giác ngồi của góc BHC nên ta có


(

) (

0

) (

0

)



1 1 1


DHB HBC HCB 90 ABC 90 ACB BAC


2 2 2



 


= + = <sub></sub> − + − <sub></sub>=


 


Do đó ta được <sub>ADE DBH DHB 90</sub>0 <sub>BAC</sub> 1<sub>BAC 90</sub>0 1<sub>BAC</sub>


2 2


= + = − + = −


Tương tự ta cũng có <sub>AED 90</sub>0 1<sub>BAC</sub>


2


= − , suy ra


ADE AED= hay tam giác ADE cân tại A. Mặt khác
ta có AK là phân giác của góc DAE nên cũng là
đường trung trực của đoạn DE, từ đó suy ra AK là
đường kính của đường trịn ngoại tiếp tam giác


ADE.


K
H


Q
P



C'
B'


D


E


C
B


A


</div>
<span class='text_page_counter'>(43)</span><div class='page_container' data-page=43>

Từ đó ta có KD vng góc với AB. Chứng minh tương tự ta được KE vng góc với


AC. Gọi P là giao điểm của KD vàHB, Q là giao điểm của KE và HC. Ta có KP và HQ


cùng vng góc với AB nên KP song song với HQ. Tương tự ta cũng được KQ song


song với HP, do đo tứ giác KPKQ là hình bình hành.Gọi BB’ và CC’ là các đường cao
của tam giác ABC, ta có DPsong song với HC’ và QE song song với HB’. Do đó theo


định lí Thales ta có PB DB ; QC EC


PH = DC' QH= EB'. Theo tính chất đường phân giác của tam
giác ta có DB HB ; EC DB


DC'=HC' EB' =HB'. Vì B, C, B’, C’ cùng thuộc đường trịn đường kính
BC nên hai tam giác BHC’ và CHB’ đồng dạng với nhau, từ đó ta được HB HC



HC' =HB'.
Kết hợp với các kết quả trên ta được PB QC


PH =QH, nên theo định Thales ta được PQ


song song với BC. Do HK đi qua trung điểm của PQ nên HK cũng đi qua trung điểm
của BC.


<b>Bài 33. </b>Cho tam giác ABC có AB AC 3BC+ = . Đường tròn

( )

I nội tiếp tam giác ABC
tiếp xúc với BC tại D. Gọi T là một điểm đối xứng với D qua I. Các đường thẳng BT và
CT cắt cạnh AC, AB lần lượt tại M, N. Chứng minh rằng tâm đường tròn ngoại tiếp
tam giác TMN nằm trên AI.


<b>Lời giải</b>
Gọi X là trung điểm của AI. Ta sẽ chứng minh X là
tâm đường tròn ngoại tiếp tam giác TMN. Gọi E, F
là tiếp điểm của đường tròn

( )

I với AC và AB. Gọi
đường tròn

( )

O ngoại tiếp tam giác ABC. Gọi giao
điểm của AI với đường tròn

( )

O là J. Gọi giao
điểm của đường tròn

( )

I với đường tròn ngoại tiếp
tam giác IBC là K, L sao cho K nằm giữa L và C. Do


các tứ giác BCIL và TLDE nội tiếp đường trịn kết
hợp với tính chất của hai tiếp tuyến cắt nhau khi
đó ta có BLI 180= 0−ICB và TLE TDE ICB= = .


N


M
A



B D C


I


J
E
F


K
L


</div>
<span class='text_page_counter'>(44)</span><div class='page_container' data-page=44>

AB. Ta có 0


MTN=BTC EDF 90= = −BAC. Đặt AB c= và p là nửa chu vi tam giác
ABC. Ta lại có AM CE p AB= = − . Ta cũng có


(

)

AC BC AB


NF AB 2BF AB AB BC CA AC BC p c


2
+ −


= − = − + − = − = = −


Nên ta được AM NF= , từ đó suy ra AXM= FXN do đó FNX=AMX


Từ đó suy ra tứ gác AMXN nội tiếp. Lại có AX là phân giác của góc MAN nên
MAX=NAX hay AM=XN. Do đó ta được XM XN= nên X nằm trên đường trung



trực của MN.Lại có 0


NXM 180= −BAC 2MTN= nên X là tâm đường tròn ngoại tiếp


tam giác MNT


<b>Bài 34.</b>Cho tam giác ABC nội tiếp đường tròn

( )

O có AB BC 2BC+ = . Đường trịn

( )

I


nội tiếp tam giác ABC tiếp xúc với các cạnh BC, CA, AB lần lượt tại D, E, F. Gọi


b c


M , M lần lượt là trung tuyến của CA và AB. Giao điểm của M Mb c và EF là L.


Chứng minh rằng đường tròn

(

L; LI

)

tiếp xúc với các đường trịn

( )

I và đường trịn
đường kính AO.


<b>Lời giải</b>


Gọi Ma là trung điểm của BC. Gọi giao điểm của


AI với đường tròn

( )

I là K. Do đường tròn

( )

I nội
tiếp tam giác ABC nên ta suy ra AB, BC, CA là các
tiếp tuyếncủa đường tròn

( )

I . Theo tính chất tiếp
tuyến cắt nhau và kết hợp giả thiết ta được


AB AC BC 2BC BC BC


AE AF



2 2 2


+ − −


= = = =


K
D


L
I O
F


E
M<sub>c</sub> M<sub>b</sub>


M<sub>a</sub> C
B


A


Từ đó suy ra AF=BM<sub>a</sub>. Do AK là phân giác của góc BAC nên K nằm chính giữa
cung BC khơng chứa A của đường trịn

( )

O .


</div>
<span class='text_page_counter'>(45)</span><div class='page_container' data-page=45>

Nên được FAI=KBM<sub>a</sub>và KM<sub>a</sub> ⊥BC . Từ đó dẫn đến AFI= BM K<sub>a</sub> nên AI=BK.


Ta lại có IBK IBC CBK 1

(

BAC ABC , BKI ACB

)


2



= + = + = , do đó ta được


(

)

(

)



(

) (

)



0 0 1


BIK 180 IBK BKI 180 BAC ABC ACB


2


1 1


BAC ABC BAC ABC BAC ABC IBK


2 2


= − + = − + −


= + − + = + =


Do đó ta được tam giác BKI cân tại K, suy raBK=IK. Khi đó ta được AI=BK=DI,


suy ra I là trung điểm của AK. Do đó ta được AK vng góc với OI. Do O là tâm


đường tròn ngoại tiếp tam giác ABC nên suy ra OM , OM , OM<sub>a</sub> <sub>b</sub> <sub>c</sub> lần lượt là đường


trung trực của BC, CA, AB. Nên ta được 0



c b


AM O=AM O 90= . Từ đó suy ra M , M<sub>b</sub> <sub>c</sub>


thuộc đường trịn đường kính AO. Lại có 0


AIO 90= nên I thuộc đường tròn đường


kinh AO. Do IE⊥AC, IF⊥AB nên E, F nằm trên đường thẳng Simson của I đối với


tam giác AM M<sub>b</sub> <sub>c</sub>. Từ đó suy ra IL⊥M M<sub>b</sub> <sub>c</sub> nên ba điểm I, L, D thẳng hàng. Điều này
dẫn đến ba điểm A, L, M<sub>a</sub> thẳng hàng nên từ đó ta suy ra L là trung điểm của M M<sub>b</sub> <sub>c</sub>.


Gọi T là tâm đường trịnđường kính OA, khi đó TI đi qua trung điểm L. Từ đó hai
đường trịn

(

L; LI

)

( )

T tiếp xúc với nhau.Từ IA=IK và M Ka =ID ta được


DI
LI


2


= nên suy ra

(

L; LI

)

tiếp xúc với đường tròn

( )

I .


<b>Bài 35.</b>Cho tam giác nhọn ABC khơng cân nội tiếp đường trịn

( )

O có các đường cao
AD, BE, CF cắt nhau tại H. Đường thẳng EF cắt BC tại P. Qua D vẽ đường thẳng song
song với EF cắt AC tại Q và cắt AB tại M. Chứng minh rằng đường tròn ngoại tiếp
tam giác PQM di qua trung điểm của BC.


</div>
<span class='text_page_counter'>(46)</span><div class='page_container' data-page=46>

Vẽ đường kính AA’ của đường tròn

( )

O .



Gọi giao điểm của A’H với BC là S và cắt
đường tròn

( )

O tại điểm thứ hai là K. Khi
đó dễ thấy HB song song với A’C và CF
song song với A’B. Từ đó suy ra tứ giác
BHCA’ là hình bình hành, nên S là trung
điểm của BC.


O


S
A'


K


M


P
Q


H'
H
F


E
D C
B


A


Như vậy để chứng minh đường tròn ngoại tiếp tam giác PQM di qua trungđiểm của


BC. Ta cần chứng minh được tứ giác MSQP nội tiếpđường tròn. Thật vậy, dễ thấy
được A' AC=A' KC CBA'= . Suy ra ta có CAH HAA' AKE KEC+ = + =A' BC. Dễ thấy
tứ giác AKEH nội tiếp đường tròn nên ta được HAE HKE= , suy ra A' AD EKC= mà
tứ giác BFEC nội tiếp đường tròn nên ta có ABC FEA= , do đó ABD FEA CEP= = . Lại


có 0


ABC CBA' 90+ = nên ta được 0


A' AH HAC ABC A' AH HAC FEA 90+ + = + + =


Do đó ta suy ra được AA’ vng góc với EF. Từ đó ta có


0 0


ACB=BPF FEA+ 90 −CAH=BPF 90+ −CAH A' AH− BPH=A' AH EKC=


Từ đó dẫn đến tứ giác EKPC nội tiếp đường tròn, nên ta được FEA CEP= =CKP. Do


ta có 0


FEA CAH A' AH FEA HKE EKC 90+ + = + + = nên 0


HKE EKC CKP+ + =90 . Từ
đó ta được AKH PKH 180+ = 0 nên ba điểm A, K, P thẳng hàng và H là trực tâm tam


giác APS. Gọi giao điểm thứ hai của đường tròn

( )

O với AD là H’. Đến đây ta thấy tứ
giác ASH’P nội tiếp đường tròn nên suy ra DS.DP AD.DH' DB.DC= = . Lại do MQ
song song với EF nên MQA FEA ABC= = nên tứ giác BMQC nội tiếp đường trịn.Từ
đó ta được DC.DB DQ.DM= Kết hợp hai kế quả trên ta được DM.DQ DS.DP= nên tứ

giác MSDP nội tiếp đường tròn. Vậy đường tròn ngoại tiếp tam giác PQM di qua
trung điểm của BC.


</div>
<span class='text_page_counter'>(47)</span><div class='page_container' data-page=47>

tại K và


2


R .AH
KI.KJ


BC


= .


<b>Lời giải</b>


Gọi R ,R1 2 lần lượt là bán kính đường


trịn nội tiếp các tam giác ABH, ACH.
Đặt AB c, BC a,CA b,AH h= = = = .


Gọi giao điểm của AI, AJ với BC lần
lượt là M, N.Gọi giao điểm của BI với


AN là E và giao điểm của CJ với AM


là F.


K



D
Q


N
M


F E
V
T J
I


H C


B


A


Từ giả thiết ta được ABF HAF= =BCF ACF= và ABE CBE HAE CAE= = = .


Khi đó tam giác ABC đồng dạng với tam giác HBA nên AB R1 BH


BC= R = AB (1) và tam


giác ABC đồng dạng với tam giác HAC nên AC R2 CH


BC = R = AC(2).


Từ (1) và (2) ta được


2 2 2 2



2 2 2


1 2


1 2


2 2


R R AB AC


1 R R R


R BC


+ +


= =  + = (3)


Ta có

(

R1+R2

)

2 2 R

(

12+R22

)

nên suy ra R1+R2  2.R (4)


Cũng từ (1) và (2) và áp dụng bất đẳng thức Cauchy ta được


2


1 2


1 2


R R BH CH BH.CH AH h h



2 2 2 R R 2R.


R AB AC AB.AC BC.AH a a


+


= +  = =  +  (5)


Dễ thấy h AD a
2


 = hay 0 x h 1


a 2


 =  nên x 1 2x

(

)

  0 x 2x2  x 2.x.


Do đó ta suy ra h 2.h


a  a, nên ta được 1 2


2 2.h.R


R R


a


+  (6).



Đẳng thức xẩy ra khi và chỉ khi R<sub>1</sub> =R<sub>2</sub> hay tam giác ABC vuông cân tại A.
Từ (1) và (2) ta có


2 2


1 2 2


R.AB R.AC R .ah R .h


R R .


BC BC a a


= = = (7)


Từ tâm Q của đường trịn nội tiếp tam giác ABC hà đường vng góc xuống các cạnh
của tam giác ABC ta có hệ thức b c a 2R+ = + . Áp dụng hệ thức này vào các tam giác


ABH và ACH ta được


</div>
<span class='text_page_counter'>(48)</span><div class='page_container' data-page=48>

(

1 2

) (



2 R R R AH


<b>Website: </b>


<b>tailieumontoan.com</b> BH AB

(

)

AH CH AC

(

)

AB 
AC BC

)

2AH


+ + =



Từ đó ta được R R+ <sub>1</sub>+R<sub>2</sub> =h (8)


Kẻ TJ vng góc với AH tại T và JP//AH, IP//CB cắt JP tại P và cắt AH tại V. Tam giác
IJP vng tại P có 2 2 2


IJ =IP +JP hay 2

(

) (

2

)

2


1 2 1 2


IJ = R +R + R −R  =IJ 2R=AQ (9)


Ta có

(

)

0


ABE BAE+ = ABE HAE+ +BAH=ABC ACB 90+ = và BA BN= , AE EN= .


Tương tự ta có CF⊥AM và CA CM,AF FM= = . Do đó EF là đường trung bình của


tam giác AMN nên EF//BC và EFC FCB= (10). Từ đó suy ra các điểm I, J, E, F nằm
trên đường trịn đường kính IJ. Từ IE⊥AJ, JF⊥AI và IE cắt JF tại Q nên AK chính là
đường cao của tam giác AIJ hay AQ⊥IJ tại K. Từ đó ta được A, J, K, F, T thuộc
đường tròn đường kính AJ và A, I, E, K, V thuộc đường trịn đường kính AI. Từ đó ta
được EAK IAH= và EIJ EFJ= , Kết hợp với (10) ta được EAK IAH= , suy ra


IAV JAK


 ∽  nên ta có IV KJ R1 AI


AI= AJKJ =AJ (11).



Hoàn toàn tương tự ta được FAK=JAH nên JAT∽ IAK suy ra R1 AJ


KI= AI (12)


Từ (11) và (12) ta được 1
2


R KI


KJ =R nên KI.KJ=R .R1 2.


Kết hợp với (7) ta được


2


R .AH
KI.KJ


BC
=


<b>Bài 37. </b>Cho hai đường tròn tâm O và O’ cắt nhau tại A và B sao cho OA vng góc với
O’A, OO’ cắt hai đường trịn tại C, E, D, F sao cho các điểm C, O, E, D, E, O’, F nằm
trên đường thẳng theo thứ tự đó. BE cắt đường trịn (O) tại điểm thứ hai là K và cắt
CA tại M. BD cắt đường tròn (O’) tại điểm thứ hai là L và cắt AF tại N. Chứng minh
rằng: KE LN. O'E


KM LD = OD


<b>Lời giải</b>



Ta có AOO' 2ACF= và AO' O=2AFC. Mà 0


OAO' 90= nên 0


</div>
<span class='text_page_counter'>(49)</span><div class='page_container' data-page=49>

Từ đó suy ra ACF AFC+ =450. Mà tam


giác O’AL cân nên ta lại có


(

0

)

0


0 0 0


1 1


O' AL 180 AO' L 90 AO' L


2 2


90 ABL 90 ABD 90 ACF


= − = −


= − = − = −


Mặt khác CAL CAO OAO' O' AL= + + .


Do đóta được


F


K


L


O'
O


N
M


E D
C


B
A


0 0 0


CAL=ACF 90+ +90 −ACF 180=


Từ đó suy ra ba điểm C, A, L thẳng hàng. Chứng minh tương tự ta được K, A, F thẳng


hàng. Áp dụng định lí Menelaus cho tam giác CME vớiba điểm K, A, F thẳng hàng ta


có KE AM FC. . 1
KM AC FE = hay


KE AC.EF


KM =AM.FC. Áp dụng định lí Menelaus cho tam giác CME



với ba điểm K, A, F thẳng hàng ta có LD AN CF. . 1
LN AF CD= hay


LN AN.CF


LD = AF.CD. Mà ta lại có


EF 2EO' O'E


CD= 2OD = OD . Do đó


KE LN AC.EF.AN.CF AC.AN.FE AC.AN.O'E


.


KM LD =AM.FC.AF.CD= AM.AF.CD= AM.AF.OD . Do


hai tứ giác AEBF và ACBD nội tiếp nên ta được EBA EFA; ABD ACD= = . Suy ra


(

)



0 0 0 0


0


MAN CAF 180 ACF AFC 180 45 135


MBN EBA ABD EFA ACD CAF 45



 <sub>=</sub> <sub>=</sub> <sub>−</sub> <sub>+</sub> <sub>=</sub> <sub>−</sub> <sub>=</sub>





 <sub>=</sub> <sub>+</sub> <sub>=</sub> <sub>+</sub> <sub>=</sub> <sub>=</sub>




Từ đó ta được MAN MBN 180+ = 0 nên tứ giác AMBN nội tiếp đường trịn.
Từ đó ta được AMN=ABN=ACD=ACF, suy ra MN song song với CF.
Theo định lí Thales ta được AC AF


AM=AN hay


AC.AN
1
AM.AF= .


Do đó từ KE LN. AC.AN.O'E


KM LD = AM.AF.OD ta thu được


KE LN O'E


.


KM LD = OD .


<b>Bài 38.</b>Cho tam giác ABC nhọn có các đường cao AA’, BB’, CC’. Gọi D, E, F lần lượt là


tâm đường tròn bàng tiếp trong các góc B' AC', C' A' B, B' A' C của các tam giác
AB’C’, BC’A’, CA’B’ tương ứng. Đường trịn bàng tiếp trong góc BAC của tam giác
ABC tiếp xúc với BC, CA, AB lần lượt tại M, N, P. Chứng minh rằng tâm đường tròn
ngoại tiếp tam giác DEF là trực tâm tam giác MNP.


</div>
<span class='text_page_counter'>(50)</span><div class='page_container' data-page=50>

Gọi I là tâm đường tròn bàng tiếp góc A của
tam giác ABC. Gọi H là tiếp điểm của đường


tròn

( )

E với đường thẳng BC. Nhận thấy ba
điểm E, B, I thẳng hàng và ba điểm I, C, F
thẳng hàng. Dễ thấy tứ giác AC’A’C nội tiếp
đường trịn,do đó ta có


1 1


EA'H BA'C' BAC IAP


2 2


= = = I


P


N
M


E


F
D



H


O


C'
B'


A' C
B


A


Xét hai tam giác vng EA’H và IPA có EA'H IAP= nên suy ra hai tam giác EA’H và
IAP đồng dạng với nhau, do đó ta được HA' EH


PA = IP . Mà ta lại có IM IP, MB PB= = và
HE song song với IM nên ta có EH HB


IM = BM.


Từ đó suy raHA' EH EH HB HB


PA = IP = IM = BM= PB suy ra


HB PA


HA'= PB . Do đó ta được HP song


song với AA’. Nên theo tiên đề Ơclit thì ba điểm E, H, P thẳng hàng, do đó IM song



song với EP. Từ đó ta được PEI PIB EHM= = nên ta có tam giác EPI cân tại P, suy ra


EP PI IM= = . Tứ giác EPIM có EP song song với IM và lại có EP PI IM= = nên nó là
hình thoi. Hồn tồn tương tự ta cũng được PIND, FNIIM là các hình thoi. Từ đó ta
được EM, IP, DN song song với nhau và EM PI= =DN; đồng thời ta có FM, NI, DP


song song với nhau và FM NI DP= = . Suy ra các tứ giác EMND, FMPD là các hình
bình hành nên ta được PMsong song với FD và MN song song với DE. Gọi O là tâm
đường tròn ngoại tiếp tam giác DEF, kho đó ta có OD OE OF= = . Kết hợp với


PE PI= =PD nên ta được PO là đường trung trực của DE hay vng góc với DE. Mà
ta có DE song song với MN nên ta có PO vng góc với MN. Chứng minh tương tự ta
đượcNO vng góc với MP. Vậy O là trực tâm tam giác MNP.


<b>Bài 39.</b>Cho tam giác ABC vuông tại A. Gọi I là giao điểm ba đường phân giác của tam
giác. Gọi D, E, F lần lượt là hình chiếu của I trên các cạnh BC, AC, AB. Gọi M là trung
điểm cạnh AC. Đường thẳng MI cắt AB tại N, đường thẳng DF cắt đường cao AH tại
P. Chứng minh tam giác ANP là tam giác cân.


</div>
<span class='text_page_counter'>(51)</span><div class='page_container' data-page=51>

Ta có IE và AB cùng vng góc với AC
nên IE song song với AB. Theo định lí


Thales ta có AN AM


EI = EM . Từ đó ta suy


ra được


(

)




AM.EI AC.EI


AN


EM 2 AM AE


= =




Vì BAC 90= 0 nên tứ giác AEIF là hình
vng, do đó suy ra AE=EI.


K


H
P


N I


M
F


E


D C


B



A


Vì D, E, F lần lượt là hình chiếu của I trên các cạnh BC, AC, AB nên ta có


(

)



1


AE CD BD AB BC CA


2


+ + = + + nên IE AE AB CA BC


2
+ −
= =


Lại có BC AB CD AF CE AE− = − = − =2 AM AE

(

)

. Kết hợp các kết quả lại ta được


(

)

(

(

)

)

(

)



2 2


AC AB CA BC


AC.EI BC AB AB.AC AC.BC BC AB AC


AN



2


2 AM AE 2 BC AB 2 BC AB


+ − <sub>−</sub> <sub>+</sub> <sub>−</sub> <sub>+</sub> <sub>−</sub>


= = = =


− − −


Từ A kẻ đường thẳng song song với BC cắt DF tại K, khi đó tam giác APK vng tại


A. Ta có BD=BFdo đó ta có AK=AF AE= và AKF BDF 900 1B BIF
2


= = − = . Trong


tam giác AKP có <sub>AP AK.tan AKF AK.tan 90</sub>0 1<sub>B</sub> <sub>AK.cot</sub>B <sub>AE.cot</sub>B


2 2 2


 


= = <sub></sub> − <sub></sub>= =


  . Do


đó AP AC AB BC BD.


2 ID



+ −


= Mà BD BC AB AC


2
+ −


= và DI AE AC AB BC


2
+ −


= = . Từ


đó suy ra AP AC AB BC BC AB AC. AB BC AC


2 AB AC BC 2


+ − + − + −


= =


+ − .


Như vậy từ các kết quả trên ta được AN AP= nên tam giác ANP cân tại A.


<b>Bài 40.</b> Cho tam giác nhọn ABC, lấy điểm X nằm trong tam giác và điểm Y, Z nằm
ngoài tam giác sao cho các tam giác XBC, YCA, ZBA đồng dạng với nhau. Đường
thẳng YZ cắt AB, AC lần lượt tại M và N. Gọi P là trung điểm của BC. Chứng minh


rằng YN ZM= khi và chỉ khi PAB=XAC.


<b>Lời giảiTrước hết ta phát biểu và chứng minh bổ đề. </b><i>Nếu P là trung điểm cạnh BC và </i>


<i>điểm S nằm trong tam giác ABC sao cho </i>PAB SAC= <i> thì ta ln có </i>


2
SAB


2
SAC


S AB


</div>
<span class='text_page_counter'>(52)</span><div class='page_container' data-page=52>

S


P C


B


A


S
V
U


H


K
N


M


P
Z


Y


X


C
B


A


<b>Chứng minh.</b> Do PAB SAC= nên BAS CAP= . Khi đó ta có


SAB
PAC


S AB.AS.sinBAS AB.AS


S = <sub>AC.AP.sin CAP</sub>= AC.APvà


PAB
SAC


S AB.AP.sin BAP AB.AP


S =<sub>AC.AS.sin SAC</sub>= AC.AS



Do AP là đường trung tuyến nên ta được S<sub>PAB</sub>=S<sub>PAC</sub>


Do đó ta được


2


SAB PAB SAB


2


PAC SAC SAC


S S AB.AS AB.AP S AB


. .


S S = AC.AP AC.AS S = AC .


<b>Trở lại bài toán</b>. Gọi S là giao điểm của AX và YZ. Gọi K và H lần lượt là hình chiếu
của Y và Z trên AB và AC. Ta chứng minh tứ giác AZXY là hình bình hành


Thật vậy, theo giả thiết ta có hai tam giác XBC và YCA đồng dạng với nhau nên ta


được BCX=ACY, suy ra BCA=XCY. Và ta cũng có XC BC


YC =AC nên hai tam giác ABC


và YXC đồng dạng với nhau. Từ đó ta được BAC CYX= và ABC=YXC. Chứng


minh hoàn toàn tương tự ta được hai tam giác ABC và ZBX đồng dạng với nhau nên


ta được BZX=BAC và BXZ=ACB. Từ đó ta được XYC XZB= .


Mà ta có AZB AYC= nên ta được AZX AYX= . Mặt khác ZAY=ZAB BAC YAC+ + .
Ta có biến đổi góc


(

)



0 0 0


0


ZXY 360 ZXB BXC CXY 360 ACB ABC 180 XBC XCB


180 ACB ABC XBC XCB BAC XBC XCB


= − − − = − − − − −


= − − + + = + +


Để ý là BAZ=BCX; YAC CBX= nên ta được ZAY ZXY= . Như vậy ta được ta được
tứ giác AZXY là hình bình hành.


<b>+ Điều kiện cần</b>. Nếu có YN ZM= , kết hợp với giả thiết ABZ∽ CAYta được


AYN
AZM


S


AC YK YK.AN AM AM YN AM AM



. . .


</div>
<span class='text_page_counter'>(53)</span><div class='page_container' data-page=53>

hình bình hành nên ta có SY SZ= , mà YN ZM= nên ta được SN SM= . Kết hợp với
hai tam giác ABC và ANM đồng dạng với nhau ta được hai tam giác ABP và ANS


đồng dạng với nhau. Do đó ta được PAB SAN XAC= = .


<b>+ Điều kiện đủ</b>. Nếu có PAB=XAC, khi đó với U và V lần lượt là hình chiếu của S
trên AB và AC, kết hợp với giả thiết ABZ và CAY đồng dạng với nhauthì theo bổ đề


trên ta được SAB


SAC


S
SU.AB


SV.AC=S hay ta được


2
SAB


2
SAC


S


SU AC AB AC AB ZH



. .


SV =S AB = AC AB =AC = YK . Do


đó ta được SU SV.


ZH =YK Do SU song song với ZH và SV song song với YK nên theo


định lí thales ta có SM SU SV SN
ZM= ZH= YK= YN.


Kết hợp với SY SZ= ta được ZM SM ZM SM SZ 1


YN SN YN SN SY


+


= = = =


+ .


Do đó ta được YN ZM= . Bài tốn được chứng minh.


<b>Bài 41.</b> Cho tam giác ABC khơng vng có các đường cao BE và CF cắt nhau tại H.
Gọi M, N, P, Q, S theo thứ tự là trung điểm của BF, CE, BE, CF, EF. Đường thẳng qua
M vng góc với BS và đường thẳng qua N vng góc với CS cắt nhau tại K. Đường
thẳng qua P vng góc với BS và đường thẳng qua Q vng góc với CS cắt nhau tại L.
Chứng minh rằng 2KL=AH.


<b>Lời giải</b>


Do M, N, P, Q, S theo thứ tự là trung điểm của
BF, CE, BE, CF, EF nên ta được


1 1 1 1


MB BE, MS BE, NS CF, CN CE


2 2 2 2


= = = =


Trong các tam giác vuông BCE và BCF có


2 2 2 2 2


BC =BE +CE =BF +CF


Gọi I là giao điểm của BS và MK, ta có Km


vng góc với BS và KN vng góc với CS nên


ta được <sub>KB</sub>2 =<sub>KI</sub>2+<sub>SI ; KS</sub>2 2=<sub>KI</sub>2+<sub>BI</sub>2


Suy ra <sub>KB</sub>2−<sub>KS</sub>2 =<sub>BI</sub>2−<sub>SI</sub>2


F


R
L



K
E
S


Q


P N


M


C


D
B


A


H


</div>
<span class='text_page_counter'>(54)</span><div class='page_container' data-page=54>

Chứng minh tương tự ta được <sub>MB</sub>2−<sub>MS</sub>2=<sub>BI</sub>2−<sub>SI</sub>2<sub> và</sub><sub>NC</sub>2−<sub>NS</sub>2=<sub>KC</sub>2−<sub>KS</sub>2<sub>. </sub>


Suy ra 2 2 2 2 1 2 1 2 1 2 1 2 2 2 2 2


KB KS MB MS BF BE CE CF NC NS KC KS


4 4 4 4


− = − = − = − = − = −


Từ đó ta được KC KB= . Gọi H là trực tâm của tam giác ABC, khi đó BF và CE là các


đường cao của tam giác HBC.Tương tự như trên ta chứng minh được LB LC= . Từ đó
ta suy ra LK là đường trung trực của đoạn thẳng BC nên LK vng góc với BC.


Dễ thấy LKN=BCS. Lại thấy QLK SCB 180+ = 0 suy ra QLK LKN 180+ = 0. Do đó ta


được QLsong song với KN. Lấy các điểm D và R sao cho tứ giác AHDF và QRKL là
các hình bình hành, suy ra R thuộc KN. Ta có HE⊥NC và HD//AF, CF//NS, AF⊥CF
suy ra HD⊥NS. Từ đó ta được EHD CNS= . Dễ thấy hai tam giác CHE và CAF đồng


dạng nên ta có HE HE EC 2NC NC


HD = AF = HC= 2NS = NS. Từ đó suy ra tam giác EHD và CNS


đồng dạng với nhau nên ta được HED NCS= . Ta có DE và RN cùng vng góc với


SN nên DE song song với KR, mà CF song song với RQ và FE song song với QN nên


ta được hai tam giác DEF và RNQ đồng dạng với nhau. Từ đó KL RQ QN 1


HA = DF = FE = 2


nên ta được 2KL=AH.


<b>Bài 42.</b>Cho tam giác ABC có D là chân đường cao hạ từ đỉnh C. Trên cạnh AB lấy
điểm các E, F sao cho ACE=BCF 90= 0. Lấy điểm X trên đoạn CD và điểm K trên
đoạn FX sao cho BK BC= . Lấy điểm L trên đoạn EX sao cho AL AC= . Gọi giao điểm
của AL và BK là M. Chứng minh rằng ML MK= .


<b>Lời giải</b>
<b>+ Trường hợp 1</b>. Xét tam giác ABC vuông tại C.


Khi đó ta được hai điểm A và F trùng nhau, hai
điểm B và E trùng nhau


Vẽ AI vng góc với BX, đường thẳng AI cắt
CD tại N. Tam giác NAB cao ND và BI là đường
cao nên X là trực tâm, do đó ta được à vng


góc với BNtại S.Tam giác ACB vng tại C có


CD là đường cao nên ta có AC2=AD.AB.


D
L
K


S
I


N


C
X


B
A


Theo bài ra ta có AL AC= nên ta được <sub>AL</sub>2 =<sub>AD.AB</sub><sub>. Ta </sub><sub>lại </sub><sub>có </sub>AD AI


</div>
<span class='text_page_counter'>(55)</span><div class='page_container' data-page=55>

AIL900



. Tam giác ALN vng tại L có LI là đường cao
nên ta được LN2=NI.NA. Tương tự ta có NK2=NS.NB. Mà ta có NI NS


NB= NA nên ta


được NI.NA NS.NB= . Do đó ta được NK NL= . Từ đó suy ra hai tam giác vng
KMN và LMN bằng nhau. Từ đó ta suy ra ML MK=


<b>+ Trường hợp 2.</b> Xét tam giác ABC có góc


0


ACB 90 . Kẻ đường thẳng qua A vng
góc với XE cắt CD tại N. Khi đó X là trực


tâm tam giác NAE. Suy ra à vuông góc


với. Từ đó ta được AEX=AND.


Ta có <sub>AL</sub>2=<sub>AC</sub>2=<sub>AD.AE</sub><sub> n</sub><sub>ên AL tiếp xúc </sub>


với đường tròn ngoại tiếp tam giác EDL. <sub>F</sub> D E


K


L
X


M
N



C


B
A


Suy ra ALD LED AEX= = . Từ đó ta được AND ALD= nên bốn điểm A, N, L, D cùng


nằm trên một đường trịn. Do đó 0


NLA=NDA 90= hay NL vng góc với AL.


Lại có hai tam giác XED và AND đồng dạng với nhau nên ta được XD ED


AD= ND. Hai


tam giác ACE và BCF cùng vng góc tại C nên ta được <sub>CD</sub>2 =<sub>AD.DE BD.FD</sub>= <sub> hay </sub>


AD FD


BD =ED. Do đó ta được


XD BD


FD =ND, nên hai tam giác BDN và XDF đồng dạng với
nhau, suy ra BND=DFX=BFK. Mặt khác ta có BK2=BC2=BD.BF nên BK tiếp xúc
với đường tròn ngoại tiếp tam giác DKF nên BKD BFK= . Từ đó ta được BND=BKD


nên bốn điểm B, D, K, N cùng nằm trên một đường trịn. Do đó ta được NK vng



góc với BK, áp dụng định lí Pitago ta được


(

) (

)



(

) (

) (

) (

)



2 2 2 2 2 2 2 2


2 2 2 2 2 2 2 2


MK ML NL NK NA AL NB BK


NA NB CB CA CA CB CA CB 0


− = − = − − −


= − + − = − + − =


Từ đó ta được MK2=ML2 hay MK ML= .


<b>+ Trường hợp 3.</b> Xét tam giác ABC có 0


ACB 90 . Chứng minh hoàn toàn tương tự ta
được ML MK= .


</div>
<span class='text_page_counter'>(56)</span><div class='page_container' data-page=56>

<b>Bài 43.</b>Cho tam giác ABC có M là trung điểm của AC và D là điểm trên cạnh BC sao


cho DB DM= . Biết rằng <sub>2BC</sub>2−<sub>AC</sub>2 =<sub>AB.AC</sub><sub>. Chứng minh </sub>


(

)




2


AC .AB
BD.CD


2 AB AC
=


+ .


<b>Lời giải</b>
<b>Lời giải 1.</b>Một ý tưởng rất tự nhiên đó là tìm cách
biểu diễn BD và CD theo các cạnh của tam giác
ABC và kết hợp với giả thiết đã cho. Quan sát hình
vẽ ta thấy có thể biểu diễn BD theo BM và


cosCBM.
I
D
M
C
B
A


Để ý BM là đường trung tuyến của tam giác ABC nên ta có thể biểu diễn BM và


cosCBM theo các cạnh của tam giác ABC. Từ đó kết hợp với 2BC2−AC2=AB.AC ta


có thể tính được BD.CD theo các cạnh của tam giác ABC.Để đơn giản trong qua trình



biến đổi ta đặt AB c; BC a; CA b.= = = Để tính được cosCBM ta cần tạo ra tam giác
vng bằng cách vẽ DI vng góc với BM với I thuộc BM. Khi đó ta được DI là đường
trung tuyến của tam giác cân BDM. Do đó ta được BD BI BM


cosCBM cosCBM


= = .


Mặt khác theo định lí cosin ta có


2
2 2


2 2 2 BM a b


BM BC CM <sub>4</sub>


2 cos CBM


BC.BM a.BM


+ −


+ −


= =


Từ đó ta được



2
2
2 2
a.BM
BD
b
BM a
4
=
+ −
.


Mặt khác ta có

(

)



2


2 2 2


2
2
2 2


b


a BM a


4
BD.CD BD a BD


b


BM a
4
 

 
 
= − =
 
+ −
 
 


Lại có

(

)



2


2 1 2 2 b


BM a c


2 4


= + − và kết hợp với giả thiết <sub>2a</sub>2−<sub>b</sub>2 =<sub>bc</sub> <sub>ta được</sub>


(

) (

2

)

2


2 2 2


2 2 2 2 2 2



2 2 b a c b 2 2c 2bc b bc b c 2c b


BM a a


4 2 4 16


+ +


 <sub>+</sub> <sub>−</sub>  <sub>=</sub> + − <sub>+</sub>  <sub>=</sub> + + +  <sub>=</sub>


     


     


Ta có

(

)

(

)



2


2 2 2 2 2


2 2 2


2 2 2 b a 2a 2c b b bc b a bc 2c bc


a .BM a .


4 4 2 4 16


+ − +



   + 


− = − =


   


</div>
<span class='text_page_counter'>(57)</span><div class='page_container' data-page=57>

Từ đó ta được

(

)



(

) (

)

(

) (

)



2


2 2 2


2 2


a bc 2c bc <sub>b c</sub> <sub>AC .AB</sub>


BD.CD


2 b c 2 AB AC


b c 2c b


+


= = =


+ +



+ +


<b>Lời giải 2.</b>Một ý tưởng khác cũng khá tự nhiên đó ta
tìm cách biến đổi giả thiết 2BC2−AC2 =AB.AC sao


cho có thể xác định được các yếu tố phụ một cánh
hợp lí. Chẳng hạn khi ta viết lại giả thiết của bài toán


thành 2

(

)



2BC =AC AB AC+ ta chú ý đến xác định
một điểm P trên AC sao cho PA=AB, khi đó thì ta
được CP AC AC= + .


P


M
D C
B


A


Từ đó ta được <sub>2BC</sub>2 =<sub>AC.CP 2CM.CP</sub>= <sub> hay </sub><sub>BC</sub>2=<sub>CP.CM</sub><sub>. </sub><sub>Hệ thức này làm ta tiên </sub>


tưởng đến tính chất tiếp tuyến và cát tuyến của đường tròn.Tức là BC sẽ là tiếp tuyến
của đường tròn ngoại tiếp tam giác BMP hay BD là tiếp tuyến của đường tròn ngoại
tiếp tam giác BMP. Mặt khác ta lại thấy BD DM= nên DM cũng là tiếp tuyến của
đường tròn ngoại tiếp tam giác BMP. Khi đó dễ dàng chứng minh được MDC=BAM


nên tứ giác ABDM nội tiếp, từ đó dẫn đến AD là phân giác của BAC. Đến đây ta có


thể biểu diễn BD, CD qua các cạnh của tam giác ABC.


Trên tia đối của tia AC lấy điểm P sao cho PA=AB, khi đó từ giả thiết ta được


2 2


2BC =AC.CP 2CM.CP= BC =CP.CM


Từ đó suy ra BC là tiếp tuyếncủa đường tròn ngoại tiếp tam giác PBM với tiếp điểm


là B. Ta lại có DB DM= với D thuộc BC nên DM cũng là tiếp tuyến của đường tròn
ngoại tiếp tam giác PBM.


Từ đó ABP=MPB=DBM=DMB. Lại có MDC=DBM BMD+ và BAM=APB ABP+
Suy ra MDC=BAM nên tứ giác MDBA nội tiếp, do đó DBM=DMB CAD= =BAD.


Do đó ta được AD là phân giác của góc BAC. Áp dụng tính chất đường phân giác


trong tam giác ta được BD AB BD AB BD AB.BC


CD= AC BC= AB AC+  =AB AC+


Từ đó suy ra DC AC.BC
AB AC
=


+ , kết hợp với giả thiết ta được


(

)

(

)




2 2


2


AB.AC.BC AC .AB


BD.CD


2 AB AC
AB AC


= =


</div>
<span class='text_page_counter'>(58)</span><div class='page_container' data-page=58>

<b>Bài 43. </b>Cho tam giác ABC và các điểm D, E, F lần lượt là các tiếp điểm của đường trịn
bàng tiếp góc A, B, C với các cạnh BC, CA, AB của tam giác ABC. Chứng minh rằng
nếu tâm đường tròn ngoại tiếp tam giác DEF nằm trên đường tròn ngoại tiếp tam


giác ABC thì tam giác ABC vng.


<b>Lời giải</b>


<b>Lời giải 1. </b>Trước hết ta phát biểu và chứng minh bài toán phụ.


<i> Cho D là một điểm nằm trên cạnhAC của tam giác ABC khi đó ta ln có </i>


2 2 2


AB .CD BC .AD BD AC AC.CD.AD+ − =


<b>Chứng minh.</b>Gọi H là chân đường cao hạ từ B xuống AC, giả sử D thuộc đoạn AH.



Trong tam giác ABD có


(

)

2


2 2 2 2 2 2 2


BC =HC +HB = BD DH− +BD −AD =BD +CD −2DC.DH


Tương tự trong tam giác ABD ta có <sub>AB</sub>2=<sub>BD</sub>2+<sub>AD</sub>2+<sub>2AD.DH</sub>


Từ đó ta được AB .CD BC .AD2 + 2 =BD AD DC2

(

+

)

+DC AD AD DC2 + 2
Suy ra <sub>AB .CD BC .AD BD AC AC.CD.AD</sub>2 + 2 − 2 =


<b>Trở lại bài toán.</b>Đặt AB c; BC a; CA b; p a b c
2
+ +


= = = =


Gọi S là tâm đường tròn ngoại tiếp tam giác
DEF. Theo giả thiết thì S nằm trên đường
tròn ngoại tiếp tam giác ABC. Khơng mất
tính tổng qt ta giả sử S nằm trên cung BC
có chứa điểm A. Khi đó ta có SE SF= . Theo


tính chất của đường tròn bàng tiếp tam


giác ta có CE BF p a= = −



Vì ABS=ACS nên ta được SEC= SFB,
suy ra SB SC= hay S nằm chính giữa cung


BC. Áp dụng đổ đề trên cho tam giác ABC
với điểm D trên cạnh BC ta có


(

)



2 2 2


SC .DB AB .DC BC SD+ = +DB.DC


S


O
N
M


K


J


I


F E


B <sub>D</sub> C


A



</div>
<span class='text_page_counter'>(59)</span><div class='page_container' data-page=59>

Để ý là DB DC BC+ = nên ta được <sub>SB</sub>2−<sub>SD</sub>2=<sub>DB.DC</sub><sub>. </sub>


Mà ta có BD p b; CD p c= − = − , do đó ta được

(

)(

)

(

)



2
2


2 2 a b c


SB SD p b p c


4
− −


− = − − =


Gọi N và M lần lượt là hình chiếu vng góc của S trên AC và AB


Khi đó ta được SBM= SCN suy ra AM AN= nên ta được BM CN b c
2
+
= =


Mặt khác ta có


2 2


2 2 2 2 b c b c b c a


SB SF MB MC



2 2 2


 +   + + − 


− = − =<sub></sub> <sub></sub> −<sub></sub> − <sub></sub>


   


Do SD SF= nên ta có <sub>SB</sub>2−<sub>SD</sub>2 =<sub>SB</sub>2−<sub>SF</sub>2<sub>. Do vậy ta được </sub>


(

)

2 <sub>2</sub> <sub>2</sub>


2


2 2 2


a b c <sub>b c</sub> <sub>b c</sub> <sub>b c a</sub>


a b c


4 2 2 2


− − <sub> + </sub> <sub> +</sub> <sub>+ − </sub>


=<sub></sub> <sub></sub> −<sub></sub> − <sub></sub>  = +


   


Do vậy tam giác ABC vng tại A. Từ đó ta có điều phải chứng minh.


<b>Lời giải 2.</b>Trước hết ta phát biểu và chứng minh các bổ đề sau


<b>Bổ đề 1.</b><i>Cho tam giác ABC nội tiếp đường trịn </i>

( )

O
<i>có các đường cao AD, BE, CF. Khi đó, đường thẳng </i>
<i>qua A vng góc với EF, đường thẳng qua B vng </i>
<i>góc với FD, qua C vng góc với DE đồng quy tại O.</i>


<b>Chứng minh.</b> Gọi H là trực tâm tam ABC và I, J


theo thức tự là giao của BE, CF với

( )

O . Dễ thấy
E, F theo thứ tự là trung điểm của HI, HJ nên EF


song song với IJ.


J


H


I
O


F


E
C
D


B


A



Lại có ABI=ACJ suy ra AI=AJ nên AO vng góc với EF . Tương tự OB vng góc
với DF và OC vng góc với DE. Từ đó ta có điều phải chứng minh.


<b>Bổ đề 2.</b> <i>Cho tam giác ABC nội tiếp đường tròn </i>

( )

O <i>và M là trung điểm BC. Khi đó ta có</i>
BC


OM
2


= <i>khi và chỉ khi </i> 0


BAC=45 <i>. </i>


<b>Bổ đề 3.</b> <i>Cho tam giác ABC có J là tâm đường tròn bàng tiếp góc A. Khi đó ta ln có </i>


0 BAC


</div>
<span class='text_page_counter'>(60)</span><div class='page_container' data-page=60>

<b>Trở lại bài tốn.</b>Khơng mất tính tổng qt


ta giả sử BAC là góc lớn nhất trong tam


giác ABC.


+ Trường hợp AB AC= bài toán hiển nhiên
đúng.


+ Xét trường hợp<sub> AB AC</sub> .


Đặt BAC=A. Gọi S là điểm chính giữa



cung BAC của đường tròn ngoại tiếp ta


giác ABC. Dễ thấy SBF= SCE nên ta có
SE SF= . Do đó suy ra BSC ESF= =A. Gọi
I, J, K theo thứ tự là tâm đường trịn bàng
tiếp các góc A, B, C của tam giác ABC.


A


D C


B


E
F


I


J


K


M
O


S


Dễ thấy IA, JB, KC là các đường cao của tam giác MNP và S là trung điểm của JK.
Theo bài toán phụ 1 ta có ID, JE, KF đồng quy tại điểm M là tâm đường tròn ngoại


tiếp tam giác IJK.Chú ý rằng các tứ giác BFDM, CEDM, AEMF nội tiếp đường tròn


nên BDF=BMF và CDE CME= . Lại có EMF 180= 0−A. Khi đó ta có biến đổi góc


(

) (

0

)

0 0


BMC EDF+ = BMF FME EMC+ + + 180 −BDF CDE− =EMF 180+ =360 −A. Từ
đó ta được BMC EDF1 2 1800 A


2


 


+ = <sub></sub> − <sub></sub>


  .


Tâm đường tròn ngoại tiếp tam giác DEF nằm trên đường tròn ngoại tiếp tam giác
ABC khi và chỉ khi S là tâm đường tròn ngoại tiếp tam giác DEF, điều này xẩy ra khi
và chỉ khi


(

0

)

0 A 0 A

(

0

)



ESF 2 180 EDF EDF 180 BMC 180 BSC A 2 180 BMC


2 2


= −  = −  = −  = = −


Hay S là tâm ngoại tiếp của tam giác BMC. Do vậy ta được



0 0 0 0


JK A


SM SE SC SM KIJ 45 90 45 A 90


2 2


= =  =  =  − =  =


Do đó ta có điều phải chứng minh.


</div>
<span class='text_page_counter'>(61)</span><div class='page_container' data-page=61>

khi và chỉ khi


AB AC 3BC+ =


<b>Lời giải</b>


Gọi D là tiếp điểm của đường tròn

( )

I với BC.
Gọi

( )

O là đường tròn ngoại tiếp tam giác
ABC. Gọi J là giao điểm khác của AI với đường


tròn

( )

O . Trước hết ta chứng minh I là tâm
đường tròn nội tiếp tam giác DKL.


Thật vậy, do I là tâm đường tròn nội tiếp tam


giác ABC nên ta có <sub>BIC 90</sub>0 1<sub>BAC</sub>



2


= + .


T


X
Y
L


K
Q


P


J
I


D C


B
A


Mà AP và AQ là hai tiếp tuyến của đường tròn

( )

I nên tam giác APQ cân tại A. Từ
đó ta có <sub>BAP 90</sub>0 1<sub>BAC</sub>


2


= + . Do đó suy ra BQL BIC= nên tứ giác BQLI nội tiếp, nên
ta được LQI LBI= . Mà ta có <sub>LQB LIB 180</sub><sub>+</sub> <sub>=</sub> 0<sub></sub><sub>LQI LIB 90</sub><sub>+</sub> <sub>=</sub> 0 <sub>nên ta được </sub>



0


LBI LIB 90+ = . Từ đó suy ra BLI=900 hay BL vng góc với LC. Chứng minh tương
tự ta cũng được BK vng góc với KC.


Gọi T là giao điểm của BL và CK, khi đó I là trực tâm của tam giác TBC. Do đó ta được
D, I, T thẳng hàng.Dễ thấy các tứ giác BLKC, MLID và CKID nội tiếp đường tròn nên
ta có KBL KCL; LBI LDI; KCI KDI= = = . Từ đó suy ra LDI KDI= nên DI là phân giác


của góc LDK. Hồn tồn tương tự ta cũng được LI là phân giác của góc KLD. Do đó


I là tâm đường trịn nội tiếp tam giác DKL.Đường tròn ngoại tiếp tam giác IKL tiếp
xúc với đường tròn

( )

I khi và chỉ khi R<sub>( )</sub><sub>IKL</sub> r


2


= với r là bán kính đường trịn

( )

I và


( )IKL


R là bán kính đường trịn ngoại tiếp tam giác ILK. Kẻ IX vng góc với LD tại X,
IY vng góc với PQ tại Y. Dễ dàng chứng minh được hai tam giác ILK và IBC đồng
dạng với nhau nên ta được ( )


( )


ILK
IBC



R <sub>IY</sub> <sub>IX</sub> <sub>A</sub>


sin IDX sin


R =ID =ID = = 2


</div>
<span class='text_page_counter'>(62)</span><div class='page_container' data-page=62>

Hay R( )ILK IP r


IJ =IA=AI . Từ đó suy ra ( )IKL
r
R


2


= khi và chỉ khi IA=2IJJA=3JB 3JC=


Do tứ giác ABJC nội tiếp dường trịn (O) nên áp dụng định lí Ptoleme ta được


(

)



AI.BC=AB.JC AC.JB+ = AB AC .JB+


Kết hợp với JA=3JB 3JC= ta được AB AC 3BC+ = . Vậy bài toán được chứng minh.


<b>Bài 45.</b> Cho đường trịn

(

O; R

)

có đường kính AB và một điểm M bất kì nằm trong


( )

O nhưng khơng nằm trên đường kính AB. Gọ N là giao điểm của đường phân giác
trong của góc AMB với đường trịn

( )

O . Đường phân giác ngồi của góc AMB cắt
đường thẳng NA, NB lần lượt tại P và Q. Đường thẳng MA cắt đường tròn đường
kính NQ tại R, đường thẳng MB cắt đường trịn đường kính NP tại S và R, S khác M.

Chứng minh rằng đường trung tuyến ứng với đỉnh N của tam giác NRS luôn đi qua
một điểm có định khi M di động phía trong đường trịn.


<b>Lời giải</b>


O
I


S


R


Q


P


N
M


D
C


B
A


Qua R kẻ đường thẳng song song với PQ cắt AN tại C, qua S kẻ đường thẳng song
song với PQ cắt BN tại D. Gọi I là trung điểm của CD. Ta sẽ chứng minh CD song
song với AB.


Thật vậy, do N nằm trên đường tròn đường kính AB nên ta có 0



ANB 90= suy ra AN


vng góc với BN, do đó BN là tiếp tuyến của đường trịn đường kính PN. Từ đó ta


có tam giác BMN đồng dạng với BNS. Vì PQ là đường phân giác ngoài của tam giác


AMN nên ta có SMP AMP QMR BMQ= = = . Mặt khác SMP SNP= và QMR QNR= .


</div>
<span class='text_page_counter'>(63)</span><div class='page_container' data-page=63>

Xét hai tam giác BNS và RNC có CNR=SNB và RCN MPN= =NSM=NSB nên hai


tam giác đó đồng dạng với nhau. Từ đó suy ra BNS∽ RNC∽ BMN. Lập luận
chứng tương tự ta cũng có DSN∽ RAN∽ NAM.


Ta thấy BNS RNC NB NS NB.NC NR.NS


NR NC


 ∽   =  =


NS ND


DSN RAN NA.ND NR.NS


NA NR


 ∽   =  =


Từ đó ta được NB.NC NA.ND= hay NA NC



NB =ND, theo định lí Thales đao ta được AB


song song với CD.


Do đó trung điểm của AB, trung điểm của CD và N thẳng hàng. Tức là N, O, I thẳng


hàng. Lại có BMN RNC MN BN RC NB.NC


NC RC MN


 ∽   =  =


DSN NAM DN DS SD NA.ND


MN NA MN


 ∽   =  =


Kết hợp các điều trên ta được RC SD= . Mà ta có RC song song với SD nên tứ giác


RCSD là hình bình hình. Do đó hai đường chéo CD và SR cắt nhau tại trung điểm của
mỗi đường, suy ra I là trung điểm của CD cũng là trung điểm của SR. Khi đó NI là
đường trung tuyến của tam giác NSR. Do đó ta được đường trung tuyến NI luôn đi
qua điểm O cố định. Vậy đường trung tuyến xuất phát từ N của tam giác NRS luôn đi
qua điểm O cố định khi điểm M di động trong đường tròn

( )

O .


<b>Bài 46.</b> Cho tam giác nhọn ABC cố định và không cân nội tiếp đường tròn

( )

O , đường


phân giác AD. Lấy điểm P di động trên đoạn thẳng AD và điểm Q trên đoạn thẳng



AD sao cho PBC QBA= . Gọi R là hình chiếu của Q trên BC. Đường thẳng d đi qua R
và vng góc với OP. Chứng minh rằng đường thẳng d luôn đi qua điểm cố định khi
P di chuyển.


</div>
<span class='text_page_counter'>(64)</span><div class='page_container' data-page=64>

<b>Lời giải 1.</b>Gọi giao điểm thứ của AD với đường


tròn

( )

O là E nên E là điểm chính cung BC. Vẽ
đường kính EF của

( )

O . Gọi M là trung điểm
của BC. Khi đó ba điểm E, O, F thẳng hàng. Lấy
điểm N đối xứng với M qua AD và H là trung
điểm của MN. Khi đó H thuộc AD. Ta sẽ chứng
minh đường thẳng d đi qua điểm N cố định. Dễ
thấy RMN OEP= .


H
F


D
O


N
K


E
R M


P
Q


C


B


A


Do QR song song với MN nên ta có QD DE QD DE QE


DR DM DR DM RM


+


= = =


+ . Dễ thấy hai tam
giác vuông HDM và MDE đồng dạng với nhai nên DE ME 2ME


DM=MH= MN . Do đó ta được


QE 2ME


RM= MN hay


MN 2ME


MR = QE . Dễ thấy rằng CBE QAC QAB= = và theo giả thiết ta có
PBE PBC CBE QBA QAB BQE= + = + = . Trong tam giác FBE vng tại B có BM là
đường cao nên <sub>BE</sub>2=<sub>EM.EF</sub><sub>. Xét hai tam giác EBP và EQB có PBE BQE</sub>= <sub> và BEQ</sub>


chung nên hai tam giác EBP và EQB đồng dạng với nhau, suy ra EP EB


EB =EQ. Từ đó ta



được <sub>EP.EQ EB</sub>= 2 =<sub>EM.EF 2EM.EO</sub>= <sub> nên </sub>2EM EP


EQ =EO. Từ đó ta được


MN EP


MR =EO . Xét
hai tam giác OPE và MNR có RMN OEP= và MN EP


MR =EO nên hai tam giác đó đồng


dạng với nhau, suy ra MNR=EPO. Gọi RN cắt OP tại K, dễ thấy tứ giác PHNK nội


tiếp nên ta được 0


</div>
<span class='text_page_counter'>(65)</span><div class='page_container' data-page=65>

<b>Lời giải 2.</b>Dựng đường có AH của tam giác ABC.
Qua H dựng đường thẳng vng góc với OD cắt
đường thẳng qua D vng góc với OA tại X, từ đó ta
được X cô định. Ta sẽ chứng minh đường thẳng d đi
qua điểm X cố định.


Thật vậy, gọi giao điểm của OD với AH là M, giao
điểm của OP với AH là L. Đường tròn

( )

O cắt đường
thẳng AD tại điểm thứ hai là F.


E
L


X


R


M
H


O
P
Q


D


C
B


A


Áp dụng định lí Menelaus cho tam giác ADM với ba điểmO, P, L thẳng hàng ta được


LA PD OM


. . 1


LM PA OD = , từ đó suy ra


LM PD OM


.
LA =PA OD .
Ta có AQB



PBD


S <sub>QA</sub> <sub>AB.BQ</sub>


S = PQ = BP.BD và


PAB
QBD


S PA BA.BP


S =QD = BQ.BD.Chú ý là OF song song với AM,


kết hợp các tỉ số trên ta được


2
2


QA BA .PD FA PD OM PD LM


. .


QD = BD .PA=FD PA= OD PA = LA
Mà do QR song song với AM nên ta được RH QA LM


RD = QD = LA .


Dễ thấy XDH∽ OAM nên suy ra XDR ∽ OAL dẫn đến XRD OLA= .Gọi giao
điểm của XR và OP là E, khi đó tứ giác LERH nội tiếp được nên ta suy ra 0



LER=90


Do vậy đường thẳng qua R và vng góc với OP đi qua điểm X cố định. Từ đó ta có
điều phải chứng minh.


<b>Bài 47.</b>Cho tam giác ABC cố định. Các điểm E và F di động trên các đoạn CA, AB sao


cho BF CE= . Giao điểm của BE và CF là D. Gọi H, K là trực tâm các tam giác DEF và
DBC. Chứng minh rằng đường thẳng HK luôn đi qua điểm cố định khi E và F di
động.


</div>
<span class='text_page_counter'>(66)</span><div class='page_container' data-page=66>

Gọi AG là phân giác của góc BAC với
G thuộc BC. Đường tròn ngoại tiếp tam
giác AGB và AGC cắt lần lượt AC và
AB tại M, N khác A.


Dễ thấy tứ giác ANGC nội tiếp đường


trịn nên có BN.BA BG.BD= . Tứ giác
AMGB nội tếp nên đường trịn nên ta


có CM.CA CG.CB= . Do AG là phân


giác của tam giác ABC nên AB GB


AC= GC.


G
D



K
H


O


T
P


F


E
N


M
A


C
B


S


Từ đó ta được BN BN.BA CA. BG.BC CA. BG CA. AB AC. 1
CM=CM.CA AB = CG.CB AB =CG AB = AC AB =


Từ đó ta được BN CM= , mà theo giả thiết ta có BF CE= nên ta được NF ME.=


Từ đó ta có CNF CNF ABC
BME ABC BME


S S S NF AC AC



. .


S =S S =AB ME = AB


Lại có BM BM AD. BC AB. AB


CN = AD CN=AC BC = AC. Từ đó ta được


CNF
BME


S CN


S = BM


Gọi

( )

O là đường tròn ngoại tiếp tam giác và P là điểm chính giữa cung BC có chứa A
của đường tròn

( )

O . Ta sẽ chứng minh HK đi qua P.


Thật vậy, gọi EH, FH lần lượt cắt PB, PC tại S, T. Do SE vuông góc với FC nên ta có


0 0 0 0


0 0


1


ESB 360 SBC FCB 90 270 90 BAC FCB


2



180 FCB NCB 180 NCF


 


= − − − = −<sub></sub> − <sub></sub>−


 


= − − = −




Tương tự ta được FTC 180= 0 −MCE. Dễ thấy SBE
CNF


S SB.SE


S =CN.CF và


TCF
BME


S TC.TF


S = BM.BE


Mặt khác chú ý là SEB TFC= nên ta được


SBE SBE CNF BME


TCF CNF BME TCF


S S S S


ES.EB SB.SE CN BM.BE SB ES EB


. . . .


FT.FC=S =S S S =CN.CF BM TC.TF =TC FT FC


Từ đó ta suy ra SB 1


CT = hay SB TC= nên ta được STsong song với BC. Lại thấy do H


</div>
<span class='text_page_counter'>(67)</span><div class='page_container' data-page=67>

CT đồng quy tại P.Vậy HK đi qua điểm cố định P. Vậy ta có điều phải chứng minh.


<b>Bài 48. </b>Cho điểm A nằm ngồi đường trịn

( )

O . Từ điểm A vẽ hai tiếp tuyến AB và


AC (B, C là hai tiếp điểm) và cát tuyến ADE thay đổi với đường tròn

( )

O sao cho tia


AD nằm giữa hai tia AO và AB. Đường thẳng qua D và song song với BE cắt BC, AB
lần lượt tại và Q. Gọi K đối xứng với B qua E. Chứng minh rằng đường thẳng PK luôn
đi qua một điểm cố định.


<b>Lời giải</b>


I


x



K
Q


P
H O


E
D


C
B


A


Gọi H và I lần lượt là giao điểm của BC với OA và DE. Ta có AB và AC là hai tiếp
tuyến với đường tròn

(

O; R

)

nên AB AC= và AO là phân giác của góc BAC. Do đó
AO là đường cao của tam giác ABC. Hai tam giác ABD và AEB có góc BAD chung và


ABD AEB= nên hai tam giác đó đồng dạng với nhau, do đó ta được <sub>AB</sub>2 =<sub>AD.AE</sub><sub>. </sub>


Trong tam giác ABO vng có BH là đường cao nên AB2=AH.AO. Từ đó ta được


AD.AE AH.AO= hay AH AD


AE =AO. Từ đó suy ra hai tam giác HAD và AEO đồng dạng


với nhau, do đó ta có AHD=AEO. Đến đây thì ta được tứ giác OEDH nội tiếp đường


trịn nên OHE=ODE. Tam giác ODE có OD OE= nên cân tại O, suy ra ODE=OED.



Từ đó ta được OHE=AHD. Để ý rằng OHE EHI+ =AHD IHD 90+ = 0 nên ta được


EHI IHD= , do đó HI là tia phân giác của góc HED. Gọi Hx là tia đối của tia HE, khi
đó ta có xHA=AHD OHE= . Do đó HA là đường phân giác của HED, từ đó ta suy ra


được ID AD


ED = AE . Trong tam giác ABE có DQ song song với BE nên theo định lí Thales


vng góc với CF suy ra SH song song với BK. Tương tự ta cũng có CKsong song với


</div>
<span class='text_page_counter'>(68)</span><div class='page_container' data-page=68>

ta có DQ AD


BE = AE. Trong tam giác IBE có BE song song với PD nên theo định lí Thales
ta có DP ID


BE = IE . Từ đó ta được


DQ DP


BE = BE nên DQ DP= . Trong tam giác ABE có DQ


song song với BE nên theo định lí Talets ta có AQ QD


AB = BE . Do đó


AQ 2DQ PQ


AB = 2BE = BK.
Hai tam giác APQ và AKB có AQ PQ



AB =BK và AQP ABK= nên đồng dạng với nhau.. Từ


đó ta được QAP BAK= nên hai tia AP và AK trùng nhau. Điều này có nghĩa là
đường thẳng PK luôn đi qua điểm cố định A.


<b>Bài 49.</b> Cho đường trịn

(

O; R

)

đường kính AB cố định. Lấy C là một điểm di động
trên đường tròn

( )

O sao cho BC AC . Vẽ CH vng góc với AB tại H, HM vng
góc với AC tại M. Đường thẳng qua O song song với BC cắt tia tiếp tuyến tại A với
đường trịn

( )

O ở E. Đường thẳng qua O vng góc với AB cắt BC tại D. Gọi I là giao
điểm của BE với AC. Các tiếp tuyến tại O và D của đường tròn ngoại tiếp tam giác
OBD cắt nhau tại G. Đường thẳng qua B vuông góc với IH cắt CH tại N. Đường trịn


ngoại tiếp tam giác AMN cắt CH tại điểm K khác N. Chứng minh rằng đường thẳng
GK luôn đi qua một điểm cố định.


<b>Lời giải</b>
Gọi F là giao điểm của FO và AC, S là giao điểm
của AE và BC. Khi đó dễ dàng thấy được E và F


lần lượt là trung điểm của AS và AC. Hai tam
giác AHI và CNB có HAI=NCB và AHI=CNB
nên hai tam giác đó đồng dạng với nhau. Do đó
ta được IA AH


CB= CN . Tứ giác AMKN nội tiếp


đường tròn nên CK.CN CM.CA= .


Lại có <sub>CH</sub>2 =<sub>CM.CA</sub><sub> và </sub><sub>CH AH.BH </sub>2



E
S


H
F


I
M


O
K G


N
D
C


B
A


Do đó ta được CK.CN AH.BH= hay AH CK


CN = BH. Do đó


AI CK


CB= BH hay


BH CK



BC = AI . Lại
có CB2=BH.AB nên BH BC


BC =AB do đó ta được


CK BC


</div>
<span class='text_page_counter'>(69)</span><div class='page_container' data-page=69>

ta được hai tam giác CBK và ABI đồng dạng với nhau, từ đó suy ra CBK=ABI nên
suy ra CBK=ABE. Ta có hai tam giác BCA và BAS đồng dạng với nhau, mà E và F


lần lượt là trung điểm của AS và AC nên suy ra hai tam giác BCF và BAE đồng dạng
với nhau. Do đó suy ra CBF=ABE, kết hợp với CBK=ABE ta được CBK=CBF nên
hai tia BK, BF trùng nhau. Do đó ba điểm B, K, F thẳng hàng. Ta có hai tam giác ODG


và HCF đồng dạng với nhau nên ta được DG OD


CF =HC. Hai tam giác BOD và BHC đồng


dạng với nhaunên ta được OD BD


HC= BC . Do đó ta được


DG BD


DF = BC mà lại có GDB FCB=
nên hai tam giác DGB và CFB đồng dạng với nhau. Từ đó ta suy ra DBG CBF= nên ta
có CBG CBF= , do đó hai tia BG và BF trùng nhau, suy ra ba điểm B, G, F thẳng hàng.
Do đó ta đượcB, G, K, F thẳng hàng nên ba điểm B, G, K thẳng hàng. Mà B cố định
nên đường thẳng GK luôn đi qua điểm B cố định.



<b>Bài 50.</b> Cho tam giác ABC nội tiếp đường trịn

( )

O có H là trực tâm. Đường trung
tuyến AM cắt đường tròn

( )

O tại N khác A. Đường thẳng AH cắt đường tròn

( )

O tại
điểm K khác A. Các đường thẳng KN, BC và đường thẳng qua H vng góc với AN
cắt nhau theo từng đôi một tạo thành tam giác XYZ. Chứng minh rằng đường tròn
ngoại tiếp tam giác XYZ tiếp xúc với đường tròn

( )

O .


<b>Lời giải</b>


Giả sử các điểm X, Y, Z có vị trí như hình vẽ. Gọi T là giao điểm thứ hai của đường
trịn đường kính AH với đường tròn

( )

O . Gọi P là hình chiếu của H trên AM. Vẽ
đường kính AQ của đường trịn

( )

O . Do H là trực tam của tam giác nên ta có CH
vng góc với AB nên suy ra CH song song với BQ, tương tự thì BH song song với
CQ, suy ra tứ giác BHCQ là hình bình hành. Mà M là trung điểm của BC nên ta có ba


</div>
<span class='text_page_counter'>(70)</span><div class='page_container' data-page=70>

Q
L


P
T


K


N
H


M


Z Y


X



O


C
B


A


Xét tam giác AMH có hai đường cao cắt nhau tại Y nên Y là trực tâm của tam giác
AHM, lại có AT vng góc với MH nên suy ra AT là đường có cịn lại của tam giác
hay ba điểm A, T, Y thẳng hàng. Đến đây ta có biến đổi góc


0 0 0


HXK 90= −PNK 90= −ANK 90= −AQK KAQ QTK HTK= = =


Suy ra tứ giác XTHK nội tiếp đường trịn.Do đó ta được TXK THA TYB= = nên tứ
giác XZTY nội tiếp đường tròn. Như vậy T là giao điểm của đường tròn

( )

O với
đường tròn ngoại tiếp tam giác XYZ. Từ T kẻ tiếp tuyến TL với đường tròn

( )

O , khi


đó để ý đến các tứ giác nội tiếp đường trịn ta có biến đổi góc


XTL XTK LTK XHK TAK TYH TZX= − = − = =


</div>
<span class='text_page_counter'>(71)</span><div class='page_container' data-page=71>

<b>Bài 51.</b>Cho ta giác đều ABC nội tiếp đường tròn

(

O; R

)

. Quay tam giác ABC một góc


0


90 quanh O ta thu được tam giác A B C<sub>1</sub> <sub>1</sub> <sub>1</sub>. Tính diện tích phần chung của hai tam



giác ABC và A B C<sub>1</sub> <sub>1</sub> <sub>1</sub>


<b>Lời giải</b>


Giả sử đường trịn

( )

O có chiều A→ → →B C A là


chiều ngược với chiều của kim đồng hồ. Giả sử quay
tam giác ABC một góc <sub>90</sub>0<sub>quanh điểm O theo chiều </sub>


thuận kim đồng hồ thì thu được tam giác A B C<sub>1</sub> <sub>1</sub> <sub>1</sub>.


Khi đó các điểm A ; B ; C<sub>1</sub> <sub>1</sub> <sub>1</sub> lần lượt thuộc các cung


AC; AB; BC. Do đó cạnh A B<sub>1</sub> <sub>1</sub> cắt AB, AC lần lượt
tại M, N.


T
K


E


C<sub>1</sub>
B<sub>1</sub>


A<sub>1</sub>
O


A'
Q



P
N
M


C
B


A


Tương tự thì A C<sub>1</sub> <sub>1</sub> cắt AC, BC lần lượt tại P, Q và B C<sub>1</sub> <sub>1</sub> cắt BC, BA lần lượt tại T, K.
Như vậy phần chung của hai tam giác ABC và A B C<sub>1</sub> <sub>1</sub> <sub>1</sub> là đa giác MNPQTK. Gọi diện
tích của lục giác MNPQTK là S, khi đó ta được S S= <sub>ABC</sub>−S<sub>AMN</sub>−S<sub>BKT</sub>−S<sub>CPQ</sub>. Mặt khác


ta có


2 2


ABC


BC 3 3 3R


S


4 4


= = .


Khi quay một góc 90 0 quanh điểm O thì ta được OA<sub>1</sub>⊥OA; OB<sub>1</sub>⊥OB; OC<sub>1</sub> ⊥OC. Từ
đó ta suy ra được OA1song song với BC, OB1 song song với CA, OC1 song song với



AB. Gọi giao điểm của OA<sub>1</sub> với AC là E. Do các cung AA ; BB ; CC<sub>1</sub> <sub>1</sub> <sub>1</sub> có sô đo bằng


0


90 nên ta được AMN 90= 0, mà ta lại có MAN=600 nên ta suy ra được AN 2AM= .


Dễ thấy NEA<sub>1</sub> cân tại E. Lại do OE song song với BD và AO 2AA'
3


= nên ta được


2 2R


AE AC


3 3


= = và OE BC R


3 3


= = . Từ đó ta được EN EA<sub>1</sub> OA<sub>1</sub> OE R R


3


= = − = − .


Đến đây suy ra AN AE EN 2 R R R

(

3 1 R

)



3 3



 


= − = −<sub></sub> − <sub></sub>= −


  . Do vậy ta được


(

)

2

(

)

2


2
AMN


3 4 2 3 R 2 3 3 R


MA.MN 1 AN AN 3 3AN


S . .


2 2 2 2 8 8 4


− −


</div>
<span class='text_page_counter'>(72)</span><div class='page_container' data-page=72>

Hoàn toàn tương tự ta cũng được

(

)



2
BTK CPQ


2 3 3 R


S S



4


= = .


Do vậy ta được

(

)



2


9 3 3 R
S


4


= .


<b>Bài 52.</b>Cho tứ giác lồi nội tiếp ABCD có đường trịn nội tiếp tâm I. Gọi O là giao điểm
của AC và BD. Chứng minh rằng


2
2


AO AI


CO = CI .


<b>Lời giải</b>



Gọi r là bán kính đường trịn tâm I. Giả sử rằng


M,N,P,Q lần lượt là tiếp điểm của đường tròn tâm I
với các cạnh AB, BC,CD, DA. Đặt


AM AQ x; BM BN y; CN CP z; DP DQ t= = = = = = = =


Ta có AI r ,CI r


A C
sin sin
2 2
= = .
O
Q
P
N
M
I
C
D
B
A


Do đó ta suy ra được


2
2
2
2


C
sin


AI <sub>2</sub> 1 cos A


A 1 cos C


CI


sin
2




= =


− . Sử dụng định lý cosin cho tam giác


BAD ta có

(

) (

)



(

)(

)



2 2 <sub>2</sub>


2 2 2 <sub>x y</sub> <sub>x t</sub> <sub>BD</sub>


AB AD BD


cos A



2AB.AD 2 x y x t


+ + + −


+ −


= =


+ +


Suy ra

(

) (

<sub>(</sub>

<sub>)(</sub>

)

<sub>)</sub>

<sub>(</sub>

(

<sub>)(</sub>

)

<sub>)</sub>



2 2 <sub>2</sub> <sub>2</sub> 2


x y x t BD BD y t


1 cos A 1


2 x y x t 2 x y x t


+ + + − − −


− = − =


+ + + + .


Tương tự ta cũng chứng minh được


(

) (

)




(

)(

)

(

(

)(

)

)



2 2 <sub>2</sub> <sub>2</sub> 2


z y z t BD BD y t


1 cos C 1


2 z y z t 2 z y z t


+ + + − − −


− = − =


+ + + + .


Do đó

(

)(

)



(

)(

)


2
CBD
ABD ABD
2
CBD


x y x t 2S 2S S


AI AB.AD AO


:



CB.CD S CO


z y z t


CI <sub>sin BAD sin BCD</sub>


+ +


= = = = =


+ +


</div>
<span class='text_page_counter'>(73)</span><div class='page_container' data-page=73>

minh rằng tam giác ABC vng tại B khi và chỉ khi diện tích của hai tam giác BDE và
HDE bằng nhau.


<b>Lời giải</b>


Do AC 2AB= nên ta được AB AC , do đó ta


được ACB ABC nên ta được ACB 90 0.


Từ A900, ACB 90 0 và AB AC suy ra điểm
K nằm trong tam giác.


Do DB AB 1


DC =AC= 2 nên ta được


BD 1 CD 2



,


BD=3 BC = 3.


H
K
E
D C
B
A


<b>+ Điều kiện cần.</b>Giả sử tam giác ABC vng tại B. Khi đó dễ thấy 0


BAC 60= . Từ đó
ta được AH 1AB 1AC


2 2


= = và AH 1CH
2


= . Áp dụng định lí Menelaus cho tam giác


ABC với ba đường thẳng AD, BH, CE đồng quy ta được EA DB HC. . 1


EB DC HA= nên ta được


AE 2



EB = 3, do đó


BE 3 EA 2


;


BA =5 AB= 5.


Khi đó ta được AEH
ABC


S AE.AH 1


S = AB.BC =10 và


CDH
ABC


S CD.CH 1


S = CB.CA =2


Cộng theo vế ta được S<sub>AEH</sub> S<sub>CDH</sub> 2S<sub>ABC</sub>
5


+ = nên S<sub>BEHD</sub> 2S<sub>ABC</sub>
5
=
Mà ta có BDE



ABC


S BD.BE 1


S = BC.BA =5 nên ta được BDE BEDH


1


S S


2


= . Từ đó suy ra SBDE =SHDE


<b>+ Điều kiện đủ.</b>Giả sử S<sub>BDE</sub> =S<sub>HDE</sub>.


Theo định lí Menelaus ta có EA DB HC. . 1


EB DC HA= nên ta được


AE 2AH


BE = HC


Suy ra AE 2AH


AB =AC AH+ và


EB AC AH



AB AC AH



=


+


Do đó BDE EAH CDH


ABC ABC ABC


S


2S S


1


S +S + S = hay


2BD.BE AE.AH CD.CH


1


BC.BA +AB.AC + BC.CA = . Từ đó ta được


(

)



(

) (

)

(

)



(

)(

)




2


2 2


2 AC AH <sub>2AH</sub> 2 AC AH


1
3AC


3 AC AH AC AH AC


AC 5AH.AC 4AH 0 AC 4AH AC AH 0 AC 4AH


− −


+ + =


+ +


</div>
<span class='text_page_counter'>(74)</span><div class='page_container' data-page=74>

Từ AC 4AH= ta được


2
2 AC


AB AC.AH


4


= = nên tam giác ABC vuông tại B.



Vậy ta có điều phải chứng minh.


<b>Bài 54.</b> Cho tam giác nhọn ABC có ba đường cao AA’, BB’, CC’. Gọi D, E và F lần
lượt là tâm đường tròn nội tiếp các tam giác AB’C’, BC’A’ và CA’B’. Tính bán kính
đường tròn ngoại tiếp tam giác DEF theođộ dài các cạnh của tam giác ABC


<b>Lời giải</b>


<i><b>Trước hết ta phát biểu và chứng minh bổ đề.</b> Cho </i>


<i>tam giác ABC vuông tại A có </i>C=  450<i>. Khi đó ta </i>
<i>ln có </i>sin2 =2sin .cos . 


<b>Chứng minh.</b>Gọi AH là đường cao và AM a= là


đường trung tuyến của tam giác ABC. B H M C


A


Khi đó tam giác AHC có <sub>AHC 90 ; C</sub>= 0 =<sub> nên </sub> AH CH


sin ; cos


AC AC


=  Vì AM là trung


tuyến ứng với BC nên AMB 2= . Xét tam giác AHM có AHM 90 ; AMH 2= 0 =  suy
ra sin AMH AH



AM


= hay sin 2 AH


a


=


Do đó sin .cos 2.CH 2AH.CH<sub>2</sub> 2AH.CH AH


AC AC BC.CH a


 = = = = .


Do đó ta được sin2 =2sin .cos 


<b>Trở lại bài tốn.</b>Gọi I là tâm đường trịn nội
tiếp tam giác ABC. Gọi r, p, S lần lượt là bán
kính đường trịn nội tiếp, nửa chu vi và diện
tích của tam giác ABC. Khi đó dễ dàng
chứng minh được S pr= . Đặt


AB c; BC a; CA b= = =
BAC 2 ; CBA 2 ; ACB 2=  =  = 


F
E


D



I
C'


B'


A' C


B


A


Khi đó ta được 0  ; ; 900 và   + + =900. Dễ dàng chứng minh được


C' A' B CA' B' 2 ; A' B' C= =  =AB' C' 2 ; B' C' A=  =BC' A' 2= 


</div>
<span class='text_page_counter'>(75)</span><div class='page_container' data-page=75>

Do đó ta được A' EB∽ A' FB'∽ AIB. Suy ra ta có A'E A'F AI
A' B= A' B'=AB


Do đó ta lại được A' EF∽ A' BB' nên ta được EF A'E


BB' =A' B'


Từ đó ta được EF AI EF AI .BB' BB'.AI AI.sin 2 .


BB' =AB = AB = AB = 


Cũng từ A' EF∽ A' BB' ta có <sub>EFA' BB' A 90</sub>= = 0−<sub>CBA 90</sub>= 0−<sub>2</sub>


Do đó ta được IFE IFA' EFA'= − =

(

 +

)

(

900−2

)

= + + + −    900 = = CBE


Tương tự ta cũng được IDE IFE= = và IDF IEF= = nên ta được EDF= + 


Hoàn toàn tương tự ta cũng được FED= + ; DFE= + .


Gọi R là bán kính đường trịn nội tiếp tam giác DEF. Áp dụng định lí sin cho tam
giác DEF và áp dụng đổ đề trên ta được ta được


(

)



EF EF EF AI.sin 2


R IA.sin r


2cos 2cos


2 sin
2 sin EDF


 <sub></sub>


 


 


= = = = = =


+


Từ đó ta được R r S p p a p b p c

(

)(

)(

)

(

p a p b p c

)(

)(

)




p p p


− − − − − −


= = = =


Vậy bán kính đường tròn ngoại tiếp tam giác DEF là R

(

p a p b p c

)(

)(

)


p


− − −


= .


<b>Bài 55.</b> Cho tam giác ABC nội tiếp đường tròn

(

O; R

)

có các đường cao AD, BE, CF.
Tìm cơng thức liên hệ giữa diện tíchtam giác ABC với chu vi tam giác DEF.


<b>Lời giải</b>


<i><b>Ta xét các trường hợp sau</b><b>. </b></i>


<b>+ Trường hợp 1.</b>Tam giác ABC vuông, không mất tính
tổng qt ta giả sử A=900. Khi đó các điểm E, F trùng
với A nên không tồn tại tam giác DEF.


<b>+ Trường hợp 2.</b>Tam giác ABC nhọn. Gọi H là trực tâm
tam giác ABC. Trước hết ta chứng minh điểm O nằm


trong tam giác ABC.



Thật vậy, vẽ đường kính AT cắt CB tại V. Giả sử O nằm


R P


O
H


F E


D C


B


</div>
<span class='text_page_counter'>(76)</span><div class='page_container' data-page=76>

ngoài tam giác ABC và thuộc đường thẳng VT, khi đó ta


được 0


180 BOT COT+ =2BAO 2CAO+ =2BAC do đó


suy ra BAC 90 0, mâu thuẫn với tam giác ABC nhọn.


Như vậy tam giác ABC nhọn thì O nằm trong tam giác. Gọi K là giao điểm AO và FE
Dễ thấy các tứ giác AEHF và BDHF nội tiếpđường trịn. Kẻ OP vng góc với AC tại


P, theo tính chất góc nội tiếp ta có 0


AEF=AHF=ABC=AOP=90 −OAE. Do đó ta
được AKE 90= 0 hay OA vng góc với EF. Chứng minh tương tự ta được OB vng


góc với DF và OC vng góc với DE. Từ đó ta được S<sub>ABC</sub> =S<sub>AFOE</sub>+S<sub>BFOD</sub>+S<sub>CDOE</sub>.


Suy ra 2S<sub>ABC</sub> =OA.EF OB.FD OC.DE R EF FD DE+ + =

(

+ +

)

=2R.p<sub>DEF</sub>


Với pDEF là nửa chu vi tam giác DEF.


<b>+ Trường hợp 3.</b> Tam giác ABC tù, khơng mất tính tổng


quát ta giả sử 0


BAC 90 . Khi đó ta có H và O nằm ngồi
tam giác ABC. Chứng minh tương tự ta cũng được OA


vuông góc với EF. Xét đường trong tâm Q đối xứng với
đường tròn tâm O qua BC. Dễ thấy

( )

Q là đường tròn
ngoại tiếp tam giác HBC và hai đường tròn

( )

O và đường


tròn

( )

Q có bán kính bằng nhau. Đoạn thẳng OQ vng


góc với BC tại M. Xét tam giác HBC có ba đường cao HD,


CE và BF.


M


N
Q


O
H


F


E


D C
B


A


Khi đó áp dụng kết quả trường hợp 2 ta được S<sub>HBC</sub> =R DE EF FD

(

+ +

)

. Mặt khác ta có


(

)



HBC ABC ABH ACH ABC ABC


2S =2 S +S +S =2S +AH.BD AH.CD 2S+ = +AH.BC


Từ đó ta được 2S<sub>ABC</sub>+AH.BC R DE EF FD=

(

+ +

)

2S<sub>ABC</sub> =R DE EF FD

(

+ +

)

−AH.BC


Kẻ BQ cắt đường tròn tâm Q tại N, ta thấy AH//CN//OQ và AH CN 2MQ OQ= = =
Trong tam giác vuông BQM có <sub>MQ</sub>2+<sub>MB</sub>2 =<sub>R</sub>2 <sub>4MQ</sub>2+<sub>BC</sub>2 =<sub>4R</sub>2


Từ đó ta được

(

)

2 2


ABC DEF


</div>
<span class='text_page_counter'>(77)</span><div class='page_container' data-page=77>

<b>Bài 56.</b>Cho tam giác ABC có diện tích S ngoại tiếp đường trịn

( )

O; r . Kẻba tiếp tuyến
của đường tròn

( )

O; r gồm tiếp tuyến song song với BC cắt CA, AB lần lượt tại M, N;
tiếp tuyến song song với CA cắt AB, BC lần lượt tại P, Q; tiếp tuyến song song với AB
cắt BC, AC lần lượt tại R, S. Chứng minh rằng ta luôn có S<sub>MNPQRS</sub> 2S


3


 .


<b>Lời giải</b>
Gọi diện tích các tam giác AMN, BPQ, CRS lần lượt


là S ; S ; S1 2 3. Gọi chiều cao hại từ A của tam giác


AMN và ABC lần lượt là h<sub>1</sub> và h<sub>a</sub> thì ta thu được


1 a


h =h −2r. Dễ thấy hai tam giác ANM và ABC


đồng dạng với nhaunên ta được


2 2 2


a


1 1


a a a


h 2r


S h 2r


1


S h h h



   −   
=<sub></sub> <sub></sub> =<sub></sub> <sub></sub> =<sub></sub> − <sub></sub>
     
S
R
Q
P
N
M
C
B
A


Tương tự nếu gọi h ; h<sub>b</sub> <sub>c</sub> lần lượt là đường cao hạ từ B, C của tam giác ABC. Khi đó ta



2
2
b
S 2r
1
S h
 
=<sub></sub> − <sub></sub>
  và


2
3
c


S 2r
1
S h
 
=<sub></sub> − <sub></sub>


  . Dễ thấy a b c

(

)



1 1 1


S a.h b.h b.h r a b c


2 2 2


= = = = + +


Suy ra


a b c


1 a b c a b c 1 1 1


r 2S 2S 2S 2S h h h


+ +


= = + + = + + . Từ đó ta được


2 2 2 2



1 2 3


a b c a b c


2 <sub>2</sub>


a b c


S S S 2r 2r 2r 1 2r 2r 2r


1 1 1 1 1 1


S h h h 3 h h h


1 1 1 1 1 1 1


3 2r 3 2r.


3 h h h 3 r 3


       
+ +
=<sub></sub> − <sub></sub> +<sub></sub> − <sub></sub> +<sub></sub> − <sub></sub>  <sub></sub> − + − + − <sub></sub>
       
    
=  −  + +  = <sub></sub> − <sub></sub> =
 
  
 



Do đó suy ra S<sub>1</sub> S<sub>2</sub> S<sub>3</sub> 1S
3


+ +  nên ta được S<sub>MNPQRS</sub> 2S
3


 . Dấu bằng xẩy rakhi và chỉ


khi <sub>a</sub> <sub>b</sub> <sub>c</sub>


a b c


2r 2r 2r


1 1 1 h h h


h h h


− = − = −  = = hay tam giác ABC đều.


<b>Bài 57.</b>Cho tam giác ABC nội tiếp đường tròn

(

O; R

)

và ba đường trung tuyến AM,
BN, CP lần lượt là .. Chứng minh rằng


a b c


1 1 1 2


m +m +m R


</div>
<span class='text_page_counter'>(78)</span><div class='page_container' data-page=78>

A'



P N


M
O


C
B


A




N'
P'


A'
P


M
N


C
B


A


<i><b>Ta xét hai trường hợp sau đây</b><b>. </b></i>


<b>+ Trường hợp 1.</b> Xét tam giác ABC khơng tù. Ta có R OA AM OM=  − , dấu bằng xẩy


ra khi O thuộc đoạn AM. Do đó ta được


a


R OA AM OM OM


1


m MA MA AM




=  = − . Gọi AA’ là


đường cao của tam giác ABC. Khi đó ta lại có OBC
ABC


S


OM OM


AM  AA'=S , đẳng thức xẩy ra khi


và chỉ khi hai điểm M, A’ trùng nhau.Do vậy ta được OBC


a ABC


S
R



1


m  −S , đẳng thức xẩy ra


khi và chỉ khi O thuộc đoạn AM đồng thời hai điểm M, A’ trùng nhau, điều này có
nghĩa là tam giác ABC cân tại A. Hoàn toàn tương tự ta cũng được OAC


b ABC


S
R


1


m  −S và


OAB


c ABC


S
R


1


m  −S . Dấu bằng xẩy ra lần lượt tại B, C Do O nằm trong tam giác ABC nên ta
có S<sub>OAB</sub>+S<sub>OBC</sub>+S<sub>OCA</sub> =S<sub>ABC</sub>. Từ đó ta được


OAB OBC OCA



a b c ABC


S S S


R R R


3 3 1 2


m m m S


+ +


+ +  − = − =


Hay ta được


a b c


1 1 1 2


m +m +m R . Dấu bằng xẩy ra khi và chỉ khi tam giác ABC đều.


<b>+ Trường hợp 2.</b> Xét tam giác ABC tù. Khơng mất tính tổng qt ta giả sử A900.


Qua A kẻ đường thẳng vuông góc với BC cắt đường trịn (O; R) tại A’. Gọi N’ và P’
lần lượt là các trung điểm của A’C và A’B.


Khi đó có thể thấy ' ' '


a a b b c c



m =AM A'M m ; m = =BN BN' m ; m = =CP CP' m =


Do đó ta được <sub>'</sub> <sub>'</sub> <sub>'</sub>


a b c a b c


1 1 1 1 1 1


m +m +m  m +m +m và ta cũng có tam giác A’BC là tam giác


nhọn. Chứng minh tương tự trường hợp 1 ta được <sub>'</sub> <sub>'</sub> <sub>'</sub>


a b c


1 1 1 2


R


</div>
<span class='text_page_counter'>(79)</span><div class='page_container' data-page=79>

Do đó ta được


a b c


1 1 1 2


m +m +m R .


Vậy ta ln có


a b c



1 1 1 2


m +m +m R . Dấu bằng xẩy ra khi và chỉ khi tam giác ABC đều.


<b>Bài 58.</b>Cho tam giác nhọn ABC có diện tích S và BC a= . Trên cạnh BC lấy điểm D sao


cho DB k


DC= . Tính diện tích tam giác có đỉnh là tâm các đường trịn ngoại tiếp các tam
giác ABC, ABC, ACD theo a, k, S.


<b>Lời giải</b>
Gọi O; O ; O<sub>1</sub> <sub>2</sub> theo thứ tự là các tâm đường tròn
ngoại tiếp của các tam giác ABC, ABD, ACD. Khi
đó ta suy ra được OO<sub>1</sub>vng góc với ABtại M và


MA MB= , OO<sub>2</sub>vng góc với AC tại N và


NA NC= . Kẻ O E1 vuông góc với BC và O E1


vng góc với OF. Các điểm M và E nằm trên
đường trịn đường kính BO<sub>1</sub> nên OO F<sub>1</sub> =ABH.


F
Q
M
R
N
K


H
E
P
D
G
O<sub>2</sub>
O<sub>1</sub>
O
C B
A


Từ đó ta được OO F<sub>1</sub> ∽ ABH nên ta được OO1 AB


OF =AH hay 1


AB


OO OF.


AH


= (1).


Ta lại có OF PE 2 PE EB

(

)

2EB BC BD CD


2 2 2


+ − <sub>−</sub>


= = = = .



Từ giả thiết suy ra BC k 1
CD= + nên


a
CD


k 1
=


+ , do đó ta được

(

)



a
OF


2 k 1
=


+ .


Kết hợp với (1) ta được


(

)



1


a AB


OO .



AH
2 k 1
=


+ (2).


Tương tự kẻ O G<sub>2</sub> ⊥BC và OK⊥O G<sub>2</sub> thì ta được


(

)



2


ak AC


OO .


AH
2 k 1
=


+ (3).


Kẻ BR⊥AC và O Q2 ⊥OO1. Xét hai tam giác O OQ2 và BAR ta có O OQ2 =BAR nên


suy ra O OQ<sub>2</sub> ∽ BAR. Từ đó ta được 2


2 2


2



O Q BR BR


O Q O O.


O O =AB = AB (4).


</div>
<span class='text_page_counter'>(80)</span><div class='page_container' data-page=80>

(

)

(

)



1 2


2 4


OO O 2 1 2 1 2 2 2


1 1 BR ka 2S k.a


S O Q.OO OO .OO . .


2 2 AB <sub>8 k 1</sub> <sub>2S</sub> <sub>16S k 1</sub>


a


= = = =


+   +


 
 


.



<b>Bài 59.</b> Cho tam giác ABC có r ; r ; r<sub>a</sub> <sub>b</sub> <sub>c</sub>lần lượt là bán kính đường trịn bàng tiếp các góc
A, B, C. Gọi R, r lần lượt là bán kính đường tròn ngoại tiếp và nội tiếp tam giác.


Chứng minh rằng r<sub>a</sub>+ + =r<sub>b</sub> r<sub>c</sub> 4R r+


<b>Lời giải</b>
Gọi D, E, F lần lượt là tâm đường trịn bàng tiếp
các góc A, B, C và S là diện tích của tam giác ABC.
Đặt BC a, CA b, AB c= = = và p a b c


2
+ +


= .


Ta thấy S<sub>DAB</sub>+S<sub>DAC</sub>−S<sub>DBC</sub> = S r b c a<sub>a</sub>

(

+ −

)

=2S.


Hay ta được r p a<sub>a</sub>

(

)

=S.


Tương tự ta cũng có r p b<sub>b</sub>

(

) (

=r p c<sub>c</sub> −

)

=S.


O
I
F


E


D
C


B


A


Cộng theo vế các đẳng thức trên ta có p r

(

<sub>a</sub>+ +r<sub>b</sub> r<sub>c</sub>

) (

− r .a r .b r .c<sub>a</sub> + <sub>b</sub> + <sub>c</sub>

)

=3S. Do đó suy


ra p r

(

<sub>a</sub> + +r<sub>b</sub> r<sub>c</sub>

) (

=2 S<sub>BDC</sub>+S<sub>ECA</sub>+S<sub>FAB</sub>

)

+3S nên p r

(

<sub>a</sub>+ +r<sub>b</sub> r<sub>c</sub>

)

=2S<sub>DEF</sub>+S.


Gọi O là tâm đường tròn ngoại tiếp tam giác DEF, khi đó dễ thấy A, B, C là các chân
đường cao của tam giác DEF nên bán kính đường trịn ngoại tiếp ABC bằng 1


2 bán


kính đường trịn ngoại tiếp tam giác DEF. Dễthấy OD vng góc với BC nên ta được


OBDC


1


S OD.BC R.a


2


= = . Tương tự ta có S<sub>OCEA</sub> =R.b; S<sub>OAFB</sub> =R.c.


Cộng theo vế các đẳng thức trên ta có SDEF =R a b c

(

+ +

)

=2Rp nên
DEF


2S



4R


p =


Từ các kết quả trên ta được r<sub>a</sub>+ + =r<sub>b</sub> r<sub>c</sub> 4R r+ .


<b>Bài 60.</b>Cho tam giác đều ABC nội tiếp đường trịn tâm O. Chứng minh rằng tổng các
bình phương của khoảng cánh từ một điểm bất kì trên đường tròn đến các cạnh của
tam giác đều ABC bằng bình phương đường cao của tam giác đó.


</div>
<span class='text_page_counter'>(81)</span><div class='page_container' data-page=81>

Giả sử K là điểm bất kì trên đường trịn

( )

O . Khi đó nếu
điểm K trùng với một đỉnh của tam giác ABC thì kết
luận trên hiển nhiên đúng. Xét điểm không trùng với
các đỉnh của tam giác ABC. Khơng mất tính tổng qt ta
giả sử K nằm trên cung nhỏ AC của đường tròn

( )

O .


Gọi A ; B ; C<sub>1</sub> <sub>1</sub> <sub>1</sub> lần lượt là hình chiếu của K trên các cạnh
BC, CA, AB. Gọi AH là đường cao của tam giác ABC.


H
K


O


C<sub>1</sub>


B<sub>1</sub>
A<sub>1</sub>


C


B


A


Ta cần chứng minh 2 2 2 2


1 1 1


KA +KB +KC =AH .


Trước hết ta chứng minh ba điểm A ; B ; C<sub>1</sub> <sub>1</sub> <sub>1</sub>thẳng hàng.


Thật vậy, dễ thấy AKC C KA= <sub>1</sub> <sub>1</sub>=1200 nên các tứ giác AC B K; KB CA<sub>1</sub> <sub>1</sub> <sub>1</sub> <sub>1</sub> nơi tiếp
đường trịn có đường kính tương ứng là AK và CK. Từ đó ta được AKC<sub>1</sub> =AB C<sub>1</sub> <sub>1</sub> và


1 1 1


CKA =CB A . Từ 0


1 1


AKC C KA= =120 ta được AKC<sub>1</sub>+C KC C KC CKA<sub>1</sub> = <sub>1</sub> + <sub>1</sub>. Do đó
ta chỉ ra được AKC<sub>1</sub> =CKA<sub>1</sub>. Do đó AB C<sub>1</sub> <sub>1</sub>=CB A<sub>1</sub> <sub>1</sub> suy ra ba điểm A ; B ; C<sub>1</sub> <sub>1</sub> <sub>1</sub>thẳng


hàng.


Để ý là


1 1 1 1 1 1



A KC C KB B KA


S =S +S và CKB=BKA 60= 0 nên KC .KA<sub>1</sub> <sub>1</sub> =KC .KB<sub>1</sub> <sub>1</sub>+KB .KA<sub>1</sub> <sub>1</sub>.


Mặt khác ta lại có SAKB+SBKC−SAKC =SABC.


Do đó ta được AB.KC<sub>1</sub>+BC.KA<sub>1</sub>−AC.KB<sub>1</sub>=BC.AH hay KC<sub>1</sub>+KA<sub>1</sub>−KB<sub>1</sub>=AH.


Từ đó ta suy ra 2 2 2 2


1 1 1 1 1 1 1 1 1


KC +KA +KB +2KC .KA −2KC .KB −2KB .KA =AH .


Mà ta có KC .KA<sub>1</sub> <sub>1</sub>=KC .KB<sub>1</sub> <sub>1</sub>+KB .KA<sub>1</sub> <sub>1</sub> nên 2KC .KA<sub>1</sub> <sub>1</sub> =2KC .KB<sub>1</sub> <sub>1</sub>−2KB .KA<sub>1</sub> <sub>1</sub>=0.


Do đó ta được 2 2 2 2


1 1 1


KA +KB +KC =AH . Vậy bài toán được chứng minh.


<b>Bài 61.</b> Cho tam giác ABC có m ,l ,l<sub>a</sub> <sub>b</sub> <sub>c</sub> và p theo thứ tự là độ dài đường trung tuyến
hạ từ đỉnh A, độ dài đường phân giác trong hạ tứ đỉnh B, C và nửa chu vi của tam
giác. Chứng minh rằng m<sub>a</sub>+ + l<sub>b</sub> l<sub>c</sub> p 3


</div>
<span class='text_page_counter'>(82)</span><div class='page_container' data-page=82>

<i><b>Trước hết ta phát biểu và chứng minh bổ đề</b><b>.</b>Với </i>


<i>mọi </i><sub>0</sub>  <sub>45</sub>0<i><sub> ta ln có </sub></i> 2 1 cos 2



cos


2



= + <i>. </i>


<b>Chứng minh.</b> Xét tam giác ABC vuông tại A có


C= và đường cao AH, đường trung tuyến AM. B H M C


A


Trong tam giác AHM có <sub>AHM 90 ; AMH 2</sub>= 0 = <sub> và </sub><sub>cos2</sub> HM


AM


= .


Do đó 1 cos2 1 HM AM HM CM HM HC


AM AM AM AM


 + +


+ = + = = =


Ta có


2 <sub>2</sub>



2 CH 2CH 2CH 2CH CH


2 cos


AC BC.CH BC 2AM AM


=<sub></sub> <sub></sub> = = = =


 


Từ đó ta được <sub>cos</sub>2 1 cos 2


2




 = + , bổ đề được chứng minh.


<b>Trở lại bài toán.</b>Đặt AB c; BC a; CA b= = = , khi đó theo cơng thức về đường phân


giác ta có


<sub>b</sub> 2<sub>b</sub> 2
B


2cacos <sub>B</sub> <sub>B</sub>


2



l ca.cos l ac.cos


c a 2 2


=   


+


Áp dụng bổ đề trên ta có <sub>cos</sub>2 B 1 cos B


2 2


+


= , từ đó ta được 2
b


1 cos B
l ac


2
 + 


 <sub></sub> <sub></sub>


 


Mà theo định lí cosin ta có


2 2 2



c a b


cos B


2ca
+ −


= .


Suy ra

(

)

(

)



2 2 2


2


b b


ac c a b


l 1 p p b l p p a


2 2ca


 + − 


 <sub></sub> + <sub></sub>= −   −


  . Tương tự lc  p p c

(

)

.



Cũng theo công thức về đường trung tuyến ta có


(

)

(

)



(

)(

)

(

)(

)



2 2


2 2 2 2 2


a


4m 2b 2c a b c a b c


b c p b p c b c p b p c


 


= + − = + −<sub></sub> − − <sub></sub>


 


   


=<sub></sub><sub></sub> + + − − <sub> </sub><sub> </sub> + − − − <sub></sub><sub></sub>


Mặt khác ta lại có


(

)(

)




(

)(

)

(

)

2


b c 2 p b p c b c 2p b c 2p


b c 2 p b p c 2p p b p c


+ + − −  + + − − =


</div>
<span class='text_page_counter'>(83)</span><div class='page_container' data-page=83>

Do đó 4m<sub>a</sub>2 2p 2p<sub></sub> −

(

p a− − p b−

)

2<sub></sub> p p b

(

)

+ p p c

(

)

 2 p

(

2−m<sub>a</sub>2

)



  .


Suy ra l<sub>b</sub>+ l<sub>c</sub> 2 p

(

2−m2<sub>a</sub>

)



Do đó ta được

(

2 2

)

(

)

(

2 2 2

)



a b c a a a a


m + + l l m + 2 p −m  1 2 m+ +p −m =p 3


Dấu bẳng xẩy ra khi và chỉ khi tam giác ABC đều.


<b>Bài 62. </b>Cho tam giác nhọn ABC có h , h , h<sub>a</sub> <sub>b</sub> <sub>c</sub> và l ,l ,l<sub>a</sub> <sub>b</sub> <sub>c</sub> tương ứng là các đường cao và
đường phân giác hạ từ đỉnh A, B, C. Gọi r và R lần lượt là bán kính đường tròn nội
tiếp và đường tròn ngoại tiếp tam giác ABC. Chứng minh rằng:


a b c


a b c



h A h B h C r


sin sin sin


l 2 l 2 l 2 4R


   


− − − 


   


   


<b>Lời giải</b>


<i><b>Bổ đề.</b>Trong tam giác nhọn ABC ta ln có </i>sinAsin sinB C 1


2 2 2 8


Chứng minh. Vẽ đường phân giác AD ta có BD CD BD BD CD BC


AB AC AB AB AC AB AC


+


=  = =


+ + .



Vẽ BI⊥BCBI BD . Tam giác ABI có


A BI BD BC BC


sin


2 = AB AB= AB AC+ <sub>2 AB.AC</sub>


Chứng minh tương tự ta có


B AC C AB


sin ; sin


2 <sub>2 AB.BC</sub> 2 <sub>2 AC.BC</sub>


I
A


B D C


Nhân vế với vế của các bất đẳng thức trên ta được sinA.sin .sinB C 1


2 2 2 8


<b>Trở lại bài toán.</b>Gọi AA’ là đường phân giác hạ từ
đỉnh A, gọi p là nửa chu vi của tam giác ABC. Đặt


AB c; BC a; CA b= = = . Ta có S<sub>ABC</sub> =S<sub>ABA'</sub>+S<sub>ACA'</sub>.
Mà ta lại có



ABC a ABA' a ACA' a


1 1 A 1 A


S a.h ; S b.l .sin ; S c.l .sin


2 2 2 2 2


= = =


l<sub>a</sub>
h<sub>a</sub>


A' C


B


</div>
<span class='text_page_counter'>(84)</span><div class='page_container' data-page=84>

Do đó

(

)

a


a a


a


h


1 1 A b c A


a.h b c l .sin sin



2 2 2 l a 2


+


= +  =


Suy ra a

(

)



a


2 p a


h A b c A b c a A A


sin 1 sin .sin .sin


l 2 a 2 a 2 a 2




 +  + −


− =<sub></sub> − <sub></sub> = =


 


Hoàn toàn tương tự ta được b

(

)

c

(

)



b c



2 p b 2 p c


h B B h C C


sin .sin ; sin .sin


l 2 b 2 l 2 c 2


− −


− = − =


Do đó a b c

(

)(

)(

)



a b c


8 p a p b p c


h A h B h C A B C


sin sin sin .sin sin sin


l 2 l 2 l 2 abc 2 2 2


− − −


   


− − − =



   


   


Mà theo bổ đề sinAsin sinB C 1


2 2 2 8 và theo các công thức về diện tích là ABC
abc
S


4R
=


Và cơng thức Heron S<sub>ABC</sub> = p p a p b p c

(

)(

)(

)

ta được


(

)(

)(

)



8 p a p b p c <sub>A</sub> <sub>B</sub> <sub>C</sub> <sub>S</sub> <sub>r</sub>


.sin sin sin


abc 2 2 2 4Rp 4R


− − −


 =


Do đó ta được a b c


a b c



h A h B h C r


sin sin sin


l 2 l 2 l 2 4R


   


− − − 


   


   


<b>Bài 63.</b>Cho hình vng ABCD có cạnh a và hai điểm M, N thay đổi lần lượt trên BC,


CD sao cho góc 0


MAN=45 . Tìm giá trị lớn nhất và nhỏ nhất của diện tích tam giác


AMN.


<b>Lời giải</b>


Đặt BM=x; DN=y 0 x; y a

(

 

)

. Khi đó ta có


(

)



AMN ABCD ABM ADN CMN



S =S − S +S +S


Hay ta được


(

)(

)

(

)



2 2


AMN


1 1


S a ax ay a x a y a xy


2  2


= − <sub></sub> + + − − <sub></sub>= −


Trên tia đối của tiaBM lấy điểm K sao cho BK y= .


K y
y
x
N
M
D
C
B
A



Khi đó ta được ABK= ADN. Từ đó AN AK= và BAK=DAN.


Để ý là 0


BAM DAN 45+ = nên ta được 0


BAK BAM KAM+ = =45 .


Dễ thấy AKM= AMN nên ta được MN MK x y= = + . Mặt khác từ tam giác vuông


</div>
<span class='text_page_counter'>(85)</span><div class='page_container' data-page=85>

(

)

2 <sub>2</sub> <sub>2</sub> <sub>2</sub> <sub>2</sub> <sub>2</sub>

(

)

(

)

<sub>2</sub>


x y+ =a −2ax x+ +a −2ay y+ xy a= −a x y+ a x y+ =a −xy


Do vậy S<sub>AMN</sub> 1a x y

(

)

1at


2 2


= + = với t= +x y. Đến đây ta nhận thấy nếu t lớn nhất thì
diện tích tam giác AMN lớn nhất và ngược lại. Như vậy ta cần tìm giá trị lớn nhất và
nhỏ nhất của t.


Để ý là ta đang có x y a+ = và <sub>x.y a</sub>= 2−<sub>at</sub><sub>. Khi đó thaeo hệ thức Vi –</sub><sub> et ta có x, y là </sub>


nghiệm của phương trình bậc hai <sub>X</sub>2−<sub>tX a</sub>+ 2− =<sub>at 0</sub><sub>. Để phương trình trên có hai </sub>


nghiệm x, y ta cần có


(

)

(

)

2

(

)




2 2 2


t 4 a at 0 t 2a 8a 0 t 2a 2 2a t 2a 2 1


 = − −   + −   +    −


Khi t 2a=

(

2 1−

)

thì phương trình bậc hai có nghiệm kép là


(

<sub>) ( )</sub>



1 2


2a 2 1


t


X X a 2 1


2 2




= = = = −


Điều này có nghĩa là x y a= =

(

2 1−

)

và Mint 2a=

(

2 1−

)

.


Vậy ta được

(

) (

)



AMN



2
S


1


Min a.2a 2 1 a 2 1


2


= − = −


Lại có xy a= 2− at at a= 2−xy nên suy ra at a 2 t a


Điều này có nghĩa là Maxt a= , khi đó


AMN


2
S


1 a


Max a.a


2 2


= =


Trong trường hợp này ta được x a; y 0= = hoặc x 0; y a= = hay M B; N C  hoặc



M C; N D 


<b>Bài 64.</b> Cho hình chữ nhật ABCD có AB BC . Vẽ nửa đường trịn đường kính AB


trên nửa mặt phẳng chứa CD có bờ là đường thẳng AB. Gọi M là điểm bất kì trên nửa
đường tròn

(

MA, B

)

. Các đường thẳng MA và MB cắt CD lần lượt tại P và Q. Các
đường thẳng MC, MD cắt đường thẳng AB lần lượt tại E và F.


</div>
<span class='text_page_counter'>(86)</span><div class='page_container' data-page=86>

Đặt AB CD a, BC= = =b a

(

b

)

. Kẻ MN vng góc


với BC tại N, khi đó theo định lí Talet ta có


PQ QM CN


AB = BM = BN và


EF EM BN


CD=MC =CN
Suy ra PQ AB.CN a.CN


BN BN


= = và EF CD.BN a.BN


CN CN


= =



Do đó ta được PQ EF a CN BN


BN CN


 


+ = <sub></sub> + <sub></sub>
 .


Q C P


M
F
E
D
N
B
A


Đặt S CN BN


BN CN


= + , khi đó ta có


(

)

2


2 2 <sub>CN BN</sub> <sub>2CN.BN</sub> 2


CN BN CN BN b



S 2


BN CN CN.BN CN.BN CN.BN


+ −


+


= + = = = −


Do đó ta được 1 CN.BN<sub>2</sub>


S 2+ = b nên


(

)

2

(

)

2


2 2


CN BN 4CN.BN CN BN


4
1


S 2 b b


+ − −


− = =



+ .


Do M nằm trên đường tròn đường kính AB nên NB AB a


2 2


 = , suy ra CN 2b a
2





Do đó ta được

(

)



2


2


2 2


b a


4 4 2ab a


1


S 2 b S 2 b


− <sub>−</sub>



−   


+ + , nên ta có


2 2


2


4b 4ab 2a


S


2ab a
− +


− .


Từ đó ta suy ra


2 2


4b 4ab 2a


EF PQ


2b a
− +
+ 



− , đẳng thức xẩy ra khi và chỉ khi


a
NB


2
= hay


M nằm chính giữa nửa đường trịn đường kính AB.
Vậy giá trị nhỏ nhất của PQ EF+ là


2 2


4b 4ab 2a


2b a
− +


− , xẩy ra khi M nằm chính giữa nửa


đường trịn đường kính AB.


<b>Bài 65.</b>Trong các tam giác nội tiếpđường tròn

(

O; R

)

cho trước, tìm tam giác có chu
vi lớn nhât.


</div>
<span class='text_page_counter'>(87)</span><div class='page_container' data-page=87>

Xét tam giác ABC nội tiếp đường tròn

(

O; R

)

. Giả sử
điểm M là điểm chính giữa cung BAC. Kẻ đường kính
MN của đường trịn

(

O; R

)

, khi đó MN vng góc với
BC tại trung điểm H của BC và MBN 90= 0. Đặt



MH h.= Trong tam giác vng MBN có BH là đường


cao nên ta có


H
O


N
M


C
B


A


2


MB =MN.MH 2R.h= và 2

(

)



BH =MH.MH=h 2R h−


Gọi p và p<sub>1</sub> lần lượt là nửa chu vi của tam giác ABC và MBC, theo bài ra ta có p p <sub>1</sub>


và dấu bằng xẩy ra khi A và M trùng nhau.


Ta có

(

)



2 <sub>2</sub> <sub>2</sub> <sub>2</sub>


2



2 2 2


1


MB MB MB MB MB


p MB BH BH 3 BH 3 BH


2 2 4 4 2


   


 


= + =<sub></sub> + + <sub></sub>  <sub></sub> + + <sub></sub>= <sub></sub> + <sub></sub>


     


Dấu bằng xẩy ra khi và chỉ khi MB BH


2 = hay MB BC=


Theo như trên ta có

(

)

(

)



2


2 2


2



MB 3R 27R


3 BH 3 Rh h 2R h 3h 3R h 2


2 2 4


 <sub>+</sub> <sub>=</sub> <sub></sub> <sub>+</sub> <sub>−</sub> <sub></sub><sub>=</sub> <sub>−</sub> <sub></sub>   <sub>=</sub>


  <sub></sub> <sub></sub>  


 


 


Dấu bằng xẩy ra khi và chỉ khi h 3R h h 3R
2
= −  =


Do đó 2 2


1


27


p R


4


 hay p<sub>1</sub> 3 3R


2


 , dấu bằng xẩy ra khi và chỉ khi


MB BC
3


MH R


2


 =





=


 hay tam


giác MBC đều.


Từ đó ta được 2p 3 3R , dấu bằng xẩy ra khi và chỉ khi A M và tam giác MBC


đều hay tam giác ABC đều. Vậy trong các tam giác nội tiếp đường tròn (O; R) thì tam


giác đều có chu vi lớn nhất và chu vi lớn nhất bằng 3 3R.


<b>Bài 66. </b>Cho tứ giác ABCD nội tiếp đường tròn. Chứng minh rằng:
AB CD−  AC BD− .



</div>
<span class='text_page_counter'>(88)</span><div class='page_container' data-page=88>

Gọi E, F lần lượt là trung điểm của AC, BD. Khi đó ta
có áp dụng cơng thức về đường trung tuyến của tam
giác ta được


2 2 2 2


2 2 2 2


2 2 2 2


1


AB AD 2AE BD


2
1


BC CD 2CE BD


2
1


EA EC 2EF AC


2


+ = +


+ = +



+ = +


M


F
E


D
B


C


A


Cộng theo vế các đẳng thức trên ta được


(

)



2 2 2 2 2 2 2 2 2 2


AB +BC +CD +DA =2 AE +EF +BD =BD +AC +4EF


Do tứ giác ABCD nội tiếp nên theo định lí Ptoleme ta được AC.BD AB.CD AD.BC= +


Từ đó ta được

(

AB CD−

) (

2+ AD BC−

) (

2 = AC BD−

)

2+4EF2


Gọi M là trung điểm của AB, khi đó ta được AD 2ME; BC 2MF= =


Từ đó suy ra 2 ME MF− = AD BC− .



Mà trong tam giác MEF ta có <sub>EF</sub> <sub>ME MF</sub>− <sub>2EF</sub> <sub>AD BC</sub>− <sub>4EF</sub>2 

(

<sub>AD BC</sub>−

)

2


Do đó kết hợp với đẳng thức trên ta được


(

) (

2

) (

2

) (

2

)

2


AB CD− + AD BC−  AC BD− + AD BC−
Suy ra

(

AB CD−

) (

2  AC BD−

)

2 AB CD−  AC BD−


Dấu bằng xẩy ra khi và chỉ khi EF= ME MF−  ba điểm M, E, F thẳng hàng, điều
này dẫn đến tứ giác ABCD là hình thang hoặc hình chữ nhật.


<b>Bài 67. </b>Cho tam giác ABC và đường tròn

( )

I nội tiếp tam giác tiếp xúc với các cạnh
AB, BC, CA lần lượt tại F, D, E. Gọi M là giao điểm của BC với đường phân giác trong
của góc BIC và N là giao điểm của EF với đường phân giác trong của góc EDF .


Chứng minh rằngba điểm A, M, N thẳng hàng.
<b>Lời giải</b>


</div>
<span class='text_page_counter'>(89)</span><div class='page_container' data-page=89>

+ Xét trường hợp tam giác ABC khơng cân tại A.
Khơng mất tính tổng qt ta giả sử AB AC .


Gọi P, Q theo thứ tự là giao điểm của AI với

( )

I


và EF. Do P là giao điểm của AI với đường tròn


( )

I nên P nằm chính giữa cung nhỏ EF của
đường trịn

( )

I . Khi đó dễ thấy ba điểm D, N, P
thẳng hàng nên ta có


P
Q
N


M
D
I
F


E


C
B


A




(

)



(

) (

)



(

)



0


0 0


0 0 0



0


0 0


1 1


IMC MIB MBI BIC MBI 180 IBC ICB MBI


2 2


1 ABC ACB ABC ABC ACB


180 90


2 2 2 2 4 4


1 1


NDC NDE EDC FDE EDC 180 90 IBD 90 ICD


2 2


1


180 FDB EDC EDC


2


IBD ICD ABC ACB



90 90


2 2 4 4


• = + = + = − − +


 


= <sub></sub> − − <sub></sub>+ = + −


 


 


• = + = + = <sub></sub> − − − − <sub></sub>


 


= − − +


= + − = + −


Do đó ta được IMC=NDC do đó ta được IM song song với ND. Do đó ta suy ra IM
song song với. Để ý là ID IP= nên ta được MID IDP QPN= = . Do BC tiếp xúc với
đường tròn

( )

I tại D và P là điểm chính giữa cung EF nên ta được


1 1 1


IDM PDM IDP sdPED IDP sdPE sdED IDP



2 2 2


1 1


sdPF sdED IDP PNF QPN PQN


2 2


= − = − = + −


= + − = − =


Do đó ta được IDM∽ PQN, suy ra IM DI


PN =QP mà ta lại có ID IP= do đó


IM PI


PN =QP


Mặt khác ta có IAE=900, EQ⊥IP và IE=IP nên ta được IQ.IA IE= 2 =IP2


Do đó ta có QP 1 IQ 1 IP PA IP IA


IP = − IP = −IA= IA PQ= AP nên ta được


IM AP


PN= IA .



</div>
<span class='text_page_counter'>(90)</span><div class='page_container' data-page=90>

<b>Bài 68.</b> Gọi O là tâm đường tròn nội tiếp tam giác nhọn ABC. Đường tròn

( )

O tiếp
xúc với các cạnh AB, BC, CA theo thứ tự tại F, E, D. Đường phân giác trong của góc


BOC cắt BC tại I và AI cắt EF tại K. Chứng minh rằng KD 1 4DE.DE EF2
2


 −


<b>Lời giải</b>


Ta định nghĩa lại điểm K là giao điểm của đường phân giác của góc EDF. Khi đó ta ta
đi chứng minh ba điểm A, K, I thẳng hàng. Để chứng minh ba điểm A, K, I thẳng
hàng ta có thể trình bày tương tự như lời giải bài toán 67. Ở đây ta trình bày lời giải
theo một hướng khác như sau.


Ta có biến đổi góc như sau


0 0 o ABC o ACB ABC ACB


EDF 180 FDB EDC 180 90 90


2 2 2


    <sub>+</sub>


= − − = −<sub></sub> −  <sub> </sub>− − <sub></sub>=


   



Mà ta lại có <sub>KDC</sub> EDF <sub>FDC</sub> ABC ACB <sub>90</sub>0 ACB <sub>90</sub>0 ABC ACB


2 4 4 2 4 4


= + = + + − = + − .


Mặt khác do OI là phân giác của góc BOC nên ta có


0 0


ABC ABC BOC 1 ABC ACB ABC ACB


OIC BOI 180 90


2 2 2 2 2 2 4 4


 


= + = + = <sub></sub> − − <sub></sub>= + −


 


Kết hợp hai kết quả trên ta suy ra KDC OIC= nên OI song song với DK


Gọi giao điểm của DK với đường trịn

( )

O là P, khi đó dễ thấy ba điểm A, P, O thẳng
hàng. Giả sử AO cắt EF tại Q. Do DK song song với OI nên ta có KPO KDO= =DOI,


điều này dẫn đến hai tam giác KQP và DIO đồng dạng với nhau. Từ đó ta được


KP PQ PQ



OI =OD =OF . Mặt khác ta lại có P là điểm chính giữa cung nhỏ EF nên FP là phân


giác của tam giác AFQ, theo tích chất đường phân giác kết hợp hai tam giác AFO và
AQF ta suy ra được PA FA AO


PQ =FQ = FO , suy ra


PA PQ


OA= FO.


Kết hợp hai kết quả trên ta được AP KP


AO= OI , mà ta có KP song song với IO nên theo


định lí Talets ta suy ra được ba điểm A, K, I thẳng hàng.


</div>
<span class='text_page_counter'>(91)</span><div class='page_container' data-page=91>

(

)(

)



(

)



(

)



(

)



(

)



2
2



2 2


3 2


2


DE.DF DE DF EF


DE.DF DE DF EF DE DF EF
DK


DE DF DE DF


DE.DF.EF EF


DE.DF DE.DF


4
DE DF


 + − 


+ + + − <sub></sub> <sub></sub>


= =


+ +


= −  −



+


Hay ta được <sub>KD</sub> 1 <sub>4DE.DE EF</sub>2


2


 − . Đẳng thức xẩy ra khi và chỉ khi DE=DF và do


đó AB AC= hay tam giác ABC cân tại A.


<b>Bài 69.</b>Cho tam giác ABC với các cạnh AB c, BC a, CA b= = = ngoại tiếp đường tròn


tâm I bán kính r. Gọi A , B , C<sub>1</sub> <sub>1</sub> <sub>1</sub> lần lượt là tiếp điểm của đường tròn I với các cạnh
BC, CA, AB. Các tia AI, AI, CI cắt đường tròn tâm I lần lượt tại A’, B’, C’. Đặt


i i i 1 1 1 1 1 1


A B =c , B C =a , C A =b với i 1, 2= . Chứng minh rằng


3 2 3 6


2 2 2
2 2 2
1 1 1


a b c 216r
abc


a b c  , dấu



đẳng thức xẩy ra khi nào?<b> </b>


<b>Lời giải</b>


<b>Bổ đề. </b><i>Trong tam giác nhọn ABC ta ln có </i>cosA p p a

(

)



2 bc



=


<b>Chứng minh.</b> Xét ta giác nhọn ABC có AD là đường phân giác trong, khi đó ta có


(

)

(

)

(

)



A


2bc.cos <sub>2 bcp p a</sub> <sub>A</sub> <sub>2 bcp p a</sub> <sub>b c</sub> <sub>p p a</sub>


2


AD cos .


b c b c 2 b c bc bc


− − <sub>+</sub> −


= =  = =



+ + +


Ngoài ra ta chú ý đến nhận xét: Trong tam giác ABC thì sinA B sin A sin B


2 2


+ <sub></sub> +


.


<b>Trở lại bài toán.</b>Gọi A, B, C lần lượt là số đo các


góc BAC; ABC; ACB, A ; B ; C<sub>1</sub> <sub>1</sub> <sub>1</sub>lần lượt là số đo


góc B A C ; A B C ; A C B1 1 1 1 1 1 1 1 1 và A ; B ; C2 2 2 lần


lượt là số đo góc B A C ; A B C ; A C B<sub>2</sub> <sub>2</sub> <sub>2</sub> <sub>2</sub> <sub>2</sub> <sub>2</sub> <sub>2</sub> <sub>2</sub> <sub>2</sub>. Gọi p
và S lần lượt là nửa chu vi và diện tích của tam
giác ABC.


C<sub>2</sub>
A<sub>2</sub>


B<sub>1</sub>
C<sub>1</sub>


B<sub>2</sub>


A<sub>1</sub>
I



C
B


</div>
<span class='text_page_counter'>(92)</span><div class='page_container' data-page=92>

Dễ dàng tính được 1 1 1 1 1 1


2 2 2


B C C A A B


A ; B ; C


2 2 2


+ + +


= = = . Khi đó áp dụng nhận


xét trên ta được


(

)(

)(

)



2 3 1 1 1 1 1 1


2 2 2 2 2 2


3


1 1 1 1 1 1



3


1 1 1 1 1 1


3


1 1 1 1 1 1


B C C A A B


a b c 8r .sin A .sin B .sin C 8r .sin .sin .sin


2 2 2


r sin B sin C sin C sin A sin A sin B


8r . sin B .sin C . sin C .sin A . sin A .sin B
8r .sin A .sin B .sin C a b c


+ + +


= =


 + + +




= =


Từ đó ta suy ra



3 2 3


3
2 2 2


1 1 1 1 1 1


2 2 2
1 1 1


a b c


a b c 8r .sin A .sin B .sin C


a b c  =


Ta lại có A<sub>1</sub> B C; B<sub>1</sub> C A; C<sub>1</sub> A B


2 2 2


+ + +


= = = và để ý là sin A<sub>1</sub> sinB C cosA


2 2


+


= =



Nên ta được


3 2 3


3
2 2 2


1 1 1
2 2 2


1 1 1


a b c A B C


a b c 8r .cos .co s .co s


2 2 2


a b c  = . Áp dụng bổ đề trên ta được


(

) (

) (

)



3 2 3 3 3 2


3
2 2 2
2 2 2
1 1 1



p p a p p b p p c


a b c 8r .p.S 8r .p


8r . . .


bc ca ab abc abc


a b c


− − −


 = =


Để ý ta ln có p 3 3r , do đó ta được


3 2 3 6


2 2 2
2 2 2
1 1 1


a b c 216r
abc


a b c  .


Bất đẳng thức được chứng minh. Dấu đẳng thức xẩy ra khi và chỉ khi tam giác ABC
đều



<b>Bài 70.</b> Cho đường tròn tâm O nội tiếp tam giác ABC. Các tiếp tuyên với (O) song
song với cá cạnh của ram giác ABC với sáu điểm M, N, P, Q, R, S sao cho
M,S AB; N,P AC; Q,R BC   . Gọi l , l , l<sub>1</sub> <sub>2</sub> <sub>3</sub> lần lượt là các đường phân giác trong
xuất phất từ đỉnh A, B, C của các tam giác AMN, BSR, CPQ. Gọi p là nửa chu vi của
tam giác ABC. Chứng minh rằng <sub>2</sub> <sub>2</sub> <sub>2</sub> <sub>2</sub>


1 2 3


1 1 1 81


l +l +l p


</div>
<span class='text_page_counter'>(93)</span><div class='page_container' data-page=93>

Gọi l , l , l<sub>a</sub> <sub>b</sub> <sub>c</sub> theo thứ tự là độ dài các đường phân
giác trong xuất phát từ đỉnh A, B, C của tam giác
ABC. Áp dụng công thức về đường phân giác cho


các tam giác ABC và AMN ta có


a
A
AB.AC.cos
2
l
AB AC
=


+ và 1


A
AM.AN.cos


2
l
AM AN
=
+


Gọi p , p , p1 2 3lầ lượt là nửa chu vi của tam giác


AMN, BSR, CPQ


l<sub>3</sub>
l<sub>2</sub>
l<sub>1</sub>
S
R Q
P
N
M
O
C
B
A


Do NM//BC nên theo định lí Talet ta có AM AN MN AM AN MN p1


AB AC BC AB AC BC p


+ +
= = = =
+ +


Suy ra
1 1
p p


AB AM. ; AC AN.


p p
= =
Do đó

(

)


1 1
a 1
1 1
1


p p A <sub>A</sub>


2 AM. AN.cos <sub>AM.AN.cos</sub>


p p 2 p <sub>2</sub> p


l . .l


p p AM AN p


AM AN
p
   
   
   


= = =
+
+


Hoàn toàn tương tự ta được <sub>b</sub> <sub>2</sub> <sub>c</sub> <sub>3</sub>


2 3


p p


l .l ; l .l


p p


= = . Do đó a b c


1 2 3 1 2 3


l l l 1 1 1


p


l l l p p p


 


+ + = <sub></sub> + + <sub></sub>


 



Mà theo tính chất các tiếp tuyến cắt nhau ta được p p= <sub>1</sub>+p<sub>2</sub>+p<sub>3</sub>


Và lại có

(

<sub>1</sub> <sub>2</sub> <sub>3</sub>

)



1 2 3


1 1 1


p p p 9


p p p


 


+ + <sub></sub> + + <sub></sub>


  , do đó


a b c


1 2 3


l l l


9
l +l +l  .


Áp dụng bất đẳng thức Bunhiacopxki ta được

(

)



2



2 2 2


a b c


a b c 2 2 2


1 2 3 1 2 3


l l l 1 1 1


l l l


l l l l l l


 


 


+ +  + +  + + 


  <sub></sub> <sub></sub>


   


Với AB c, BC a, CA b= = = , theo công thức về đường phân giác trong tam giác ta có


(

)



a



p p a
2bc


l .


b c <sub>bc</sub>



=


+ . Do đó

<sub>(</sub>

<sub>)</sub>

(

) (

)



2


a 2


4bc


l .p p a p p a


b c


= −  −


+ . Hồn tồn tương tự ta


cũng có l2<sub>b</sub>p p b ; l

(

)

2<sub>c</sub> p p c

(

)

. Do đó l<sub>a</sub>2 + + l2<sub>b</sub> l<sub>c</sub>2 p p a p b p c

(

− + − + −

)

=p2.
Suy ra



2
2


a b c


2 2 2


1 2 3 1 2 3


l l l 1 1 1


p


l l l l l l


 


 


+ +  <sub></sub> + + <sub></sub>


  <sub></sub> <sub></sub>


    nên ta được


2 2


2 2 2


1 2 3



1 1 1


81 p


l l l


 


 <sub></sub><sub></sub> + + <sub></sub><sub></sub>


 


Do đó ta suy ra được <sub>2</sub> <sub>2</sub> <sub>2</sub> <sub>2</sub>


1 2 3


1 1 1 81


l +l +l  p .


</div>
<span class='text_page_counter'>(94)</span><div class='page_container' data-page=94>

bàng tiếp các góc ở A, B, C. Gọi r là bán kính đường trịn nội tiếp tam giác ABC.
Chứng minh rằng


(

OI

)(

a

) (

OI

)(

b

) (

OI

)(

c

)



1 1


2R a b a c+ + + b c a b+ + + c a b c+ + 4r



<b>Lời giải</b>


Gọi AA ; BB ; CC<sub>1</sub> <sub>1</sub> <sub>1</sub> là các đường phân giác của tam giác ABC. Dựng EI<sub>a</sub> vuông góc


với AB tại E, FI<sub>a</sub> vng góc với AC tại F, EI<sub>a</sub> vng góc với OM tại M, FI<sub>a</sub>vng góc


với ON tại N, OP vng góc với AB tại P. Khi đó ta được B AC<sub>1</sub> <sub>1</sub>=BAC MON= (1)


Dễ thấy E và F là các tiếp điểm của đường trịn bàng tiếp góc A với AB và AC nên ta


được AE AF p; OM PE p c a b; ON p b a c


2 2 2 2


+ +


= = = = − = = − = với p a b c
2
+ +


= .


Khi đó ta được OM a b


ON a c


+
=


+ (2).



Theo tính chất đường phân giác ta có AB<sub>1</sub> bc ; AC<sub>1</sub> bc


c a a b


= =


+ + . Do đó


1
1


AB a b


AC a c


+
=


+ (3)


Từ (1), (2) và (3) ta được AB C<sub>1</sub> <sub>1</sub>∽ OMN nên suy ra B C1 1 AB1

<sub>(</sub>

2bc

<sub>)(</sub>

<sub>)</sub>



MN = OM= a b a c+ +
Suy ra


(

)(

)

(

a

)(

)

(

a

)(

)

(

)(

a

)



1 1



2bc.OI .sin MON 2bc.OI .sin BAC abcOI
2bc.MN


B C


a b a c a b a c a b a c R. a b a c


= = = =


+ + + + + + + +


Do đó ta được OI<sub>a</sub> R a b a c B C

(

)(

)

1 1
abc


+ +


= . Hoàn toàn tương tự


(

)(

)

1 1

(

)(

)

1 1


b c


R b c a b A C R b c c a A B


OI ; OI


abc abc


+ + + +



= =


Do đó

<sub>(</sub>

<sub>)(</sub>

a

<sub>) (</sub>

<sub>)(</sub>

b

<sub>) (</sub>

<sub>)(</sub>

c

<sub>)</sub>

(

)



1 1 1 1 1 1


OI OI OI R


Q A B B C A C


abc


a b a c b c a b c a b c


= + + = + +


</div>
<span class='text_page_counter'>(95)</span><div class='page_container' data-page=95>

<i><b>+ Trước hết ta chứng minh </b></i>Q 1
2R


• <b>Trường hợp 1.</b>Tam giác ABC không tù. Gọi
giao điểm của OA và B C<sub>1</sub> <sub>1</sub> là D, khi đó ta được


1 1


1 1 1 1 OB AC


R.B C =OA.B C 2S


Hoàn toàn tương tự ta được



1 1 1 1


1 1 OC BA 1 1 OB CA


R.C A 2S ; R.A B 2S


Do đó ta được R A B

(

<sub>1</sub> <sub>1</sub>+B C<sub>1</sub> <sub>1</sub>+C A<sub>1</sub> <sub>1</sub>

)

2S<sub>ABC</sub>.


F


E N


M


P D
C1


B<sub>1</sub>
A<sub>1</sub>


Ia


I
O


C
B


A



Mà lại có S<sub>ABC</sub> abc
4R


= . Từ đó ta được Q 1
2R
 .
• <b>Trường hợp 2. </b>Tam giác ABC tù, khơng mất
tính tổng qt ta giả sử 0


BAC 90 . Khi đó gọi


2


C và C3 là các điểm đối xứng với C1 qua BC


và AB. Từ đó A B<sub>1</sub> <sub>1</sub>+B C<sub>1</sub> <sub>1</sub>+C A<sub>1</sub> <sub>1</sub>C C<sub>2</sub> <sub>3</sub>. Dựng


2 3


AH⊥C C . Do 0


ACB 90 và C CC<sub>2</sub> <sub>3</sub> =2ACB
nên suy ra CC<sub>1</sub> =CC<sub>2</sub> =CC<sub>2</sub> CA.


C<sub>3</sub>
C<sub>2</sub>


B<sub>1</sub>
A<sub>1</sub>


C<sub>1</sub> H


C
B


A


Từ đó suy ra C C<sub>2</sub> <sub>3</sub> =2CC sin ACB 2b sin C<sub>3</sub>  . Do đó A B<sub>1</sub> <sub>1</sub>+B C<sub>1</sub> <sub>1</sub>+C A<sub>1</sub> <sub>1</sub> 2b sin C.


Tương tự ta cũng có A B<sub>1</sub> <sub>1</sub>+B C<sub>1</sub> <sub>1</sub>+C A<sub>1</sub> <sub>1</sub> 2c sin B.


Nên ABC


1 1 1 1 1 1


2S


bc bc


A B B C C A b sin C c sin B


2R 2R R


+ +  + = +  . Từ đó ta được Q 1


2R
 .


Kết hợp cả hai trường hợp ta được Q 1
2R


 .


<i><b>+ Chứng minh </b></i>Q 1


4r
 <i><b>. </b></i>


Theo định lí cosin ta được 2 2 2


1 1 1 1 1 1


</div>
<span class='text_page_counter'>(96)</span><div class='page_container' data-page=96>

(

)(

)



(

)(

)



(

)(

)

<sub>(</sub>

(

<sub>) (</sub>

)(

<sub>)</sub>

)

(

)(

)



2 <sub>2 2</sub> <sub>2</sub> <sub>2</sub> <sub>2</sub>


2
1 1


2 2 2 2


2


2 2


2 2



2
2


bc bc b c b c a


B C 2 .


a c a b a b a c 2bc


b c c b bc b c b c


c a c a b a b a b a c a b a c a


abc a b c b c


a bc a bc


a b a c <sub>a b</sub> <sub>a c</sub> a b a c


a bc ab. ac 1 ab ac 1 2a b c


4 4 2 36 2


4 ab. ac


    + −
=<sub></sub> <sub></sub> +<sub></sub> <sub></sub>−
+ + + +
   
   


= <sub></sub> − <sub></sub>+ <sub></sub> − <sub></sub>+
+ <sub></sub> + + <sub></sub> + <sub></sub> + + <sub></sub> + +
+ + −
= − 
+ + <sub>+</sub> <sub>+</sub> + +
 <sub>+</sub>  <sub></sub> <sub>+ +</sub>
 =  <sub></sub> <sub></sub> 
 
2

 
 


Do đó suy ra B C<sub>1</sub> <sub>1</sub> 2a b c
8
+ +


 . Tương tự ta được C A<sub>1</sub> <sub>1</sub> 2b c a; A B<sub>1</sub> <sub>1</sub> 2a b c


8 8


+ + + +


  .


Từ đó suy ra A B<sub>1</sub> <sub>1</sub> B C<sub>1</sub> <sub>1</sub> C A<sub>1</sub> <sub>1</sub> a b c p
2


+ +



+ +  = . Do đó ta được Q Rp 1


abc 4r
 = .


Như vậy ta được 1

<sub>(</sub>

OI

<sub>)(</sub>

a

<sub>) (</sub>

OI

<sub>)(</sub>

b

<sub>) (</sub>

OI

<sub>)(</sub>

c

<sub>)</sub>

1


2R a b a c+ + + b c a b+ + + c a b c+ +  4r.


Dấu đẳng thức xẩy ra khi và chỉ khi tam giác ABC đều.


<b>Bài 72.</b> Cho đường trịn tâm O bán kính R và dây cung BC(với BC R ). Điểm A di
động trên cung lớn BC và điểm D di động trên cung nhỏ BC. Xác định vị trí của A và


D để 1 1 1


DA+DB+DC đạt giá trị nhỏ nhất.


<b>Lời giải</b>


Với A, D bất kì ta ln có AD 2R . Với mỗi điểm D
trên cung nhỏ BC ta luôn tìm được điểm A trên cung
lớn BC sao cho AD 2R= để 1 1


AD =2R có giá trị bé nhất.


Kẻ DH vng góc với BC tại H. Kẻ đường kính EF
vng góc với BC tại K. Khi đó các điểm E, F, K là các


điểm cố định. Do 0



ABD CHD 90= = và DAB DCB= nên


ta được ABD∽ CHD.Từ đó suy ra


O
H
K
F
E
D
C
B
A
BD DH


DB.DC AD.DH DB.DC 2R.DH


DA= DC  =  =


</div>
<span class='text_page_counter'>(97)</span><div class='page_container' data-page=97>

Áp dụng bất đẳng thức Cauchy ta được 1 1 2 1 2 1 2
DB+DC  CB.CD = 2R.DH  <sub>2R.EK</sub>


Từ đó ta được 1 1 1 1 2 1 2


DA+DB+DC 2R+ <sub>2R.EK</sub> = 2R+BE. Dễ thấy


1 2


2R+BE là một



hằng số. Do đó 1 1 1


DA+DB+DC đạt giá trị nhỏ nhất là


1 2


2R+BE


Dấu bằng xẩy ra khi và chỉ khi DA trùng với đường kính EF.


<i><b>Nhận xét</b><b>. </b>Có nhiều cách để tìm giá nhị nhỏ nhất của </i>T 1 1
BD CD


= + <i> như: </i>


•<i> Ta có </i>DH EK <i> nên </i>EH.BC EK.BC S<sub>DBC</sub> S<sub>EBC</sub><i>, điều này dẫn đến </i>


(

0

)

(

0

)



DB.DC.sin 180 −BDC EB.EC.sin 180 −BEC


<i>Mà ta có </i>BEC=BDC<i> và </i>EB EC= <i> nên ta được </i><sub>BD.CD BE</sub> 2


•<i> Kéo dài BD một đoạn </i>DG DC= <i>, ta được </i>


BD DC BD DG BE EG BE EC 2EB+ = +  + = + =
<i>Do đó ta được </i> 1 1 4 2


BD+DC BD CD+ = BE



•<i> Áp dụng định lí Ptoleme cho tứ giác nội tiếp BFCD ta được </i>


(

)



BD.CF CD.BF+ =BC.DF DB CD BF BC.2R+ 
<i>Từ đó suy ra </i> 1 1 4 2BF


BD+DC BD CD+ =R.BC<i>. </i>


<b>Bài 73. </b>Cho đường tròn (O; R) và một điểm I nằm bên trong đường tròn. Gọi AC và
BD là hai dây cung bất kì đi qua I. Xác định vị trí của AC và BD để AB.AD BC.CD


AB.BC DA.CD
+


+


đạt giá trị lớn nhất và giá trị nhỏ nhất.


</div>
<span class='text_page_counter'>(98)</span><div class='page_container' data-page=98>

Xét hai tam giác IDC và IAB có DIC=AIB và IDC IAB=


nên ta được IDC∽ IAB. Từ đó ta được ID IC CD


IA= IB =AD


Chứng minh tương tự ta được IAD∽ IBC nên


IA ID AD



IB = IC = BC Từ đó


ID ID IA AD.DC


.


IB =IA IB = AB.BC . Suy ra ta


được ID IB AB.BC DA.CD


IB AB.BC


+ <sub>=</sub> +


hay
AB.BC DA.CD


BD .IB


AB.BC
+
=


O


D


C
B
A



Mặt khác ta lại có IC IC IA: BC.CD
IA= IB IB = AB.DA
Suy ra IC IA AB.DA BC.CD


IA AB.DA


+ <sub>=</sub> +


hay AC AB.DA BC.CD.AI
AB.DA


+
=


Từ các kết quả trên ta được


AB.DA BC.CD
.AI


AC <sub>AB.DA</sub>


AB.BC DA.CD
BD


.IB
AB.BC


+
=



+ .


Chú ý là IA AD


IB = BC ta thu được


AB.AD BC.CD AC


AB.BC DA.CD BD


+ <sub>=</sub>


+ . Đến đây ta được


+ AB.AD BC.CD
AB.BC DA.CD


+


+ đạt giá trị lớn nhất khi và chỉ khi AC lớn nhất đồng thời BD nhỏ


nhất, điều này tương đương với AC đi qua O và BD vng góc với OI


+ AB.AD BC.CD
AB.BC DA.CD


+


+ đạt giá trị nhỏ nhất khi và chỉ khi AC nhỏ nhất đồng thời BD lớn



nhất nhất, điều này tương đương với BD đi qua O và AC vng góc với OI.


<b>Bài 74.</b>Cho đường trịn

( )

O và điểm P cố định nằm trong đường trịn

( )

O (P khơng


trùng với O). Hai dây cung AC và BD thay đổi của đường trịn

( )

O vng góc với
nhau tại P. Tìm vị trí của các dây cung AC và BD sao cho diện tích tứ giác ABCD có


giá trị lớnnhất, nhỏ nhất.


</div>
<span class='text_page_counter'>(99)</span><div class='page_container' data-page=99>

Gọi E, F, G, H theo thứ tự là trung điểm AB, AD,
CD, CB. Khi đó dễ dàng chứng minh được tứ giác
EFGH là hình chứ nhật có tâm là trung điểm của
đoạn OP.


Ta cần chứng minh AB2+CD2 =AD2+BC2=4R2.


Thật vậy do AC vng góc với BC nên dễ dàng suy


ra OEA = DGO, do đó suy ra OG EA AB
2
= = .


H G


F
E


N
M



P
K


I
O


D


C
B


A


Ta có


2 2


2 2 2


AB CD


OG DG R


2 2


   


+ = + =



   


    nên ta được


2 2 2


AB +CD =4R . Hoàn toàn


tương tự ta cũng chứng minh được <sub>AD</sub>2+<sub>BC</sub>2=<sub>4R</sub>2<sub>. </sub><sub>Hạ OK vng góc với AC và OI </sub>


vng góc với BD. Khi đó ta có <sub>ID</sub>2 =<sub>R</sub>2−<sub>OI</sub>2<sub>. </sub><sub>Do đó ta được </sub><sub>BD</sub>2 =<sub>4 R</sub>

(

2−<sub>OI</sub>2

)

<sub>. </sub>


Chứng minh tương tự ta cũng có <sub>AC</sub>2 =<sub>4 R</sub>

(

2 −<sub>OK</sub>2

)



Từ đó suy ra AC2+BD2 =4R2−4 OI

(

2 +OK2

)

=8R2−4OP2 không đổi
Từ đây ta được <sub>EF</sub>2 <sub>FG</sub>2 <sub>EG</sub>2 <sub>EG</sub>2 1

(

<sub>AC</sub>2 <sub>BD</sub>2

)



4


+ =  = + khơng đổi.


Suy ra hình chữ nhật EFGH là hình chữ nhật thay đổi trên một đường tròn cố định có
tâm là trung điểm của OP. Để ý là S<sub>EFGH</sub> 1S<sub>ABCD</sub>


2


= . Do đó ta có


(

) (

)




2 2


2 2 2 2


EFGH


EF GH 1 1


S EF.GH AC BD 8R 4OP


2 8 8


+


=  = + = −


Dấu bằng xẩy ra khi và chỉ khi tứ giác EFGH là hình vng nên AC BD= .


Lại có 2 2

(

)

2

(

2 2

)

(

)

2


EFGH


1 1 1


S EF.FG EF FG EF FG 8R 4OP EF FG


2 8 2


 



= = <sub></sub> + − − <sub></sub>= − − −


 


+ Để S<sub>EFGH</sub> đạt giá trị nhỏ nhất thì EF FG− phải đạt giá trị lớn nhất.


Mà ta có AC 2R FG R và BD MN với MN đi qua P.
Do đó EF 1MN


2


 nên EF FG R MN


2
−  −


</div>
<span class='text_page_counter'>(100)</span><div class='page_container' data-page=100>

Vậy S<sub>ABCD</sub> đạt giá trịlớn nhất khi AC BD= và S<sub>ABCD</sub> đạt giá trị nhỏ nhất khi


AC 2R= .


<b>Bài 75.</b>Cho đường tròn

( )

O và điểm P cố định nằm trong đường trịn

( )

O (P khơng


trùng với O). Hai dây cung AC và BD thay đổi của đường trịn

( )

O vng góc với
nhau tại P. Tìm vị trí của các dây cung AC và BD sao cho chu vi tứ giác ABCD có giá


trị lớn nhất, nhỏ nhất.


<b>Lời giải</b>


Chứng minh tương tự như trên ta được <sub>AB</sub>2+<sub>CD</sub>2+<sub>AD</sub>2+<sub>BC</sub>2 =<sub>4R</sub>2<sub>. </sub>



Đặt m AB BC CD DA= + + + . Khi đó ta được


(

)



2 2 2 2 2


m =AB +BC +CD +AD +2 AB.BC BC.CD CD.DA DA.AB AB.BC BC.AD+ + + + +
Mà ta có AB2+BC2+CD2+AD2 có giá trị khơng


đổi.Áp dụng định lí Ptoleme cho tứ giác AEOF nội
tiếpđường trịn ta có AE.OF OE.AF R.EF+ = hay ta


được AB.BC CD.DA R.BD


4 + 4 = 2 . Tương tự ta cũng


có AB.AD CD.BC R.AC


4 + 4 = 2 . Do đó ta được


(

)



2R BD AC+ =AB.BC CD.DA AB.AD CD.BC+ + +


H G


F
E



N
M


P
K


I
O


D


C
B


A


Như vậy việc tìm giá trị lớn nhất và nhỏ nhất của m tương đương với tìm giá trị lớn
nhất và nhỏ nhất của biểu thức S=AB.CD AD.BC 2R AC BD+ +

(

+

)

. Cũng theo định lí
Ptoleme thì trong tứ giác ABCD nội tiếp đường trịn ta có AB.CD AD.BC AC.BD+ = .


Do đó suy ra S=AC.BD 2R AC BD+

(

+

)

.


Ta có

(

)

(

)



2 2


2 2


AC BD



S AC.BD 2R AC BD 2R. 2 AC BD


2
+


= + +  + + không đổi


Dấu bằng xẩy ra khi và chỉ khi AC BD=


Lại có <sub>S</sub> 1 <sub>AC</sub>2 <sub>BD</sub>2

(

<sub>AC BD</sub>

)

2 <sub>2R. 2 AC</sub>

(

2 <sub>BD</sub>2

)

(

<sub>AC BD</sub>

)

2


2


 


= <sub></sub> + − − <sub></sub>+ + − −


 


Mà ta có AC 2R; BD MN   AC BD− 2R MN− .


Suy ra S 1 AC2 BD2

(

2R MN

)

2 2R. 2 AC

(

2 BD2

)

(

2R MN

)

2
2


 


 <sub></sub> + − − <sub></sub>+ + − −


</div>
<span class='text_page_counter'>(101)</span><div class='page_container' data-page=101>

Dấu bằng xẩy ra khi và chỉ khi AC 2R= .



Vậy m AB BC CD DA= + + + đạt giá trị lớn nhất khi AC BD= và đạt giá trị nhỏ nhất


khi AC 2R= .


<i><b>Nhận xét. </b>Ta có thể tìm giá trị nhỏ nhất của chu vi tứ giác ABCD theo cách khác như sau. </i>


Kẻ đường kính BE của

( )

O . Hai tam giác vng ABE
và PAD có ADB AEB= nên đồng dạng với nhau, do
đó ta được AB.AD BE.PA 2R.PA= = . Hoàn toàn tương
tự ta được BC.CD 2R.AC.= Từ đó ta được


(

)



AB.AD CB.CD+ =2R. PA PC+ =2R.AC


Từ giả thiết AC⊥BD suy ra AE CD; AD CE= = .


E


O
N


M
P


B


D


C


A


Ta đã có <sub>AB</sub>2+<sub>CD</sub>2=<sub>AB</sub>2+<sub>AE</sub>2=<sub>4R ; AD</sub>2 2+<sub>BC</sub>2 =<sub>CE</sub>2+<sub>BC</sub>2=<sub>4R</sub>2<sub>. </sub><sub>Gọi M, N lần </sub>


lượt là trung điểm của AC và BD, suy ra OM⊥AC; ON⊥BD. Từ đó suy ra


(

) (

)



(

)



2 2 2 2 2 2 2 2


2 2 2 2 2


AC BD 4AM 4BN 4 R OM 4 R ON


8R OM ON 8R OP


+ = + = − + −


= − + = −


Đặt d OP= . Ta có <sub>AC .BD</sub>2 2 =<sub>16 R</sub>

(

2−<sub>OM</sub>2

)(

<sub>R</sub>2−<sub>ON</sub>2

) (

=<sub>16 R</sub>4−<sub>R .d</sub>2 2 +<sub>OM .ON</sub>2 2

)



Từ p AB BC CD DA= + + + ta được


(

)

(

)



(

) (

)




2


2 2 2 2 2


2 2 2


p AB BC CD DA AB BC CD DA 2 AB.AD BC.CD


2 AB.BC AD.CD 2 AB.CD AD.BC


8R 2AC.BD 4R 8R 4d 2AC.BD


= + + + = + + + + +


+ + + +


= + + − +


Thay <sub>AC.BD 4 R</sub>= 4−<sub>R .d</sub>2 2+<sub>OM .ON</sub>2 2<sub>ta được </sub>


2 2 4 2 2 2 2 2 2 4 2 2 2 2


p =8R +8 R −R .d +OM .ON +4R 8R −4d +4 R −R .d +OM .ON


Như vậy <sub>p</sub>2 <sub>8R</sub>2+<sub>8 R</sub>4−<sub>R .d</sub>2 2 +<sub>4R 8R</sub>2−<sub>4d</sub>2 +<sub>4 R</sub>4−<sub>R .d</sub>2 2 =<sub>16 R</sub>

(

2+<sub>R R</sub>2−<sub>d</sub>2

)



Nên ta được 2 2 2


p R +R R −d , dấu bằng xẩy ra khi và chỉ khi OM.ON 0= tương
đương với AC hoặc BD là đường kính của đường trịn

( )

O .


</div>
<span class='text_page_counter'>(102)</span><div class='page_container' data-page=102>

<b>Bài 76.</b> Cho tam giác ABC có BC a,CA b,AB c= = = . Gọi r và r ,r ,r<sub>a</sub> <sub>b</sub> <sub>c</sub> lần lượt là bán
kính đường trong nội tiếp và bấn kính đường trong bàng tiếp các góc A, B, C của tam


giác ABC.


Chứng minh rằng


3 3 3


a b c


abc a b c


r  r + r + r


<b>Lời giải</b>


Với kí hiệu như hình bên ta có AK p a; AH p= − = ,


trong đó 2p a b c= + + . Ta có hai tam giác AKI và


AHQ đồng dạng với nhaunên ta được


a
a


p a rp


r IK AK



r


r HQ AH p p a




= = =  =




Hoàn toàn tương tự ta được r<sub>b</sub> pr ; r<sub>c</sub> pr


p b p c


= =


− −


Xét hiệu


3 3 3


a b c


abc a b c


T


r r r r



 


= −<sub></sub> + + <sub></sub>


  ta được


r<sub>a</sub>
r
K
Q
I
C
B
A

(

)

(

)

(

)


(

)

(

)

(

)



3 3 3


3 3 3


a b c


3 3 3


a p a b p b c p c


abc a b c abc



T


r r r r r rp rp rp


1


pabc a p a b p b c p c


rp
− − −
 
= −<sub></sub> + + <sub></sub>= − − −
 
 
= <sub></sub> − − − − − − <sub></sub>


Ta cần chứng minh T 0 pabc a p a− 3

(

− −

)

b p b3

(

)

−c p c3

(

− 

)

0.


Thật vậy, vì vai trị của a, b, c như nhau nên khơng mất tính tổng qt ta có thể giả sử


a b c 0   . Đặt P 2 pabc a p a= <sub></sub> − 3

(

− −

)

b p b3

(

)

−c p c3

(

)

<sub></sub>. Khi đó ta có


(

)

(

)

(

)

(

)



(

)

(

)

(

)



(

)

(

)

(

)

(

)

(

)

(

)



(

)(

)

(

)(

)

(

)(

)




(

)(

)

(

)

(

)

(

)(

)



3 3 3


2 2 2 3 3 3 3 3 3


2 3 2 3 2 3


2 2 2


2


2 2 2 2


P a b c abc a b c a b c a b c a b c


a bc b ca c ab a b a c a b c b a b c a c b c


a b c a a c a b c a b b a b c a b c c b c


a c a b a b a b c b c b c a c


c b b a a b b a b c b c a c


= + + − + − − + − − + −


= + + − − − − − − − − −


= − − − + − − − + − − −



= − − + − − + − −


= − − − + − + − −


Dễ thấy với a b c  thì P 0 nên T 0


Từ đó suy ra


3 3 3


a b c


abc a b c


</div>
<span class='text_page_counter'>(103)</span><div class='page_container' data-page=103>

<b>Bài 77.</b> Cho tam giác ABC nội tiếp đường trịn

( )

O và ngoại tiếp đường trịn có bán
kính r. Gọi O ,R ;<sub>1</sub> <sub>1</sub> O ,R ; O ,R<sub>2</sub> <sub>2</sub> <sub>3</sub> <sub>3</sub> theo thứ tự là tâm và bán kính của đường trịn tiếp
xúc ngồi với đường tròn (O) đồng thời tiếp xúc với AB, AC; BC, BA; CA, CB tương
ứng. Chứng minh rằng R<sub>1</sub>+R<sub>2</sub>+R<sub>3</sub>12r


<b>Lời giải</b>


Giả sử đường tròn

(

O ; R<sub>1</sub> <sub>1</sub>

)

tiếp xúc ngoài với
đường tròn

(

O; R

)

tại D và tiếp xúc với hai tia AB,
AC lần lượt tại M, N. Giả sử tia AD cắt đường tròn


(

O ; R1 1

)

tại điểm thứ hai là E. Khi đó ta suy ra


được OO<sub>1</sub> =OD O D R R+ <sub>1</sub> = + <sub>1</sub>. Lại do OA song


song với EO<sub>1</sub> và <sub>AM</sub>2=<sub>AN</sub>2=<sub>AD.AE</sub><sub>. </sub><sub>Từ đó </sub><sub>suy </sub>



ra


2
2


1


AD AD OD


AE OO


AM = = hay


2
2


1


AD R


R R


AM = + l E


I
L
K
N
M


O<sub>1</sub>
P
O
D
C
B
A


Chứng minh tương tự ta cũng được


2 2


2 2


1


BD CD R


R R


BM = CN = + .


Từ đó ta thu được AD BD CD


AM= BM=CN. Mặt khác tứ giác ABDC nội tiếp nên theo định lí
Ptoleme ta có AB.CD AC.BD AD.BC+ = . Do vậy ta được AB.CN AC.BM BC.AM+ = .


Đặt BC a; CA b; AB c= = = ta được BM AM c; CN AN b AM b= − = − = − . Khi đó từ hệ
thức trên ta được AM AN 2bc



b c a
= =


+ − . Gọi L là tiếp điểm của AB với đường tròn

( )

I


nội tiếp tam giác ABC.Khi đó O M1 AM


IL = AL hay


1


R AM


r = AL . Thay

(

)



1


AL b c a


2


= + − và
2bc


AM


b c a
=


+ − vào tỉ lệ thức trên ta được 1

<sub>(</sub>

<sub>)</sub>

2


R 4bc


r = <sub>b c a</sub><sub>+ −</sub> . Chứng minh tương tự ta


được


(

)

3

(

)



2


2 2


R


R 4ac 4ab


;


r = <sub>c a b</sub><sub>+ −</sub> r = <sub>a b c</sub><sub>+ −</sub>


Từ đó ta được


(

) (

) (

)



1 2 3


2 2 2


R R R 4bc 4ac 4ab



r <sub>b c a</sub> <sub>c a b</sub> <sub>a b c</sub>


+ +


= + +


</div>
<span class='text_page_counter'>(104)</span><div class='page_container' data-page=104>

Theo bất đẳng thức AM – GM ta có


(

) (

) (

)

(

) (

) (

)



2 2 2
3


2 2 2 2 2 2


4bc 4ac 4ab a b c


12


b c a c a b a b c b c a c a b a b c


+ + 


+ − + − + − + − + − + −


Chú ý là

(

a b c b c a c a b+ −

)(

+ −

)(

+ −

)

abc, do đó ta được R1 R2 R3 <sub>12</sub>


r
+ +





Hay ta được R<sub>1</sub>+R<sub>2</sub>+R<sub>3</sub> 12r, dấu bằng xẩy ra khi và chỉ khi R<sub>1</sub> =R<sub>2</sub> =R<sub>3</sub> và
a b c= = hay tam giác ABC đều.


<b>Bài 78.</b>Cho tam giác ABC khơng cân có AD và BE là đường phân giác. Chứng minh
rằng góc nhọn tạo bởi hai đường thẳng AB và DE không vượt qua


A B
3


.


<b>Lời giải</b>
Khơng mất tính tổng qt ta có thể giả sử trong


tam giác ABC có AB. Gọi M là giao điểm của
AB và DE, khi đó góc nhọn tạo bởi hai đường
thẳng AB và DE chính là góc BMD.


Ta cần chứng minh được BMD 1

(

BAC ABC

)


2


 − .


Thật vậy, áp dụng định lí Menelaus cho tam giác
ABC với ba điểm M, D, E thẳng hàng ta có



M


E


D C


B


A


DB EC MA MA DC EA


. . 1 .


DC EA MB =  MB = DB EC


Để ý là AD và BE là các đường phân giác của tam giác ABC nên DC AC EA; BA
DB = AB EC = BC


Từ đó ta được MA AC AB. CA


MB = AB BC = BC . Từ đó suy ra CM chính là đường phân giác


ngoài tại đỉnh C của tam giác ABC. Do đó 0 1

(

)



BMC 180 MCB MBC BAC ABC


2


= − − = −



Do đó suy ra BMD CMD 1

(

BAC ABC

)


2


+ = − .


Giả sử BMD 1

(

BAC ABC

)


2


 − , khi đó rõ ràng ta có


(

)

(

)



1 1 1


CMD BAC ABC BMD BAC ABC BMD


2 6 3


</div>
<span class='text_page_counter'>(105)</span><div class='page_container' data-page=105>

Từ đó ta được sin BMD sin 2CMD 2 sin CMD.cosCMD 2 cos CMD


sin CMD sin CMD sin CMD


 = = .


Mặt khác áp dụng định lí sin cho các tam giác BMD và CMD ta được


BD sin ABC


sin BMD <sub>MD</sub> BD sin ABC



sin CMD CD sin MCD CD sin MCD
MD


= =


Lai có DB AB sin ACB


CD=AC=<sub>sin ABC</sub> nên ta được


sin BMD sin ACB sin ABC ACB


. 2 sin


2
sin CMD=sin ABC sin MCD=


Từ đó ta suy ra <sub>sin</sub>ACB <sub>cos CMD sin 90</sub>

(

0 <sub>CMD</sub>

)



2  = −


Nên ta được ACB <sub>90</sub>0 <sub>CMD</sub> <sub>CMD 90</sub>0 ACB 1

(

<sub>BAC ABC</sub>

)



2  −   − 2 =2 + , điều này vơ lí.


Do đó điều ta giả sử là sai. Nên ta được BMD 1

(

BAC ABC

)


2


 − .



Vậy góc nhọn tạo bởi hai đường thẳng AB và DE không vượt qua


A B
3


.


<b>Bài 79. </b>Cho tam giác ABC và đường tròn

( )

O nội tiếp tam giác ABC tiếp xúc với AB,
BC, CA lần lượt tại E, D, F. Đường trong bàng tiếp tâm Q tiếp xúc với BC, AB, AC lần
lượt tại K, H, P. Đường thẳng EF cắt các tia BO và CO lần lượt tại M và N. Đường
thẳng HP cắt các tia BQ và CQ lần lượt tại R và S. Chứng minh rằng DMN= KRS


và SP SR RH


AB= BC=CA.


<b>Lời giải</b>


Xét trường hợp điểm M nằm ngoài và điểm N nằm trong tam giác ABC (Các trường
hợp còn lại tương tự). Ta có ONC=BMC 90= 0. Gọi Q là giao điểm hai tia phân giác


</div>
<span class='text_page_counter'>(106)</span><div class='page_container' data-page=106>

Khi đó Q là tâm đường tròn bàng tiếp ứng với


góc BAC của tam giác ABC. Đường tròn

( )

Q


tiếp xúc với AB, BC, CA lần lượt tại H, K, P. Gọi
R và S lần lượt là giao điểm của HP với OB và
OC. Khi đó dễ thấy CPR CQB 1HAP



2


= = nên tứ


giác CRQP nội tiếp đường tròn nên ta được


0


CRQ CPQ 90= = . Lại có MBQ 90= 0 nên tứ giác
MBRC là hình chữ nhật.


K
O


Q


P
S


R
H


I
N


M
F


E



D C


B
A


Gọi I là trung điểm của BC. Ta có M và R đối xứng với nhau qua I. Tương tự thì N và
S đối xứng với nhau qua S. Từ tính chất của tiếp tuyến với đường tròn ta được


(

)



BH=AH AB p− = − AE BE+ =CD trong đó p là nửa chu vi của tam giác ABC. Từ đó


suy ra BD CK= . Do đó ta được ID IK= hay K và F đối xứng với nhau qua I. Từ đó
các tứ giác MDRK, NDSK, MNRS là các hình bình hành. Do đó ta được


DMN KRS


 = 


+ Lại thấy BKR=BHR=AHP=APH CPS= nên tứ giác RKCP nội tiếp đường tròn.
Do đó ta được KRS=ACB. Hồn tồn tương tự ta được KSR=ABC. Từ đó suy ra hai


tam giác KRS và ACB đồng dạng nên ta được KS SR KR


AB= BC= AC. Mà ta lại có KS PS= và
KR=HR nên suy ra PS SR RH


AB= BC=CA.


<b>Bài 80. </b>Cho đường trịn

(

O; R

)

nội tiếp hình thang ABCD (AB//CD). Gọi E, F, G, H

theo thứ tự là tiếp điểm của đường tròn

(

O; R

)

với các cạnh AB, BC, CD, DA. Trên


cạnh CD lấy điểm M nằm giữa hai điểmD và C sao cho chân đường vng góc kẻ từ
M đến DO là điểm K nằm ngồi đường trịn

(

O; R

)

. Đường thẳng HK cắt đường tròn


(

O; R

)

tại điểm thứ hai là T.Tính tỷ số EB


EA khi biết


4R


AB ;BC 3R


3


= = và chứng minh


rằng MT MG= .


</div>
<span class='text_page_counter'>(107)</span><div class='page_container' data-page=107>

+ Do tứ giác ABCD là một hình thang nên ta


có 0


CDA DAB 180+ = . Do DO, AO theo thứ
tự là đường phân giác của các góc CDA và


DAB. Do đó ta được 0


ODA OAD 90+ = , từ
đó ta suy ra được AOD 90= 0 nên ta giác



AOD vng tại O.


O


M
K


T
H


G


F
E


D C


B
A


Hồn tồn tương tự tam giác BOC cũng vuông tại O. Nên theo hệ thức lượng trong
tam giác vuông ta được <sub>HA.HD FB.FC R</sub>= = 2<sub>. </sub>


Mặt khác ta có HA EA; BE FB; CF CG; DG HD= = = = nên EA.GD EB.GC= , từ đó ta lại
suy ra được EB GD


EA= GC. Đặt


EB GD



k


EA = GC= . Ta có OE vng góc với AB, OG vng góc


với CD nên ba điểm E, O, G thẳng hàng. Từ đó ta được EG 2R= và


(

)

2


2 2


ED =BC − GC EB− .


Ta lại có EB k EB kAB

<sub>(</sub>

4kR

<sub>)</sub>



EB EA+ =k 1+  = k 1+ =3 k 1+ nên

(

)



4kR
BF


3 k 1
=


+


Từ đó ta được


(

4kR

)



CF BC BF 3R



3 k 1


= − = −


+ nên

(

)



4kR


CG 3R


3 k 1


= −


+


Thay các kết quả trên vào ED2 =BC2−

(

GC EB−

)

2 ta được


(

)

(

<sub>(</sub>

)

<sub>)</sub>



2 2


2 2


2


k 9


4kR 9 6 5



4R 9R 3R 5 k


11


3 k 1 <sub>9 k 1</sub>


  + <sub>− +</sub>


= − −   =  =


+ <sub>+</sub>


 


 


Vậy EB 9 6 5


EA 11


− +


= .


+ Ta có DH DG= nên tam giác DGH cân tại D. Mà OD là đường phân giác của góc


HDG nên suy GH vng góc với DO. Lại có MK vng góc với DO nên MK song


song với HG. Từ đó ta suy ra được KMG HGC= , mà GC là tiếp tuyến của đường trịn



( )

O và góc HTG là góc nội tiếp của đường trịn

( )

O . Do đó ta được HTG HGC= suy
ra KMG=HTG nên tứ giác KTGM nội tiếp đường trịn. Lại có OKM OGM 90= = 0


</div>
<span class='text_page_counter'>(108)</span><div class='page_container' data-page=108>

K, G, M, T thuộc đường trịn đường kính MO. Do đó MIO 90= 0 nên OT vng góc


với MT. Do đó MT là tiếp tuyến của đường tròn

( )

O . Ta có MT, MG là tiếp tuyến của
đường trịn

( )

O nên MT MG= .


<b>Bài 81.</b> Cho tam giác ABC có B, C cố định và điểm A thay đổi sao cho tam giác ABC
nhọn không cân. Gọi D là trung điểm của BC và E, F tương ứng là hình chiếu của D


trên AC, AB. Gọi O là tâm đường tròn ngoại tiếp tam giác ABC. Đường thẳng EF cắt
AO và BC theo thứ tự tại M và N. Các tiếp tuyếntại E, F của đường tròn ngoại tiếp
tam giác AEF cắt nhau tại T. Chứng minh rằng đường tròn ngoại tiếp tam giác AMN
luôn đi qua một điểm cố định và T luôn thuộc một đường thẳng cố định.


<i><b>Lời giải</b></i>


G
A


B D C


E
F


O


M



N


K


P


Q


T


I J


<i><b>+ </b><b>Chứng minh đường tròn ngoại tiếp tam giác AMN luôn đi qua</b><b>một điểm cố định. </b></i>


Giả sử tam giác ABC có AB AC , các trường hợp cịn lại chứng minh hồn tồn
tương tự. Khi đó điểm N nằm trên tia đối của tia BC. Tứ giác AEDF có


0


AED AFE 180+ = nên nội tiếp đường tròn đường kính AD, suy ra


MEA FEA FDA= = . Do O là tâm đường tròn ngoại tiếp tam giác ABC nên ta có


0


OAC 90= −ABC. Từ đó có biến đổi góc


0



</div>
<span class='text_page_counter'>(109)</span><div class='page_container' data-page=109>

Từ đó suy ra tứ giác AMDN nội tiếp đường tròn. Mà BC cố định nên điểm D cố định.
Vậy đường tròn ngoại tiếp tam giác AMN luôn đi qua điểm D cố định


<i><b>+ </b><b>Chứng minh T luôn thuộc một đường thẳng cố định.</b></i>


Gọi K là tâm đường trịn ngoại tiếp tam giác AEF, khi đó K là trung điểm của AD. Gọi
giao điểm của AE với DF là P và giao điểm của AF với DE là Q.Khi đó trong tam giác
APQ có hai đường cao là QE và PF cắt nhau tại D nên D là trực tâm. Do đó AD vng
góc với PQ tại G. Tam giác AED vuông tại E có EK là đường trung tuyến nên các tam
giác AKE và DKE cân. Mặt khác do TE là tiếp tuyến của đường tròn ngoại tiếp tam


giác AEF nên ta có TEQ TED EAD EQG= = = nên ET đi qua trung điểm của PQ, tương
tự thì FT đi qua trung điểm của PQ. Từ đó suy ra T là trung điểm của PQ. Dễ thấy tứ
giác PEFQ nội tiếp đường trịn đường kính PQ và có tâm là T. Gọi I, J là giao điểm của
BC với đường tròn

( )

T . Áp dụng định lý Menelaus cho tam giác ABC với bộ ba điểm
Q, D, E và P, D, F thẳng hàng ta có BF AP CD. . 1


AF CP BD = và


BQ AE CD


. . 1


AQ CE BD = .


Từ đó ta được


2
2



CD BF.AP.AE.BQ


. 1


AF.CP.AQ.CE


BD = hay


2
2


CD AF.CP.AQ.CE


BF.AP.AE.BQ


BD =


Mặt khác do tứ giác QFEP nội tiếp đường trịn nên ta có AF.AQ AE.AP= . Mặt khác
các tứ giác IEJP và JFIQ nội tiếp đường tròn

( )

T nên ta lại có CP.CE=CJ.CI và


BF.BQ=BJ.CJ. Từ đó ta suy ra được


2
2


CD CJ.CI


BJ.CJ


BD = . Mà do CD BD= nên ta có



(

)(

) (

)(

)

2 2 2 2


BI.BJ=CI.CJ ID DB ID DB− + = JD DC JD DC− + ID −BD =JD −CD


Mà lại do CD BD= nên suy ra ID=JD hay D là trung điểm của IJ. Từ đó dẫn đến TD


vng góc với BC tại D, suy ra T thuộc đường trung trực của BC. Mà BC cố định nên
đường trung trực của BC cố định. Vậy T luôn thuộc một đường thẳng cố định khi A
thay đổi.


</div>
<span class='text_page_counter'>(110)</span><div class='page_container' data-page=110>

KM KA; LN LA= = . Chứng minh rằng tâm đường tròn ngoại tiếp tam giác MND luôn
thuộc một đường thẳng cố định khi P di động trên đoạn thẳng AP.


<b>Lời giải</b>


I


L


K Q


P


N M


F
E


D



T


S


R


C
B


A


Qua K kẻ đường thẳng vng góc với CP cắt AD N. Khi đó P là trực tâm tam giác
RKC. Suy ra KP vng góc với RC. Từ đó ta được KCP=KRD. Chú ý rằng tam giác
AKC vng tại A có AD là đường cao nên ta có KM2 =KA2 =KD.KC nên KM tiếp
xúc với đường tròn ngoại tiếp tam giác DMC. Suy ra KMD MCD KCP= = . Từ đó ta
được KRD KMD= , nên bốn điểm K, D, M, R cùng nằm trên một đường trịn. Do đó ta


được 0


RMK=RDK=90 hay RM vng góc với MK.
Lại có PCD∽ KRD nên ta được PD CD


KD =RD. Hai tam giác KAC và LAB cùng vng


góc tại A nên ta được AD2 =KD.DC BD.LD= nên KD ND


BD = CD . Do đó


PD LD



CD =RD, nên


LDR PDB


 ∽  , suy ra LRD=DBP=LBN.


Mặt khác ta có <sub>LN</sub>2=<sub>LA</sub>2=<sub>LD.LB</sub> <sub>nên LN tiếp xúc với đường tròn ngoại tiếp tam </sub>


</div>
<span class='text_page_counter'>(111)</span><div class='page_container' data-page=111>

(

) (

)



(

) (

) (

) (

)



2 2 2 2 2 2 2 2


2 2 2 2 2 2 2 2


IN IM RN RM RK KM RL LN


RK RL AL AK AK AL AL AK 0


− = − = − − −


= − + − = − + − =


Từ đó ta được <sub>IM</sub>2 =<sub>IN</sub>2 <sub>hay ta được </sub><sub>IM IN</sub>= <sub>. </sub>


Giả sử đường tròn ngoại tiếp tam giác MND cắt BC tại Q. Ta sẽ chứng minh Q là điểm
cố định và khi đó tâm đường trong ngoại tiếp tam giác DMN thuộc đường trung trực
của đoạn thẳng AQ.



Thật vậy, gọi giao điểm của đường tròn ngoại tiếp tam giác DMN với IM, INlần lượt
là E, F khác M, N. Khi đó ta có EM FN= . Mà ta đã có <sub>KM</sub>2 =<sub>KA</sub>2 =<sub>KD.KC</sub><sub> nên </sub>


MCK KMD EQD= = nên suy ra EQ song song với PC. Tương tự ta cũng có FQ song
song với BP. Ta có biến đổi tỉ số sau


QK QK QC QB KE QC FN EK QC KE.KM LA QC


. . . .


QL QC QB QL EM QB FL FL QB KA LF.LN QB


LA KD.KQ QC QK LA.KD QC


. . . .


KA LD.LQ QB QL KA.LD QB


= = = =


= =


Từ đó LA.KD QC. 1


KA.LD QB = nên


LA.KD QB


KA.LD =QC. Dẫn đến



2 2 2


2 2 2


QB KD .LA KD LB


.
KC LD


QC =KA .LD = (1).


Vẽ đường phân giác AJ của tam giác ABC, khi đó dễ thấy KAJ LAJ= .


Khi đó ta có ABK
ACL


S BK


S = CL và


ABK
ACL


S AB.AK


S = AC.AL nên suy ra


BK AB.AK



CL =AC.AL.


Hoàn toàn tương tự ta cũng được BL AB.AL


CK =AC.AK. Do vậy ta được


2
2


AB BK.BL


CK.CL


AC = (2).


Mặt khác ta có DB.DL DP.DR DP.DR= = nên DK DB DK DB BK


DL DC DL DC CL




= = =


− (3).


Kết hợp các kết quả (1), (2), (3) ta được


2 2


2 2



QB AB


QC = AC hay


QB AB


QC=AC, suy ra AQ là


đường phân giác của tam giác ABC, hay điểm Q và J trùng nhau. Điều này có nghĩa Q
là điểm cố định. Từ đó theo suy luận ban đầu ta được tâm đường trịn ngoại tiếp tam
giác MND ln thuộc một đường thẳng cố định.


</div>
<span class='text_page_counter'>(112)</span><div class='page_container' data-page=112>

lượt là hình chiếu của H trên CB và CA. Gọi U, L, Q lần lượt tâm đường tròn bàng
tiếp đỉnh H của các tam giác AHE, BHF, HEF. Chứng minh rằng C là tâm đường tròn
nội tiếp tam giác QUL.


<b>Lời giải</b>


Q


J


T
Z


U Y


X



F


E


L


H O


C


B
A


Gọi giao điểm của LF, UE với HQ lần lượt là X, Y. Gọi Z là hình chiếu của Q trên HE
và T là hình chiếu của L trên BF. Để ý đến các tam giác vng cân HYE và HZQ ta có.


EY YH YH HE HE CA


EY QY+ = YH QY+ =QH= 2HZ =HE HF EF+ + = AB BC CA+ +


Lại có XH XF HF

<sub>(</sub>

HF

<sub>)</sub>

AC


XH LX+ =2XF LF+ = 2HF FT+ =2HF+ BF BH AB+ − = AB BC CA+ + .


Từ hai kết quả trên ta được EY XH


EY QY+ =XH LX+ hay ta được


EY XH



QY= XL . Chứng minh


hoàn toàn tương tự ta cũng thu được XF YH


XQ = YU. Chú ý rằng YE=YH và XF=XH


nên ta dễ dàng suy ra được 1 1


YQ.XQ = XL.YU hay YQ.XQ XL.YU= , điều này dẫn đến


hai tam giác vuông YQU và XLQ đồng dạng với nhau. Từ đó suy ra <sub>UQL 90</sub>= 0<sub>. Ta </sub>


thấy QU UY YH HE CA


</div>
<span class='text_page_counter'>(113)</span><div class='page_container' data-page=113>

nhau. Gọi J là tâm đường tròn nội tiếptam giác ABC. Ta cũng thấy hai tam giác HEU
và LFH đồng dạng với nhau nên ta có CE.CF HE.HF EU.FL= = . Mà ta có


0


CFL=UEC 135= nên hai tam giác CFL và UCE đồng dạng với nhau. Để ý rằng ta lại
có hai tam giác EUH và AJB đồng dạng với nhau. Vậy nên ta được


CU UE UE IA


CL = CF =EH = IB. Mặt khác ta có


0 0 0 0


UCL 90= +UCE LCF 90+ = +45 =135 =AJB



nên hai tam giác CUL và JAB đồng dạng với nhau. Suy ra C là tâm đường tròn nội
tiếp tam giác QUL.


<b>Bài 84.</b>Cho tứ giác ABCD khơng phải là hình thang nội tiếp đường trong tâm O. Hai


đường chéo AC và BD cắt nhau tại H. Gọi giao điểm của AB và CD, AD và BC lần
lượt là E, F. Chứng minh rằng H và hai trực tâm H ; H<sub>3</sub> <sub>4</sub> của hai tam giác FAB, FCD
cùng nằm trên một đường thẳng.


<b>Lời giải</b>
Chứng minh rằng H và hai trực tâm H ; H<sub>3</sub> <sub>4</sub> của
hai tam giác FAB, FCD cùng nằm trên một đường
thẳng. Vẽ hai đường cao CC ; DD<sub>1</sub> <sub>1</sub> cắt nhau tại
trực tâm H<sub>4</sub> của tam giác FCD. Giả sử đường cao
hạ từ A của tam giác FAB cắt HH<sub>4</sub> tại H’. tac cần
chứng minh BH’ vng góc với AF để suy ra H’ là
trực tâm của tam giác FAB hay H’ trùng với H3.


Thật vậy, gọi M là giao điểm của AC và DD<sub>1</sub>, N là


giao điểm của CC<sub>1</sub> và BD. Khi đó dễ thấy tứ giác


1 1


CDC D nội tiếp đường tròn. Suy ra ta được


1 1 1


AC D =DCD .



D


H<sub>4</sub>
H<sub>3</sub>


D<sub>1</sub>
C<sub>1</sub>


N


M
F


H


C
B
A


Mà ta lại có DCD<sub>1</sub>=FAB nên ta được AC D<sub>1</sub> <sub>1</sub> =FAB. Từ đó suy ta AB song song với


1 1


C D . Ta có NDC<sub>1</sub>=MCD<sub>1</sub> và C DH<sub>1</sub> <sub>4</sub> =D CH<sub>1</sub> <sub>4</sub> nên đặt NDC<sub>1</sub> =MCD<sub>1</sub>= và


1 4 1 4


</div>
<span class='text_page_counter'>(114)</span><div class='page_container' data-page=114>

1 1 4 1 1 4


DC N; DC H ; CD M; CD H ta được 1



1 4


C N tan


C H tan





= và 1


1 4


D M tan


D H tan





= . Do đó ta được


1 1


1 4 1 4


C N D M


C H = D H , suy ra MN song song với C D1 1. Mà ta có AB song song với C D1 1 nên



ta được AB song song với MN. Lại có AH’ song song với MH<sub>4</sub>. Áp dụng định lí


Talets ta có


4


HH' HA HA HB


;


HH =HM HM=HN nên ta được <sub>4</sub>


HH' HB


HH =HN. Từ đó theo định lí


Talets đảo ta có BH’ song song với NH4, do đó BH’ vng góc với FD. Điều này có


nghĩa là H’ là trùng với H<sub>3</sub> nên ta được H và hai trực tâm H ; H<sub>3</sub> <sub>4</sub> cùng nằm trên một
đường thẳng.


<b>Bài 85.</b>Cho tứ giác ABCD khơng phải là hình thang nội tiếp đường trong tâm O. Hai
đường chéo AC và BD cắt nhau tại H. Gọi giao điểm của AB và CD, AD và BC lần
lượt là E, F. Chứng minh rằng F và hai trực tâm H ; H<sub>1</sub> <sub>2</sub> của hai tam giác HAB, HCD
cùng nằm trên một đường thẳng.


<b>Lời giải</b>


Chứng minh rằng F và hai trực tâm H ; H<sub>1</sub> <sub>2</sub> của hai


tam giác HAB, HCD cùng nằm trên một đường
thẳng. Vẽ hai đường cao CC ; DD<sub>1</sub> <sub>1</sub> cắt nhau tại trực


tâm H<sub>2</sub> của tam giác HCD. Giả sử đường cao AA<sub>1</sub>


hạ từ A của tam giác HAB cắt FH2 tại H’. ta cần


chứng minh BH’ vng góc với AH để suy ra H’ là
trực tâm của tam giác HAB hay H’ trùng với H1.


Thật vậy, dễ thấy tứ giác CDC D<sub>1</sub> <sub>1</sub> nội tiếp đường


trịn nên ta có DCD<sub>1</sub>=BC D<sub>1</sub> <sub>1</sub>. Mà có DCD<sub>1</sub> =C BA<sub>1</sub>
nên BC D1 1=C BA1 , suy ra AB và C D1 1 song song


với nhau.


F


D


A


B


C
H


M
N



C<sub>1</sub>


D1
H2


H<sub>1</sub>


</div>
<span class='text_page_counter'>(115)</span><div class='page_container' data-page=115>

Đặt NDC<sub>1</sub>=MCD<sub>1</sub> =; C DH<sub>1</sub> <sub>2</sub> =D CH<sub>1</sub> <sub>2</sub> =. Khi đó áp dụng tỉ số lượng giác cho các


tam giác vuông ta được 1 1
1 2 1 2


C N D M tan


C H D H tan





= = . Do đó ta được MN song song với


1 1


C D , nên ta suy ra được MN và AB song song với nhau. Từ đó lập luận tương tự ý a
ta thu được H’ là trực tâm tam giác HAB hay hai điểm H’ và H<sub>1</sub> trùng nhau. Từ đó
suy ra F và hai trực tâm H ; H1 2 cùng nằm trên một đường thẳng.


<b>Bài 86. </b>Cho đường tròn

( )

O với dây BC không phải là đường kính. Gọi I là trung
điểm của BC và điểm A di động trên cung lớn BC. Gọi

( )

I1 là đường tròn qua I tiếp


xúc với AB tại B và

( )

I2 là đường tròn qua I tiếp xúc với AC tại C. Hai đường tròn


( )

I1 và

( )

I2 cắt nhau tại D khác I. Chứng minh rằng đường tròn ngoại tiếp tam giác


ADI đi qua một điểm cố định khác I.


<b>Lời giải</b>


Trước hết ta phát biểu khơng chứng minh bài tốn phụ.


<i><b>Bài tốn phụ 1. </b>Cho tam giác ABC </i>


<i>nội tiếp đường tròn </i>

( )

O <i> và có AM </i>
<i>là đường trung tuyến, AI là đường </i>
<i>phân giác. Tiếp tuyến của đường </i>
<i>tròn </i>

( )

O <i> tại B và C cắt nhau ở D. </i>
<i>Chứng minh rằng AD đối xứng với </i>
<i>đường thẳng AM qua qua đường </i>
<i>thẳng AI. </i>


<b>Bài toán phụ 2.</b><i>Cho tam giác ABC </i>
<i>có đường trung tuyến AI và đường </i>
<i>phân giác AD. Gọi M là điểm bất kì </i>
<i>trên cạnh BC. Chứng minh rằng </i>
<i>AM đối xứng với AI qua AD khi và </i>


M J


T


F


I<sub>2</sub>
I<sub>1</sub>


K


E


O


I


D


C
B


</div>
<span class='text_page_counter'>(116)</span><div class='page_container' data-page=116>

<i>chỉ khi </i>


2
2


MB AB


MC= AC <i>. </i>


<b>Trở lại bài toán.</b>Để ý rằng BA là tiếp tuyến tại B của đường

( )

I<sub>1</sub> và CA là tiếp tuyến
tại C của đường tròn

( )

I2 nên ta được



0


BDC=BDI CDI+ =ABC ACB 180+ = −BAC.


Từ đósuy ra tứ giác ABDC nội tiếp đường trịn, do đó ta có IDC=ACB=BDA . Điều
này dẫn đến đường thẳng DA đốixứng với đường thẳng đường trung tuyến DI qua


đường phân giác hạtừ D trong tam giác DBC. Từđógọi T là giao điểm hai tiếptuyến
tại B, C củađường tròn

( )

O và J là giao điểmcủa AD với BC thì theo bài tốn phụ 1
thì DA đi qua điểm T và theo bài tốn phụ 2 ta có


2
2


DB BJ


CJ


DC = . Do vậy ta có tam giác
TBD đồng dạng với tam giác TAB và tam giác TCD đồng dạng với tam giác TAC, do


đó suy ra BD TB
BA =TA và


CD TC


CA=TA. Chú ý rằng ta có TB TC= nên ta suy ra


BD CD



BA =CA


hay DB AB


DC =AC. Đếnđây ta lại có


2
2


AB BJ


CJ


AC = nên theo bài tốn phụ hai ta lại có AD đối


xứngvớiđường trung tuyến AI qua đường phân giác hạtừ A trong tam giác ABC.


Điều này dẫn đến BID IDC ICD ACB BAD ACB IAC= + = + = + =BIA nên suy ra IB là


đường phân giác của góc AID . Từ đó suy ra IO là phân giác ngồi góc AID. Gọi M là
giao điểm thứ gai của IB với đường trong ngoại tiếp tam giác AID, khi đó M nằm trên
đường trung trực của đoạn thẳng AD. Mà ta có điểm O thuộc trung trựccủa đoạn
thẳng AD. Kếthợpvới IO là phân giác ngồi của góc AID ta suy ra được O nằm trên
đường tròn ngoại tiếp tam giác AID.Do vậy đường tròn ngoại tiếp tam giác AID luôn
đi qua điểm O cố định.


<b>Bài 87. </b>Cho hai đường tròn

( )

O<sub>1</sub> và

( )

O<sub>2</sub> tiếp xúc ngoài với nhau tại T. Một đường
thẳng cắt đường tròn

( )

O<sub>1</sub> tại hai điểm phân biệt A, B và tiếp xúc với đường tròn


( )

O2 tại X (B nằm giữa A và X). Đường thẳng XT cắt đường tròn

( )

O1 tại S khác T và


</div>
<span class='text_page_counter'>(117)</span><div class='page_container' data-page=117>

đoạn thẳng CY và ST không cắt nhau. Gọi I là giao điểm của đường thẳng XY và SC.
Chứng minh rằng SA SI=


<b>Lời giải</b>


I
O<sub>2</sub>
O<sub>1</sub>


S


T


Y
X


C
B


A


Do hai đường tròn

( )

O<sub>1</sub> và

( )

O<sub>2</sub> tiếp xúc ngoài với nhau tại T và hai đường thẳng


1 2


O O , SX cắt nhau tại T nên ta có TO S<sub>1</sub> =TO X<sub>2</sub> .Từ đó suy ra TAS TYX= =TYI. Mặt
khác do do tứ giác ABTS nội tiếp đường tròn TAS=TCI nên suy ra TCI TYI= , do đó
tứ giác CTIY nội tiếp đường trịn. Ta có O ST1 =O TS1 =XTO2 =TXO2 nên O S1 song



song với O X<sub>2</sub> . Mặt khác ta có O X<sub>2</sub> vng góc với AB nên O S<sub>1</sub> vng góc với BA.
Điều này dẫn đến O S<sub>1</sub> đi qua trung điểm của AB nên tam giác SAB cân tại S, suy ra


SA SB= . Mặt khác chú ý đến tứ giác CTIY nội tiếp đường tròn ta có


TIS TIC TYC TXI= = = , mà ta lại có TSI=XSI nên suy ra tam giác STI và tam giác SIX


đồng dạng với nhau, do đó ta được <sub>SI</sub>2=<sub>ST.SX</sub><sub>. </sub> <sub>Để </sub> <sub>ý </sub> <sub>ta </sub> <sub>có </sub>


TBS TAS TCI= = =TYI=BXS, mà ta lại có BST=BSX nên tam giác STB đồng dạng
với tam giác SBX , do đó <sub>SB</sub>2 =<sub>ST.SX</sub><sub>. Từ các kết quả trên ta được </sub> <sub>SA</sub>2=<sub>SI</sub>2<sub> hay </sub>


SA SI= .


<b>Bài 88.</b>Cho tam giác ABC nhọn nội tiếp đường tròn

( )

O và ngoại tiếp đường tròn

( )

I .


</div>
<span class='text_page_counter'>(118)</span><div class='page_container' data-page=118>

điểmthứ hai của ID với đường tròn ngoại tiếp tam giác ODH. Chứng minh rằng ba
điểm A, H, K thẳng hàng.


<b>Lời giải</b>
Vẽ đường kính PQ của đường trịn


( )

O và gọi giao điểm của AI và BC
là F. Do I là tâm đường tròn nội tiếp


tam giác ABC và P là điểm chính
giữa cung nhỏ BC của đường tròn


( )

O nên AI đi qua điểm P. Ta có



IBP=IBA CAP+ =IBA IAB+ =BIP


nên tam giác PBI cân tại P, do đó P
là tâm đường tròn ngoại tiếp tam


giác BIC.


N
P


O
Q
H T


D
F


J
K


M


I


C
B


A


Điều này dẫn đến MI vng góc với IP. Do J là điểm đối xứng với I qua O nên tứ giác


PIQJ là hình bình hành. Do đó IQ song song với PI nên ta lại suy ra được MI vng
góc với QJ. Để ý ta thấy IMF APQ= nên hai tam giác vuông MIF và PAQ đồng dạng
với nhau nên ta có IF IM


AQ =AP hay ta được IF.AP IM.AQ= . Lại thấy BI là phân giác


của tam giác ABF nên ta có FI BF


AI =BA. Mặt khác do FBP=CAP=BAP nên tam giác


BPF và tam giác APB đồng dạng với nhau, suy ra BF PB PI


BA= PA=PA. Từ đó ta có


FI PI


AI =PA hay FI.PA IA.IP= . Kết hợp hai kết quả lại ta thu được IM.AQ IA.IP= hay


AQ PI


AI =MI nên hai tam giác vuông IAQ và MIP đồng dạng. Điều này dẫn đến
MPI AQI= . Chú ý đến tứ giác PIQJ là hình bình hành thì IPJ IQJ= và AQJ 90= 0.
Trong tam giác MPJ có <sub>MPJ MPI IPJ AOI IQJ AQJ 90</sub>= + = + = = 0<sub> nên tam giác MPJ </sub>


</div>
<span class='text_page_counter'>(119)</span><div class='page_container' data-page=119>

Gọi giao điểm thứ hai của MA với đường tròn

( )

O là T. Do MI là tiếp tuyến của
đường tròn ngoại tiếp tam giác BIC nên ta có <sub>MI</sub>2 =<sub>MB.MC</sub><sub>. Mà ta lại có tứ giác </sub>


ATBC nội tiếp đường tròn

( )

O nên ta lại có MB.MC MT.MA= . Do đó suy ra


2



MI =MT.MA, mà tam giác MIA vuông tại I nên IT trở thành đường cao hay IT
vng góc với AI. Gọi N là giao điểm thứ hai của TI với đường trịn

( )

O , khi đó AN
là đường kính cỉa đường trịn, mà ta đã có <sub>AQJ 90</sub>= 0 <sub>nên ba điểm Q, J, I thẳng hàng. </sub>


Do đó AP song song với QA và AQ song song với PN. Điều này dẫn đến


ITP NTP ANQ ATQ= = = , mà AQT APT= nên tam giác AQT và tam giác PIT đồng
dạng với nhau. Từ đó suy ra QA PI QJ


TA =TI = TI nên ta lại được hai tam giác vuông AQJ


và ATI đồng dạng với nhau. Từ đó suy ra AJQ AIT AMI= = hay JAI IMA= nên MA


vng góc với AJ Từ đó dẫn đến các điểm A, J, P, H, M cùng nằm trên đường trịn
đường kính AJ. Do vậy ta suy ra đươc KHO KDO APJ= = =AHJ nên ba điểm H, A, K
thẳng hàng.


<b>Bài 89.</b>Cho tứ giác lồi ABCDnội tiếp đường trịn tâm Ovà có các cặp cạnh đối không
song song. Gọi M, Ntương ứng là giao điểm của các đường thẳng AB và CD, AD và


BC. Gọi P, Q, S, T tương ứng là giao điểm các đường phân giác trong của các cặp góc


MAN và MBN, MBN và MCN, MCN và MDN, MDN và MAN. Giả sử bốn điểm


P, Q, S, T đôi một phân biệt.Gọi Ilà tâm của đường tròn đi qua bốn điểm P, Q, S. Gọi


E là giao điểm của các đường chéo ACvà BD. Chứng minh rằng ba điểm E, O, Ithẳng


hàng.



</div>
<span class='text_page_counter'>(120)</span><div class='page_container' data-page=120>

I


E


P


Q S
T


O


N
M


D


C


B


A


Gọi A, B, C, D là các góc trong của tứ giác ABCD. Ta giả sử MN nằm cùng phía với B
đối với đường thẳng AC như hình vẽ (Các trường hợp khác chứng minh tương tự).


Xét tam giác ABP ta có


(

)




( )



0 0


0 0


1 1


TPQ APB 180 PAB PBA 180 MAN MBN NBA


2 2


1 1 1


180 C B C 90 C D


2 2 2


 


= = − + = −<sub></sub> + + <sub></sub>


 


 


= −<sub></sub> + + <sub></sub>= − +


 



Xét tam giác CDS ta có


(

)

( )



0 0 1 1 0 1


QST CSD 180 SDC SCD 180 D C A 90 C D


2 2 2


 


= = − + = −<sub></sub> + + <sub></sub>= − +


 


So sánh hai đẳng thức trên, ta được TPQ QST= hay tứ giác PQTS nội tiếp.
Ở phần này, vấn đề khó hơn hẳn và


việc xác định ví trị của các điểm E, I
khiến ta khó định hướng hơn trong
việc chứng minh các điểm thẳng hàng.


Tuy nhiên, ta chú ý rằng nếu E là giao
của hai đường chéo tứ giác ABCD và
các điểm M, N thì OE vng góc với


MN.


K


A
B
C


D


M


N
O


</div>
<span class='text_page_counter'>(121)</span><div class='page_container' data-page=121>

Ta sẽ chứng minh điều đó qua bổ đề: Cho tứ giác ABCD nội tiếp đường tròn

( )

O có


M, N lần lượt là giao điểm của các cặp cạnh đối AB, CD và AD, BC. Gọi E là giao điểm
của hai đường chéo. Khi đó ta có OE vng góc với MN.


Thật vậy, gọi K là giao điểm khác E của đường tròn ngoại tiếp tam giác ABE, CDE. Do
tứ giác ABCD nội tiếp đường tròn nên ACD 1AOD


2


= và ABD 1AOD
2


= , do đó suy ra


ACD ABD+ =AOD. Ta có DKE 180= 0−DCE 180= 0−DCA và AKE 180= 0−DBA.


Từ đó kết hợp các kết quả trên ta được DKA=3600−DKE AKE− =DOA nên tứ giác



DOKA nội tiếp đường tròn. Mặt khác ta cũng có


BKC=BKE CKE EAB EDC+ = + =BOC nên tứ giác OKBC nội tiếp đường trịn. Ta có
K và E là giao điểm của hai đường tròn ngoại tiếp tam giác AEB và CDE, A và B là hai
giao điểm của đường tròn

( )

O và đường tròn ngoại tiếp tam giác ABE, C và D là là


hai giao điểm của đường tròn

( )

O và đường trịn ngoại tiếp tam giác CDE. Từ đó theo
mộ bài tốn phụ quen thuộc ta có ba điểm K, E, M thẳng hàng. Hai tứ giác DOKA và
OKBC nội tiếp đường trịn nên suy ra K cũng chính là giao điểm thứ hai khác O của
hai đường tròn ngoại tiếp các tam giác OBC và OAD. Từ đó suy luận tương tự ta cũn
được các điểm O, K, N thẳng hàng. Mặt khác cũng bằng cách xét các góc nội tiếp


trong các tứ giác nội tiếp ta có


MKN MKB NKB EAB OCB EDC OBC EKC OKC MKO= + = + = + = + = . Hơn nữa đây
là hai góc bù nhau nên mỗi góc bằng 90 0 hay ME vng góc với ON. Chứng minh
tương tự ta có NE vng góc với OM. Từ đó suy ra E là trực tâm của tam giác OMN.
Từ đó ta được OE vng góc với MN. Bổ đề được chứng minh.


Đến đây, chỉ cần chứng minh được thêm OI vng góc với MN thì bài tốn
hồn tất do có các điểm O, I, E cùng nằm trên một đường thẳng vng góc với MN
Do O, I lần lượt là các tâm đường tròn ngoại tiếp các tứ giác ABCD và PQTS nên một
ý tưởng tự nhiên là ta sẽ chứng minh MN chính là trục đẳng phương của hai đường


</div>
<span class='text_page_counter'>(122)</span><div class='page_container' data-page=122>

Tuy nhiên, thực hiện điều này khơng q khó! Theo các xác định các điểm thì
Q, T lần lượt là tâm đường tròn bàng tiếp của các tam giác BCM và ADM nên chúng
phải cùng nằm trên phân giác ngồi của góc AMB hay M, Q, T thẳng hàng. Hơn nữa,
cũng do các tâm đường tròn bàng tiếp nên


0 BCM 0 BAD



MQB 90 90 BAT


2 2


= − = − =


Từ đó ta được tứ giác ABQT nội tiếp đường tròn. Suy ra MA.MB MQ.MT=


hay M có cùng phương tích đến hai đường trịn

( )

O và

( )

I . Hồn tồn tương tự với
điểm N. Từ đó suy ra MN chính là trục đẳng phương của hai đường trịn

( )

O và

( )

I


và theo các lập luận trên thì ta có dễ dàng có được điều phải chứng minh.


<b>Bài 90.</b>Cho tam giác ABC nội tiếp đường tròn

( )

O có AB AC và I tâm đường tròn
nội tiếp tam giác. Gọi D, E lần lượt là giao điểm của tia AI với BC, đường trịn

( )

O .


Đường thẳng qua I vng góc với AI cắt BC tại K và KA, KE cắt lại đường tròn

( )

O


theo thứ tự tại M, N. Các tia ND, NI cắt lại đường tròn

( )

O lần lượt tại Q, P. Chứng
minh rằng PM PQ= .


<b>Lời giải</b>


K


O
I


N



M <sub>Q</sub>


P


E


D C
B


A


Do I là tâm đường trịn nội tiếp nên ta có EIB IAB IBA 1

(

BAC ABC

)


2


= + = + . Mặt khác


</div>
<span class='text_page_counter'>(123)</span><div class='page_container' data-page=123>

góc BAC, do đó ta có IBE IBE CBE 1

(

ABC BAC

)


2


= + = + . Do vậy ta có BIE EBI= hay


tam giác BEI cân tại E. Chứng minh tương tự ta được EB EC EI= = nên E là tâm


đường tròn ngoại tiếp tam giác BIC. Do KI vuông góc với AI nên KI trở thành tiếp
tuyến tại I của đường tròn ngoại tiếp tam giác BIC. Đồng thời KBC trở thành cát tuyến
của đường tròn ngoại tiếp tam giác BIC. Từ đó dễ thấy hai tam giác KBI và KIC đồng
dạng với nhau nên ta được KI2 =KB.KC. Để ý rằng ta có tứ giác BCEN nội tiếp đường


tròn

( )

O nên ta lại có KB.KC KN.KE= . Do đó ta được <sub>KI</sub>2 =<sub>KN.KE</sub><sub> hay ta có </sub>


KI KN


KE= KI nên tam giác KIE đồng dạng với tam giác KNI. Điều này dẫn đến


0


NKI=KIE 90= hay tam giác INE vng tại N.


Ta có PNE=900 nên PE là đường kính của đường trịn

( )

O . Khi đó hai tam giác BDE


và ABE có góc AEB chung và BAD CAE CBE= = nên hai tam giác đó đồng dạng với
nhau. Từ đó ta suy ra được BE DE


AE = BE hay


2


BE =AE.DE. Để ý rằng tam giác KIE
vuông tại I có đường cao IN nên ta có BE2 =IE2 =EN.EK, do đó ta suy ra được


EA.ED EN.EK= . Điều này dẫn đến tứ giác ADNK nội tiếp đường trịn. Từ đó suy ra


MAE=DNE nên hai cung nhỏ ME và QE của đường tròn

( )

O có số đo bằng nhau.
Do vậy hai cung nhỏ PM và PQ của đường tròn

( )

O có số đo bằng nhau. Đến đây ta
suy ra được PM PQ= <i>. </i>


<b>Bài 91.</b>Cho tam giác ABC nhọn có các đường phân giác trong AD, BE, CF cắt nhau tại
O. Chứng minh rẳng nếu bán kính đường tròn nội tiếp các tam giác AOF, BOD, COE
bằng nhau thì tam giác ABC là tam giác đều.



</div>
<span class='text_page_counter'>(124)</span><div class='page_container' data-page=124>

Gọi O ; O ; O<sub>1</sub> <sub>2</sub> <sub>3</sub> lần lượt là tâm đường tròn nội
tiếp các tam giác AOF, BOD, COE. Giả sử


1 2 3


OO ; OO ; OO lần lượt cắtAB, BC, CA lại


M, N, P.


Do bán kính đường trịn nội tiếp các tam giác
AOF, BOD, COE nên ta được O O<sub>1</sub> <sub>2</sub> song song


với AD; O O1 3 song song với CE và O O2 3


song song với BE.


O<sub>3</sub>
O2


O<sub>1</sub>


K


H


O P


F



D


E


N
M


C
B


A


Mặt khác ta có MO1 NO2 PO3


MO = NO = OP do đó ta được O O1 2 song song với MN; O O1 3


song song với MP và O O<sub>2</sub> <sub>3</sub> song song vớiCF. Từ đó ta được MN song song với AD,
NP song song với BE và MP song song với CF. Gọi giao điểm của MO và NP là H,
giao điểm của BO và MN là K. Ta có MO MK


OH = NK , mặt khác ta có


MK KN


AO = DO nên ta


được MK AO


KN = DO. Do đó ta được



MO AO


HO = DO. Theo tính chất đường phân giác trong


tam giác ta có AO AB AC AB AC


DO BD DC BC


+


= = =


Mặt khác ta có MNO NOD; PNO= =BON mà BON=NOD nên ta được


MNO PNO= , suy ra No là tia phân giác của MNP. Tương tự ta được MO là phân


giác của NMP nên O là tâm đường tròn nội tiếp tam giác tam giác MNP.


Theo tính chất đường phân giác của tam giác ta có MO MN MP MN MP


HO HN HP NP


+


= = =


Từ đó ta được AB AC MN MP


BC NP



+ +


= nên ta được AB AC BC MN MP NP


BC NP


+ + + +


=


Hay NP MN MP NP


BC AB AC BC


+ +
=


+ + . Tương tự ta được


MN MP MN MP NP


AB AC AB AC BC


+ +


= =


+ + . Do đó ta


được NP MN MP



</div>
<span class='text_page_counter'>(125)</span><div class='page_container' data-page=125>

<b>Bài 92. </b>Cho tam giác ABC có AB c; BC a; CA b= = = . Chứng minh rằng tam giác ABC
vuông tại B hoặc C khi và chỉ khi một trong các đẳng thức sau đây xẩy ra:


a) tanA a


2 = b c+ b)


2 A b c


tan


2 b c



=


+


<b>Lời giải</b>


+ Trước hết ta chứng minh. Nếu tam giác ABC vng tại
B hoặc C thì ta ln có tanA a


2 = b c+ và


2 A b c


tan



2 b c



=


+


Thật vậy, xét tam giác ABC vuông tại B(trường hợp tam
giác ABC vuông tại C chứng minh tương tự) .


Khi đó gọi AD là đường phân giác của tam giác ABC.


Trong tam giác BAD vuông tại B có tanA BD
2 =BA


Theo tính chất đường phân giác của tam giác ta có


BD CD BD CD BC a


AB AC AB AC AB AC b c


+


= = = =


+ + +


K
F



E


D C


B
A


Do đó ta được tanA a


2 = b c+ . Từ


A a


tan


2 = b c+ được

(

) (

)



2 2 2


2


2 2


A a b c b c


tan


2 <sub>b c</sub> <sub>b c</sub> b c


− −



= = =


+


+ +


Do đó ta được tanA a
2 = b c+ và


2 A b c


tan


2 b c



=


+


+ Ta chứng minh: Nếu có tanA a


2 = b c+ hoặc


2 A b c


tan


2 b c




=


+ thì tam giác ABC vuông


tại B hoặc C.


Thật vậy, gọi AD là đường phân giác của tam giác ABC. Trên tia đối của tia AB lấy
điểm E sao cho AE AC b= = .


Khi đó ta có BAD=DAC=ACE=AEC nên ta được AD//EC. Kẻ BK vng góc với BE
cắt đường thẳng EC tại K.


Ta có tanA tan E BK BK
2 = = BE = b c+ .


</div>
<span class='text_page_counter'>(126)</span><div class='page_container' data-page=126>

a) Nếu có tanA a


2 = b c+ thì từ


A BK BK


tan tan E


2 = = BE =b c+ ta được BK a BC= = do đó K


trùng với điểm C. Điều này có nghĩa là tam giác ABC vuông tại B.
Vậy khi tanA a



2 =b c+ thì tam giác ABC vng tại B hoặc C.


b) Nếu<sub>tan</sub>2 A b c


2 b c



=


+ , khi đó giả sử b c kết hợp với


A BK BK


tan tan E


2 = = BE = b c+ thì ta


thu được


(

)

(

(

)(

)

)



2


2 2 2


2 2


b c b c
BK



BK b c


b c b c


− +


=  = −


+ + . Mặt khác ta có


2 2 2


AK =BK +c nên


ta được AK2=b2=AC2. Từ đó suy ra K trùng với C. Điều này có nghĩa là ABC 90= 0
hay tam giác ABC vuông tại B.


Vậy khi <sub>tan</sub>2 A b c


2 b c



=


+ thì tam giác ABC vng tại B hoặc C.


<b>Bài 93. </b>Cho tam giác ABC có G là trọng tâm và I là giao điểm ba đường phân giác.
Chứng minh rằng GI song song với BC khi và chỉ khi tan .tanB C 1


2 2 =3.



<b>Lời giải</b>


G
I
E
D
M


B C H


A


Hạ AH⊥BC, GD⊥BC, IE⊥BC. Do đó ta được GDsong song với IE.


Gọi độ dài các cạnh của tam giác ABC là a, b, c và đặt p a b c
2
+ +


= . Gọi r là khoảng


cách từ I đến các cạnh của tam giác ABC. Khi đó ta chứng minh được


BE p b; CE p c= − = − .Ta có

(

)



(

)(

)(

)



2


r p a p



B C IE IE r r


tan .tan . .


2 2 BE CE p b p c p p a p b p c




= = =


− − − − −


</div>
<span class='text_page_counter'>(127)</span><div class='page_container' data-page=127>

Ta được

(

)



(

)(

)(

)

(

)



2


2


a


r p a p S.r. p a <sub>p.r r.a</sub>


B C r.a 2r


tan .tan 1 1


2 2 p p a p b p c S S S h



− − <sub>−</sub>


= = = = − = −


− − −


+ Ta chứng minh nếu tan .tanB C 1


2 2 = 3 thì GI song song với BC.


Thật vậy, khi tan .tanB C 1


2 2 = 3 ta suy ra <sub>a</sub>


2.r 1 IE 1


1


h 3 AH 3


− =  =


Gọi M là trung điểm của BC, khi đó ta có MG 1


MA= 3


Do đó ta được MG GD 1


MA=AH = 3, suy ra



IE DG


IE GD


AH= AH =


Suy ra tứ giác GIED là hình bình hành. Do đóta được GI//BC.
+ Ta chứng minh nếu GIsong song với BC thì tan .tanB C 1


2 2 =3


Thật vậy, khi GI song song với BC thì ta có IE GD= . Theo định lí Thales ta có


MG GD IE 1


MA=AH=Ah =3. Do đó ta được <sub>a</sub>
2.r 1
1


h 3


− = hay tan .tanB C 1
2 2 = 3.


Vậy bài toán được chứng minh.


<b>Bài 94.</b>Cho tam giác nhọn ABC. Dựng bên ngoài tam giác ABC các tam giác cân DAC,


EAB, FBC sao cho DA DC, EA EC, FB FC= = = và ADC=2A, AEA=2B, CFB 2C= .



Gọi M là giao điểm của BD và EF,N là giao điểm của EC và DF, P là giao điểm của FA
và DE. Chứng minh rằng BD CE AF 4


MD+NE+ PF = .


<b>Lời giải</b>


Từ ADC=2A, AEA=2B, CFB 2C= ta suy ra


0 0 0


ADC 180 ; AEB 180 ; BFC 180  


Do đó ta được <sub>DAC 90</sub>0 1<sub>ADC 90</sub>0 <sub>A</sub>


2


= − = − ,


0 1 0


BAE 90 AEB 90 B


2


= − = −


Suy ra 0 0



DAE=DAC A BAE 180+ + = − B 90


Tương tự ta được EBF 90 ; FCD 90 0  0. Do đó


O


F
N
M


P
E


D


C
B


</div>
<span class='text_page_counter'>(128)</span><div class='page_container' data-page=128>

đa giác ADCFBE là đa giác lồi


Suy ra ta có ADC AEB BFC+ + =2 A B C

(

+ +

)

=3600.


Dựng các đường tròn

(

D; AD

)

(

E; EB

)

cắt nhau tại O, suy ra DA DC OD= =


Do đó ta được AOC 360= 0−2ADC 460= 0−4A.


Lại có AE EB EO= = nên ta được 0


AOB 360= −4B



Suy ra 0

( )



AOC AOB 720+ = −4 A B+ =4C


Do đó ta được 0

(

)

0


BOC=360 − AOB AOC+ =360 −4C suy ra FB FC FO= =


Do đó ba đường trịn

(

D; AD

)

,

(

E; EB

)

(

F; FB

)

cắt nhau tại O.


Từ đó ta được hai điểm O và C đối xứng qua DF, hai điểm O và A đối xứng qua DE,
hai điểm O và B đối xứng qua EF.


Điều này dẫn đến S<sub>AED</sub> =S<sub>OED</sub>; S<sub>BEF</sub>=S<sub>OEF</sub>; S<sub>CDF</sub> =S<sub>ODF</sub>


Ta lại có BEF OEF


DEF DEF


S
S


BD MD MB MB


1 1 1


MD MD MD S S


+



= = + = + = +


Hoàn toàn tương tự ta được ODF ODE


EDF FDE


S S


CE AF


1 ; 1


NE= + S PF = + S


Do đó ta được OEF ODF ODE


DEF


S S S


BD CE AF


3 4


MD NE PF S


+ +


+ + = + = . Bài toán được chứng minh



<b>Bài 95.</b> Cho ABC nhọn (AB < AC) có AH ⊥ BC tại H. Gọi D, E lần lượt là hình chiếu
vng góc của H trên AB và AC. Đường thẳng DE cắt tia CB tại S. Đường thẳng SA
cắt đường trịn đường kính AH tại M. Các đường thẳng BM và AC cắt nhau tại F.
Chứng minh rằng FA.FC SB.SC SF+ = 2


</div>
<span class='text_page_counter'>(129)</span><div class='page_container' data-page=129>

Ta có ADH=AEH 90= 0 nên tứ giác ADEH
nội tiếpđường tròn. Mặt khác C=AHE và
AHE ADE= nên C=ADE, do đó ta được


0


C EDB 180+ = nên tứ giác BDEC nội tiếp.


Ta có 0


C EDB 180+ = mà 0


SDB EDB 180+ =
nên C SDB 180= = 0 mà lại có S chung, do


đó hai tam giác SDB và SCE đồng dạng với


nhau. Suy ra SB SD


SE= SChay SE.SD SB.SC=


M


S H



N
F


D


E


C
B


A


Chứng minh tương tự ta được SD.SE SM.SA= . Suy ra SB.SC SM.SA= hay SB SM
SA= SC .
Lại có BSM chung nên hai tam giác SMB và SCA đồng dạng với nhau, do đó ta được


C SMB= . Mà ta có 0


SMB AMB 180+ = nên 0


C AMB 180+ = suy ra tứ giác AMBC nội
tiếp đường tròn. Chứng minh tương tự ta được hai tam giác FMA và FBC đồng dạng
với nhau nên FA FM


FB = FC hay FA.FC FM.FB=


Trên SF lấy N sao cho FNM=MBS ta có 0


FNM MNS 180+ = nên 0



MBS MNS 180+ = .


Lại có MBH MAC 180+ = 0 mà MBH MBS 180+ = 0 nên ta được MBS MAC= mà
FNM=MBS nên ta được FNM MAC= . Từ đóta được tứ giác FAMN nội tiếp đường


tròn. Tương tự ta được FNM∽ FBS nên FN.FS FM.FB= và SNM ∽ SAF nên
SN.SF SM.SA= .Từ đó ta được


(

)

2


FA.FC SB.SC FM.FB SM.SA FN.FS SN.SF FS FS SN+ = + = + = + =SF


<b>Bài 96.</b>Cho tam giác ABC nhọn nội tiếp đường tròn (O). Gọi A’, B’, C’ lầ lượt là điểm
chính giữa các cung nhỏ BC; CA; AB của đường tròn (O). Gọi E, Q lần lượt là giao
điểm của B’C’ với AB, AC; M, F lần lượt là giao điểm của A’C’ với BC, AB; P, N lần
lượt là giao điểm của A’B’ với AC, BC. Gọi I là giao điểm của MQ, NE, PF. Chứng
minh rằng S<sub>IMN</sub>+S<sub>IPQ</sub>+S<sub>IEF</sub> S<sub>INP</sub>+S<sub>IQE</sub>+S<sub>IMF</sub>


</div>
<span class='text_page_counter'>(130)</span><div class='page_container' data-page=130>

Gọi I là tâm đường tròn nội tiếp tam giác ABC. Khi đó ba đường thẳng AA’, BB’, CC’
đồng quy tại I.


Thật vậy, sử dụng tính chất góc nội tiếp và góc có đỉnh nằm bên trong đường trịn ta


có B' AI 1sdB' A' 1sdB'C 1sdA'C


2 2 2


= = + và B'IB 1sdB' A 1sdA' B


2 2



= +


Mà ta lại có sdB' C sdB' A; sdA' C sdA' B= =


Từ đó ta được B' AI=B'IA nên tam giác B’IA
cân tại B’, suy ra B' I=B' A. Chứng minh tương
tự ta cũng được C'I C' A= . Từ đó suy ra B’C’ là
đường trung trực của đoạn thẳng AI. Lặp lại cách
chứng minh như trên ta được C’A’ và A’B’ lần
lượt là đường trung trực của đoạn thẳng IB, IC.
Theo tính chất điểm nằm trên đường trung trực
của đoạn thẳng ta thu được MB MI= và FB FI= .


O
I
F


E Q


P


N
M


C'


C
B'



B


A'
A


Từ đó suy ra tam giác BMI cân tại M và tam giác FBI cân tại F. Đến đây ta suy ra được


MBI=MIB; FBI=FIB. Mà ta lại có MBI=FBI nên ta được FBI=MIB MBI= =FIB, suy
ra IM song song với BF và IF song song với BM. Kết hợp với MBI=FBI ta suy ra được
tứ giác BMIF là hình thoi. Chứng minh hoàn toàn tương tự ta được các tứ giác AQIE


và CNIP là hình thoi. Từ đó ta được IFsong song với BC và IP song song với CN nên
IP song song với BC, suy ra FP đi qua điểm I.Chứng minh tương tự ta được QM, NE
cũng đi qua điểm I.Vậy các đường thẳng MQ, NE, PF đồng quy tại điểm I.


Sử dụng tính chất về tỉ số diện tích của hai tam giác đồng dạng ta được. Chứng minh
được hai tam giác IMN và ABC đồng dạng nên


2


IMN
IMN


ABC <sub>ABC</sub>


S


S MN MN


S BC <sub>S</sub> BC



 


=<sub></sub> <sub></sub>  =


  .


Hai tam giác IPQ và BCA đồng dạng với nhau nên IPQ


BCA


S <sub>IP</sub> <sub>NC</sub>


BC BC


S = =


Hai tam giác IEF và CAB đồng dạng với nhau nên IEF


CAB


S IF BM


BC BC


</div>
<span class='text_page_counter'>(131)</span><div class='page_container' data-page=131>

Từđó ta được IMN IPQ IEF


ABC BCA CAB


S



S S MN NC BM BC


1


BC BC BC BC


S + S + S = + + = = .


Hay ta đươch S<sub>IMN</sub> + S<sub>IPQ</sub> + S<sub>IEF</sub> = S<sub>ABC</sub> .


Áp dụng bất đẳng thức Bunhiacopxki ta được


(

)

(

)

2


IMN IPQ IEF IMN IPQ IEF ABC


3 S +S +S  S + S + S =S


Lại có S<sub>ABC</sub> =

(

S<sub>IMN</sub>+S<sub>IPQ</sub>+S<sub>IEF</sub>

) (

+ S<sub>AQIE</sub>+S<sub>BMIF</sub>+S<sub>CNIP</sub>

)



Mà các tứ giác AQIE, BMIF, CNIP là các hình thoi nên ta được


(

) (

)



ABC IMN IPQ IEF INP IQE IMF


S = S +S +S +2 S +S +S


Từ đó ta được 3 S

(

<sub>IMN</sub>+S<sub>IPQ</sub>+S<sub>IEF</sub>

) (

 S<sub>IMN</sub>+S<sub>IPQ</sub>+S<sub>IEF</sub>

) (

+2 S<sub>INP</sub>+S<sub>IQE</sub>+S<sub>IMF</sub>

)




Hay ta được S<sub>IMN</sub>+S<sub>IPQ</sub>+S<sub>IEF</sub>S<sub>INP</sub>+S<sub>IQE</sub>+S<sub>IMF</sub>.


<b>Bài 97.</b>Cho tam giác ABC không cân tại A và có các góc ABC, ACB là các góc nhọn.
Xét điểm D di động trên cạnh BC sao cho D khơng trùng với B, C và hình chiếu của A
trên BC. Đường thẳng d vng góc với BC tại D cắt đường thẳng AB, AC lần lượt tại
E và F. Gọi M, N, P lần lượt là tâm đường tròn nội tiếp các tam giác AEF, BDE, CDF.
Chứng minh rằng bốn điểm A, M, N, P cùng nằm trên một đường tròn khi và chỉ khi
d đi qua tâm đường tròn nội tiếp tam giác ABC.


<b>Lời giải</b>
Để chứng minh bài toán ta tách ra chứng minh hai ý


sau:


+ Trước hết ta chứng minh d đi qua tâm đường tròn
nội tiếp tam giác ABC khi và chỉ khi tiếp tuyến
chung khác d của hai đường tròn

( )

N và

( )

P đi qua
điểm A.


• Giả sử đường thẳng d đi qua điểm tâm đường
tròn nội tiếp tam giác ABC, khi đó do D tà tiếp điểm
của đường trịn nội tiếp tam giác ABC với BC nên ta


P
I
F


T
E



N
d


D C


B


</div>
<span class='text_page_counter'>(132)</span><div class='page_container' data-page=132>

được ta có BD AB BC AC; CD CA CB AB


2 2


+ − + −


= =


Từ đó suy ra BD DC BA BC AC CA CB AB AB AC


2 2


+ − + −


− = − = −


Gọi giao điểm của tiếp tuyến đi qua A (khác AB) của đường tròn

( )

N với đường
thẳng là T. Khi đó tứ giác TABD ngoại tiếp đường tròn nên AB TD+ =AT BD+ . Kết
hợp với đẳng thức trên ta được AC TD AT CD+ = + . Điều này chứng tỏ tứ giác TACD
ngoại tiếp đường tròn. Vậy AT tiếp xúc với đường trịn

( )

P hay nói cánh khác AT là


tiếp tuyến chung của hai đường tròn

( )

N và

( )

P đi qua điểm A.


• Giả sử tiếp tuyến khác d của hai đường tròn

( )

N và

( )

P đi qua A. Gọi giao điểm
của tiếp tuyến đó với đườngthẳng d là T. Khi đó dễ thấy các tứ giác TABD và TACD
ngoại tiếp đường trịn. Do đó AB TD+ =AT BD+ và AC TD AT DC+ = + . Từ đó ta
được BD DC AB AC− = − .


Lại có BD CD BC+ = nên ta được BD AB BC AC; CD CA CB AB


2 2


+ − + −


= =


Từ đó suy ra D là tiếp điểm của đường trịn nội tiếp tam giác ABC với cạnh BC. Từ dó
suy ra d đi qua tâm đường tròn nội tiếp tam giác ABC.


+ Tiếp theo ta chứng minh rằng bốn điểm A, M, N,
P cùng nằm trên một đường tròn khi và chỉ khi tiếp
tuyến chung khác d của hai đường tròn

( )

N và


( )

P đi qua điểm A.Ta chứng minh hai chiều sau:
• Giả sử tiếp tuyến chung của hai đường tròn

( )

N

( )

P đi qua A, ta cần chứng minh bốn điểm A,
M, N, P cùng nằm trên một đường tròn.


Thật vậy, để ý là các ba điểm E, M, N thẳng hàng
và ba điểm F, M, P thẳng hàng.


M


A


B D C


d


N
E


T
F


I


P


Do đó ta có


0


0 0 0 EAF 0 180 BAC BAC


NMP 180 EMF 180 90 90


2 2 2


  <sub>−</sub>


= − = −<sub></sub> + <sub></sub>= − =



</div>
<span class='text_page_counter'>(133)</span><div class='page_container' data-page=133>

Mặt khác do tiếp tuyến chung khác d của hai đường tròn

( )

N và

( )

P đi qua điểm A


nên NAP 1BAC
2


= Mặt khác do tiếp tuyến chung khác d của hai đường tròn

( )

N và


( )

P đi qua điểm A nên NAP 1BAC
2
=


Do đó ta được NMP=NAP nên tứ giác AMNP nội tiếp đường trịn.


• Giả sử ta bốn điểm A, M, N, P cùng nằm trên một đường tròn. Ta cần chứng minh
tiếp tuyến chung khác d của hai đường tròn

( )

N và

( )

P đi qua A.


Thật vậy, lập luận như trên ta được NMP BAC
2


= . Do tứ giác AMNP nội tiếp đường
tròn nên ta được NMP=NAP. Từ đó suy ra NAP 1BAC


2


= . Qua A vẽ tiếp tuyến Ax


của đường tròn

( )

N , khi đó ta được NAx 1BAx
2
=



Do đó ta được APx NAP NAx 1BAC 1BAx 1CAx PAC


2 2 2


= − = − = =


Từ đó suy ra Ax đối xứng với AC qua AP, mà AC tiếp xúc với đường tròn (P) nên Ax
cũng tiếp xúc với đườn tròn

( )

P . Suy ra tiếp tuyến chung khác d của hai dường tròn


( )

N và

( )

P đi qua điểm A.


Vậy từ hai ý trên ta được bốn điểm A, M, N, P cùng nằm trên một đường tròn khi và
chỉ khi d đi qua tâm đường trònnội tiếp tam giác ABC.


<b>Bài 98.</b>Cho tam giác ABC có trực tâm H và nội tiếp đường tròn

( )

O . Một tiếp tuyến
thay đổi của đường tròn

( )

O cắt HB, HC theo thứ tự tại E, F. Điểm K đối xứng với H
qua EF. Chứng minh đường tròn ngoại tiếp tam giác KEF luôn tiếp xúc với một
đường tròn cố định.


<b>Lời giải</b>


</div>
<span class='text_page_counter'>(134)</span><div class='page_container' data-page=134>

<i>của các cặp đường thẳng AB và DE, BC và EF, CD và AF. Chứng minh rằng ba điểm G, H, K </i>
<i>thẳng hàng khi và chỉ khi điểm F nằm trên đường trịn </i>

( )

O <i>. </i>


<i><b>Chứng minh bài tốn phụ.</b></i>


<b>+ Điều kiện cần.</b>Giả sử điểm F nằm trênđường tròn

( )

O .


Gọi I là giao điểm của hai đường trong ngoại
tiếp tam giác GBD và tam giác KDF (I khác


D). Khi đó ta có


(

) (

)



(

) (

)



BIF BID DIF BGD DKF


1 1


sdAFE sdBCD sdABC sdCDF


2 2


1 1


sdCAE sdBCF sdBAF sdCDE


2 2


= + = +


= − + −


= − = −


Mà ta có BHF 1

(

sdBAF sdCDE

)


2


= − .



O


I H
K


G
F


E
D


C
B
A


Do đó BIF BHF= nên tứ giác BIHF nội tiếp đường tròn, suy ra BIH BFH 180+ = 0. Mặt


khác ta có BIG BDG 1sdBG
2


= = . Mà BDG=BFE nên 0


BIH BIG+ =BIH BFH 180+ = ,


suy ra 0


HIG 180= . Do đó ba điểm H, I, G thẳng hàng. Chứng minh tương tự ta được
I, H, K thẳng hàng. Như vậy bốn điểm H, I, G, K thẳng hàng. Do đó ba điểm G, H, K
thẳng hàng.



<b>+ Điều kiện đủ.</b> Giả sử ba điểm H, G, K thẳng hàng. Gọi T là giao điểm của EH với


</div>
<span class='text_page_counter'>(135)</span><div class='page_container' data-page=135>

<b>Trở lại bài toán.</b> Gọi P là tiếp điểm của EF


với đường tròn

( )

O . Gọi M, N theo thứ tự là
giao điểm của BH, CH với đường trịn

( )

O .


Khi đó dễ thấy M, N đối xứng với H theo
thứ tự qua AC và AB. Do đó đường trịn
tâm A bán kính AH đi qua các điểm M và
N. Do A và H cố đinh nên đường tròn

( )

A


cố định. Gọi V, U theo thứ tự là giao điểm


thứ hai của PN, PM với đường tròn

( )

A .


Q


H


P
N


M
V


U
T



K


F
O


E


C
B


A


Gọi T là giao điểm của EV và FU. Ta có các điểm H, M, N, U, V cùng nằm trên
đường tròn

( )

A và để ý rằng ba điểm E, P, F thẳng hàng nên theo bài tốn phụ trên
thì T nằm trên đường tròn

( )

A . Do EF là tiếp tuyến tại P với đường tròn

( )

O nên ta
có NPE=NMP=NVU suy ra UV song song với EF. Để ý đến các tứ giác TVNH và


TMUH nội tiếp ta có 0


TVP 180= −THN và 0 0


TUP 180= −TUM 180= −THM. Do tính


đối xứng ta có MAN=2BAC 2 180=

(

0 −MHN

)

. Từ đó ta có biến đổi góc


(

) (

)



(

)

(

)



(

)




(

)

(

)



0 0


0 0 0


0 0


0 0 0 0


0 0 0


ETF 180 TVU TUV 180 TVP UVP TUP VUP


180 180 THN UVP 180 THM VUP


MHN 180 UVP VUP MHN 180 UVP VUP MHN MPN


MHN 180 MAN MHN MAN 180 MHN 2 180 MHN 180


180 MHN 180 BHC 180 EKF


= − − = − − − −


= − − + − − +


= − + + = − − − = −


= − − = + − = + − −



= − = − = −


Suy ra tứ giác TEKF nội tiếp đường tròn. Giả sử TQ là tiếp tuyến với đường trịn

( )

A
ta có QTV QTE TFE TUV= = = nên TQ cũng là tiếp tuyến của đường tròn ngoại tiếp
tam giác EKF. Vậy đường tròn ngoại tiếp tam giác EKF luôn tiếp xúc với đường trịn
tâm A bán kính AH cố định.


</div>
<span class='text_page_counter'>(136)</span><div class='page_container' data-page=136>

tam giác PCE đồng dạngvới tam giác BAO và tam giác PBF đồng dạng với tam giác
CAO. Tiếp tuyến tại P của đường tròn

( )

O cắt đường tròn ngoại tiếp các tam


giác PCE và PBF theo thứ tự tại M và N khác P. Gọi Q là giao điểm của EM và FN.


Chứng minh rằng đường tròn ngoại tiếp tam giác QMN ln tiếp xúc một đường trịn
cố định khi P thay đổi.


<b>Lời giải</b>


<b>Trước hết ta phát biểu và chứng minh các bài toán phụ. </b>


<b>Bài toán phụ 1.</b><i>Cho tứ giác ABCD có các cạnh AB và CD cắt nhau tại M, các cạnh AD và </i>
<i>BC cắt nhau tại N. Chứng minh rằng đường tròn ngoại tiếp các tam giác ABN, CDN, ADM, </i>
<i>BCM cùng đi qua một điểm. </i>


<b>Chứng minh. </b>Giả sử hai đường tròn ngoại
tiếp tam giác AD và CDN cắt nhau tại điểm E
khác D. Khi đó các tứ giác AMED và CDNE
nội tiếp đường trịn nên ta có MDE CNE= và


MAE MDE= .



Từ đó ta được MAE CNE= =BNE nên tứ giác
ABNE nội tiếp đường tròn. Mặt khác cũng từ
các tứ giác trên nội tiếp đường trịn ta có


ECN=EDN và EDN=AME.


E


N
M


D


C
B


A


Do đó ta được AME ECN ECM= = nên tứ giác BCEM nội tiếp đường tròn. Vậy


</div>
<span class='text_page_counter'>(137)</span><div class='page_container' data-page=137>

<b>Chứng minh.</b> Gọi S là giao điểm thứ hai của
đường tròn ngoại tiếp tam giác BFM và
đường tròn ngoại tiếp tam giác CEM. Khi đó
để ý đến các tứ giác nội tiếp đường tròn và
tiếp tuyến EF của đường tròn ngoại tiếp tam


giác BMC ta có biến đổi góc


(

)




(

)



0 0


0


0


0 0


ERF QRF QRE 180 ACQ 180 ABQ


360 ACQ ABQ BAC BQA


BAC 180 BMC BAC MBC MCB


180 MSE PSF 180 ESF


= + = − + −


= − + = +


= + − = + +


= − + = −


R


S



Q
L


F


E
M


O


C
B


A


Điều này dẫn đến tứ giác SERF nội tiếp đường trịn.


Mặt khác ta lại có 0


BSC=BSM CSM+ =AEF AFE 180+ = −BAC nên tứ giác ABSC nội
tiếp đường tròn. Gọi SL là tiếp tuyến tại S của đường trịn

( )

O . Khi đó ta có biến đổi


góc ESL=BSL ESB+ =BCS EMB+ =BCS MCB MCS+ = =MES. Từ đó SL cũng là tiếp
tuyến tại S của đường tròn ngoại tiếp tam giác ESF, điều này kéo theo SL là tiếp tuyến
của đường tròn ngoại tiếp tam giác REF. Vậy đường tròn ngoại tiếp tam giác REF tiếp


</div>
<span class='text_page_counter'>(138)</span><div class='page_container' data-page=138>

<b>Trở lại bài toán.</b> Do có tam giác PCE đồng
dạngvới tam giác BAO và tam giác PBF đồng
dạngvới tam giác CAO nên ta suy ra được



0


EPF BPF BPC CPE OAC BPC OAB
BAC BPC 180


= + + = + +


= + =


Gọi K và L lần lượt là giao điểm của BN với


CM và BF với CE. Do có tam giác PCE đồng
dạngvới tam giác BAO và tam giác PBF đồng
dạngvới tam giác CAO nên ta có


(

)

(

)



(

)



0 0


0 0


0


BLC 180 EFL FEL 180 2 PBF PCE


180 2 BAO CAO 180 2BAC



180 BOC


= − + = − +


= − + = −


= −


O


X


L
N


M
Q


K
F


E
P


C
B


A


Do đó ta được tứ giác tứ giác BOCL nội tiếp đường tròn. Để ý đến các tứ giác BFNP


và CMEP nội tiếp đường tròn ta lại có ECM EPM= =FPN FBN= nên tứ giác BCKL
nội tiếp đường tròn hay điểm K thuộc đường trònngoại tiếp tam giác BOC. Xét tứ
giác PDQN có F là giao điểm của EP với QN và M là giao điểm của NP với QE, nên


theo <b>bài tốn phụ 1</b>ta có đường trịn ngoại tiếp các tam giác FNP, EMP và QMN cùng


đi qua mộtđiểm, ta gọiđiểmđó là X. Khi đótừ các tứ giác BXPF, CXPE, BOPF, COPE
cùng nội tiếp đường tròn. Do vậy ta thu được biến đổi góc là


0 0


BXC=BXP CXP 180+ = −BFP 180+ −CEP=BOP COP+ =BOC nên X thuộc đường


tròn ngoạitiếp tam giác BOC. Xét tam giác KBC có P là điểmnằm trong tam giác, tiếp
tuyếntại P củađường tròn ngoạitiếp tam giác PBC cắt KB, KC tại N, M. Lại có điểm


X nằm trên đường trịn ngoại tiếp tam giác KBC, hai đường tròn ngoại tiếp hai tam
giác XBN và XCM cắt nhau tại Q. Khi đó theo <b>bài tốn phụ 2</b> thì đường trịn ngoại
tiếp tam giác QMN ln tiếp xúc với đường ngịn ngoại tiếp tam giác KBC hay ln


tiếp xúc vớiđường trịn ngoạitiếp tam giác BOC. Mà ta có tam giác BOC cốđịnh nên


đường tròn ngoại tiếp tam giác BOC cố định. Vậy đường tròn ngoại tiếp tam giác


QMN luôn tiếp xúc một đường tròn cố định.


</div>
<span class='text_page_counter'>(139)</span><div class='page_container' data-page=139>

CN. Một đường thẳng qua H cắt AC, BD theo thứ tự tại K, L. Trên BC lấy các điểm Q,
R sao cho KR song song với BM và LQ song song với CN. Gọi P là giao điểm của KR
với QL. Chứng minh rằng đường tròn ngoại tiếp tam giác PQR tiếp xúc với đường
tròn ngoại tiếp tam giác EBC.



<b>Lời giải</b>
<b>Trước hết ta phát biểu và chứng minh </b>


<b>bài toán phụ.</b><i>Cho tứ giác ABCD nội tiếp </i>
<i>đường trịn </i>

( )

O <i> có các cạnh AB và CD cắt </i>
<i>nhau tại M, các cạnh AD và BC cắt nhau tại </i>
<i>N. Khi đó đường tròn ngoại tiếp các tam </i>
<i>giác ABN, CDN, ADM, BCM cùng đi qua </i>
<i>một điểm E và ba điểm M, E, N thẳng hàng.</i>


O


E


N
M


D


C
B


A


<b>Chứng minh. </b>Giả sử hai đường tròn ngoại tiếp tam giác MAD và CDN cắt nhau tại
điểm E khác D. Khi đó các tứ giác AMED và CDNE nội tiếp đường tròn nên ta có


MDE CNE= và MAE MDE= , từ đó ta được MAE CNE= =BNE nên tứ giác ABNE
nội tiếp đường tròn. Mặt khác cũng từ các tứ giác trên nội tiếp đường trịn ta có



ECN EDN= và EDN=AME nên ta được AME ECN ECM= = nên tứ giác BCEM nội
tiếp đường tròn. Vậy đường tròn ngoại tiếp các tam giác ABN, CDN, ADM, BCM
cùng đi qua một điểm E. Do tứ giác ADEM và CDEN nội tiếp đường trịn nên ta có


0


MAD MED 180+ = và DCN DEN 180+ = 0. Mà ta lại có tứ giác ABCD nội tiếp đường


trịn nên MAD DCN 180+ = 0. Từ đó ta được MED NED 180+ = 0 hay ba điểm M, E, N
thẳng hàng.


</div>
<span class='text_page_counter'>(140)</span><div class='page_container' data-page=140>

T
S


R


Y


X


Q
P
D


E


H


K


L


N


M


C
B


A


Xét tứ giác XKYL có E là giao điểm của XK với YK và P là giao điểm của XY với LK,
khi đó theo bài tốn phụ trên thì các đường tròn ngoại tiếp các tam giác XEL, XKP,
KYE và YLP cùng đi qua một điểm S và ma điểm P, S, E thẳng hàng. Từ đó dễ thấy


XKS=XPS=LPS=LYS và KSX=KPX=YPL=YSL nên suy ra hai tam giác SKX và


SYL đồng dạng với nhau. Mặt khác theo do KR song song với BM và LQ song song
với CN nên theo định lý Thales ta có BL HL CX


BY=HK =CK nên ta có hai tam giác SCK và


SBY đồng dạng với nhau, hai tam giác SCX và SBL đồng dạng với nhau. Đến đây thì
ta thu được hai tam giác SCB và SKY đồng dạng với nhau. BSC KSY= . Để ý rằng tứ
giác SEYK nội tiếp đường trịn nên ta có BEC=YEK=YSK=BSC hay từ giác BESC
nội tiếp đường tròn. Cũng từ hai tam giác SCB và SKY đồng dạng với nhau ta được


SBC SYL= . Do đó CSK=BSY=CRK nên suy ra tứ giác SCKR nội tiếp đường tròn.
Lập luận chứng minh hồn tồn tương tự ta có tứ giác SXCQ nội tiếp đường trịn. Đến
đây thì ta có XQS XCS PRS= = nên tứ giác PQSR nội tiếp đường trịn. Điều này có



nghĩa là hai đường tròn ngoại tiếp hai tam giác PQR và BEC cùng đi qua điểm S. Giả
sử ST là tiếp tuyến của đường tròn ngoại tiếp tam giác PQR. Để ý đến các tứ giác nội


</div>

<!--links-->

×